You are on page 1of 172

AIPTS/ILP VETERANS -

Exam Title :
2020 T...
Email : sahil.law.jmi@gmail.com
Contact : 9540210113

Note: If the exam is multi-lingual i.e. English and Hindi. Hindi solutions will be
after the completion of English solutions.
QUESTION 1. NjYxMjUrTW9oYW1tYWQgWW91bmlzIEFobWFkK3NhaGlsLmxhdy5qbWlAZ21haWwuY29tKzk1
UVTVElPTiAw
From the below statements, identify the appropriate definition of statutory liquidity ratio:

a) It is a measure of the reserves that commercial banks are required to hold in the form of
government bonds, gold, and similar liquid assets.
b) It is a measure of the reserves that banks need to hold in the form of cash.
c) It is a measure of highly liquid assets which can easily be converted into cash that banks
are required to hold.
d) It is a specified minimum fraction of the total deposits of customers, which commercial
banks have to hold as reserves either in cash or as deposits with the central bank.
Correct Answer: A
Your Answer: A
Explanation

Solution (a)

Statement analysis:

Statement (a) Statement (a) Statement (a) Statement (a)

Correct Incorrect Incorrect Incorrect

SLR, or statutory Cash Reserve Ratio


liquidity ratio, is a LCR, or liquidity (CRR) is a specified
measure of the coverage ratio, is a minimum fraction of
CRR, or cash reserve
reserves that measure of highly the total deposits of
ratio, is a measure of
commercial banks are liquid assets which customers, which
the reserves that
required to hold in can easily be commercial banks
banks need to hold in
the form of converted into cash have to hold as
the form of cash.
government bonds, that banks are reserves either in
gold, and similar required to hold. cash or as deposits
liquid assets. with the central bank.

Do you know?

· SLR, CRR, and LCR - All three are policy tools used by the Reserve Bank of India (RBI) to
influence the total amount of reserves held by banks at any point in time.

· These reserve requirements influence the amount of loans that banks can extend to borrowers.

IASbaba
Score:
Web: http://ilp.iasbaba.com/
69.00 /
Email: ilp@iasbaba.com
Page 1 200
AIPTS/ILP VETERANS -
Exam Title :
2020 T...
Email : sahil.law.jmi@gmail.com
Contact : 9540210113

QUESTION 2. NjYxMjUrTW9oYW1tYWQgWW91bmlzIEFobWFkK3NhaGlsLmxhdy5qbWlAZ21haWwuY29tKzk1
UVTVElPTiAx
What happens when the RBI tightens reserve requirements?

1. Banks are forced to cut down lending

2. Money supply in the economy shrinks

3. Money supply in the economy rises

4. Rate of inflation is expected to decrease

Choose the correct answer:

a) 2 and 4 only
b) 1, 2 and 4
c) 3 only
d) 3 and 4 only
Correct Answer: B
Your Answer: B
Explanation

Solution (b)

When the RBI tightens reserve requirements, banks are forced to cut down lending and this
causes money supply in the economy to shrink.

Money supply rises when reserve requirements are eased by the RBI. This increases the
nation's money supply and expands the economy. But the increased spending activity can also
work to increase inflation.

Statement analysis:

Statements 1, 2 and 4 Statement 3

Correct Incorrect

When the RBI tightens reserve


requirements, banks are forced to cut down
lending and this causes money supply in the When the RBI tightens reserve
economy to shrink. requirements, the money supply in the
economy doesn’t rise, rather it shrinks.
Since the money supply is decreased, rate of
inflation is decreased.

QUESTION 3. NjYxMjUrTW9oYW1tYWQgWW91bmlzIEFobWFkK3NhaGlsLmxhdy5qbWlAZ21haWwuY29tKzk1
UVTVElPTiAy

IASbaba
Score:
Web: http://ilp.iasbaba.com/
69.00 /
Email: ilp@iasbaba.com
Page 2 200
AIPTS/ILP VETERANS -
Exam Title :
2020 T...
Email : sahil.law.jmi@gmail.com
Contact : 9540210113

Which of the following correctly defines the term “crowding out”?

a) It is an economic theory which believes that more expenditure in social security generates
economic growth.
b) It is an investment process under which general public is kept out.
c) It is an economic theory which believes that rise in public sector spending drive down or
even eliminate private sector spending.
d) It is an economic theory which suggests government borrowing can actually increase
demand by generating employment, thereby stimulates private spending.
Correct Answer: C
Your Answer: D
Explanation

Solution (c)

Explanation:

Crowding out effect Crowding in effect

The crowding out effect is an economic


theory arguing that rising public sector
spending drives down or even eliminates Crowding in, on the other hand, suggests
private sector spending. government borrowing can actually increase
demand by generating employment, thereby
The crowding out effect suggests rising stimulating private spending.
public sector spending drives down private
sector spending.

QUESTION 4. NjYxMjUrTW9oYW1tYWQgWW91bmlzIEFobWFkK3NhaGlsLmxhdy5qbWlAZ21haWwuY29tKzk1
UVTVElPTiAz
Which one of the following statements appropriately describes the “fiscal stimulus”?

a) It is a massive investment by the Government in manufacturing sector to ensure the supply


of goods to meet the demand surge caused by rapid economic growth
b) It is an intense affirmative action of the Government to boost economic activity in the
country
c) It is Government’s intensive action on financial institutions to ensure disbursement of loans
to agriculture and allied sectors to promote greater food production and contain food
inflation
d) It is an extreme affirmative action by the Government to pursue its policy of financial
inclusion
Correct Answer: B
Your Answer: B
Explanation

IASbaba
Score:
Web: http://ilp.iasbaba.com/
69.00 /
Email: ilp@iasbaba.com
Page 3 200
AIPTS/ILP VETERANS -
Exam Title :
2020 T...
Email : sahil.law.jmi@gmail.com
Contact : 9540210113

Solution (b)

Approach:

A ‘stimulus’ is an attempt by policymakers to kickstart a sluggish economy through a package


of measures.

Monetary stimulus Fiscal stimulus

A fiscal stimulus entails the Government


spending more from its own coffers or
A monetary stimulus will see the central slashing tax rates to put more money in the
bank expanding money supply or reducing hands of consumers.
the cost of money (interest rates), to spur
consumer spending. Fiscal stimulus is an affirmative action taken
by a government in order to encourage and
support economic growth.

QUESTION 5. NjYxMjUrTW9oYW1tYWQgWW91bmlzIEFobWFkK3NhaGlsLmxhdy5qbWlAZ21haWwuY29tKzk1
UVTVElPTiA0
Consider the following statements with reference to Board for Financial Supervision (BFS) –

1. It is a specialized independent agency that manages the internal and external liabilities of the
Central Government in a holistic manner and advises on such matters in return for a fee.

2. It is also called as the Investment Banker or Merchant Banker to the Government.

Which of the statements given above is/are correct?

a) 1 only
b) 2 only
c) Both 1 and 2
d) Neither 1 nor 2
Correct Answer: D
Your Answer: D
Explanation

Solution (d)

Statement analysis:

Statement 1 Statement 2

Incorrect Incorrect

IASbaba
Score:
Web: http://ilp.iasbaba.com/
69.00 /
Email: ilp@iasbaba.com
Page 4 200
AIPTS/ILP VETERANS -
Exam Title :
2020 T...
Email : sahil.law.jmi@gmail.com
Contact : 9540210113

“Public Debt Management Agency” (PDMA)


Board for Financial Supervision (BFS), is also called as the Investment Banker or
which was constituted in November 1994, is Merchant Banker to the Government.
as an autonomous body under the RBI.
Board for Financial Supervision (BFS)
The Reserve Bank of India performs the supervises the money market institutions in
supervisory function under the guidance of the country.
the Board for Financial Supervision (BFS).
The primary objective of BFS is to
The proposed “Public Debt Management undertake consolidated supervision of the
Agency” (PDMA) is a specialized financial sector comprising Scheduled
independent agency that manages the Commercial and Co-operative Banks, All
internal and external liabilities of the India Financial Institutions, Local Area
Central Government in a holistic manner Banks, Small Finance Banks, Payments
and advises on such matters in return for a Banks, Credit Information Companies, Non-
fee. Banking Finance Companies and Primary
Dealers.

QUESTION 6. NjYxMjUrTW9oYW1tYWQgWW91bmlzIEFobWFkK3NhaGlsLmxhdy5qbWlAZ21haWwuY29tKzk1
UVTVElPTiA1
Which among the following is/are the basic function(s) of the Reserve Bank?

1. to regulate the issue of Bank notes

2. to operate the currency and credit system of the country to its advantage

3. to maintain price stability

Choose the appropriate code:

a) 1 only
b) 2 only
c) 1 and 2
d) 1, 2 and 3
Correct Answer: D
Your Answer: D
Explanation

Solution (d)

Explanation:

The Preamble of the Reserve Bank of India describes the basic functions of the Reserve Bank
as:

"to regulate the issue of Bank notes and keeping of reserves with a view to securing monetary
stability in India and generally to operate the currency and credit system of the country to its
advantage; to have a modern monetary policy framework to meet the challenge of an

IASbaba
Score:
Web: http://ilp.iasbaba.com/
69.00 /
Email: ilp@iasbaba.com
Page 5 200
AIPTS/ILP VETERANS -
Exam Title :
2020 T...
Email : sahil.law.jmi@gmail.com
Contact : 9540210113

increasingly complex economy, to maintain price stability while keeping in mind the objective of
growth."

QUESTION 7. NjYxMjUrTW9oYW1tYWQgWW91bmlzIEFobWFkK3NhaGlsLmxhdy5qbWlAZ21haWwuY29tKzk1
UVTVElPTiA2
Which of the below statements is/are true with regard to Non- Scheduled Banks?

1. They do not follow the rules made by the RBI.

2. They are not eligible for having loans from the RBI for day to day activities.

3. Cash Reserve Ratio conditions do not apply to them.

Choose appropriate code:

a) 1 only
b) 2 only
c) 1 and 2
d) 1, 2 and 3
Correct Answer: C
Your Answer: B
Explanation

Solution (c)

Statement analysis:

Statement 1 Statement 2 Statement 3

Correct Correct Incorrect

Non- Scheduled Banks have


Non- Scheduled Banks are to follow CRR conditions.
also not eligible for having
Non-scheduled banks do not
loans from the RBI for day to However, these banks can
follow the rules made by the
day activities but under the have CRR fund with
RBI.
emergency conditions RBI themselves as no compulsion
can grant loan to them. has been made by the RBI to
deposit it in the RBI.

Do you know?

Key differences between the Scheduled Banks and Non- Scheduled Banks are

1. Scheduled banks follow the rules made by the RBI while Non-scheduled banks do not follow
the rules made by the RBI.

IASbaba
Score:
Web: http://ilp.iasbaba.com/
69.00 /
Email: ilp@iasbaba.com
Page 6 200
AIPTS/ILP VETERANS -
Exam Title :
2020 T...
Email : sahil.law.jmi@gmail.com
Contact : 9540210113

2. Scheduled banks are eligible for inclusion in the second schedule to the Reserve Bank of
India Act, 1934 while Non-scheduled banks are not included in the second schedule.

3. Scheduled banks are allowed to borrow money from RBI for regular banking purposes while
Non-scheduled banks are not allowed.

4. Scheduled banks can become a member of clearing house while Non-scheduled banks can't.

5. Scheduled banks and Non-scheduled banks both need to maintain the Cash Reserve Ratio but
Scheduled banks have to deposit this amount in the RBI while Non-scheduled banks can deposit
this amount with themselves.

QUESTION 8. NjYxMjUrTW9oYW1tYWQgWW91bmlzIEFobWFkK3NhaGlsLmxhdy5qbWlAZ21haWwuY29tKzk1
UVTVElPTiA3
Mutually advantageous relationships between business leaders and government officials is
known as –

a) Corporatocracy
b) Jingoism
c) Nepotism
d) Crony capitalism
Correct Answer: D
Your Answer: D
Explanation

Solution (d)

Approach:

Corporatocracy sounds similar, but differs in the fact that here big businesses and its leaders
control governments.

Analysis/Explanation:

Corporatocracy Jingoism Nepotism Crony capitalism

IASbaba
Score:
Web: http://ilp.iasbaba.com/
69.00 /
Email: ilp@iasbaba.com
Page 7 200
AIPTS/ILP VETERANS -
Exam Title :
2020 T...
Email : sahil.law.jmi@gmail.com
Contact : 9540210113

Jingoism is
nationalism in the
form of aggressive
foreign policy, such as
a country's advocacy
Crony capitalism is an
for the use of threats Nepotism is the
economy in which
Corporatocracy is a or actual force, as granting of jobs to
businesses thrive not
term used to refer to opposed to peaceful one's relatives in
as a result of risk, but
an economic and relations, in efforts to various fields,
rather as a return on
political system safeguard what it including business,
money amassed
controlled by perceives as its politics,
through a nexus
corporations or national interests. entertainment, sports,
between a business
corporate interests. religion and other
The definition of class and the political
activities.
jingoism is extreme class.
and aggressive
patriotism that results
in aggressive foreign
policy.

QUESTION 9. NjYxMjUrTW9oYW1tYWQgWW91bmlzIEFobWFkK3NhaGlsLmxhdy5qbWlAZ21haWwuY29tKzk1
UVTVElPTiA4
Which among the following is/are Quantitative measure of Credit Control?

1. Moral Suasion

2. Bank rate policy

3. Rationing of credit

4. Open Market Operations

Choose appropriate code:

a) 2 only
b) 2 and 4
c) 2, 3 and 4
d) 1, 2 and 4
Correct Answer: B
Your Answer: B
Explanation

Solution (b)

Approach/Elimination:

If you are aware of various methods employed by the RBI to control credit, you can easily
answer this question.

IASbaba
Score:
Web: http://ilp.iasbaba.com/
69.00 /
Email: ilp@iasbaba.com
Page 8 200
AIPTS/ILP VETERANS -
Exam Title :
2020 T...
Email : sahil.law.jmi@gmail.com
Contact : 9540210113

There are two important methods adapted by RBI to Control Credit Creation - quantitative
controls and qualitative controls.

· Quantitative controls are designed to regulate the volume of credit created by the banking
system. Qualitative measures or selective methods are designed to regulate the flow of credit in
specific uses.

· Quantitative or traditional methods of credit control include banks rate policy, open market
operations and variable reserve ratio. Qualitative or selective methods of credit control include
regulation of margin requirement, credit rationing, moral suasion (credit monitoring
arrangement), regulation of consumer credit and direct action.

Analysis:

Options 1 and 3 Options 2 and 4

Incorrect Correct

Qualitative measures include –

1. Changes in margin requirements Quantitative controls include –

2. Rationing of credit 1. Bank Rate Policy

3. Regulation of Consumer Credit 2. Open Market Operations

4. Moral suasion and credit monitoring 3. Variable Reserve Ratios


arrangement

QUESTION 10. NjYxMjUrTW9oYW1tYWQgWW91bmlzIEFobWFkK3NhaGlsLmxhdy5qbWlAZ21haWwuY29tKzk


UVTVElPTiA5
Which of the following correctly defines the term “opportunity cost”?

a) It is the cost or amount of money spent on searching an opportunity.


b) It represents the benefits an individual, investor or business misses out on when choosing
one alternative over another.
c) It is the amount of money that today’s assets will cost in future.
d) They are fixed, one-time expenses incurred on the purchase of land, buildings, construction,
and equipment used in the production of goods or in the rendering of services.
Correct Answer: B
Your Answer: D
Explanation

Solution (b)

Explanation:

IASbaba
Score:
Web: http://ilp.iasbaba.com/
69.00 /
Email: ilp@iasbaba.com
Page 9 200
AIPTS/ILP VETERANS -
Exam Title :
2020 T...
Email : sahil.law.jmi@gmail.com
Contact : 9540210113

Opportunity costs represent the benefits an individual, investor or business misses out on when
choosing one alternative over another.

A benefit, profit, or value of something that must be given up to acquire or achieve something
else.

Do you know?

· Opportunity costs are fundamental costs in economics, and are used in computing cost benefit
analysis of a project.

· Such costs, however, are not recorded in the account books but are recognized in decision
making by computing the cash outlays and their resulting profit or loss.

QUESTION 11. NjYxMjUrTW9oYW1tYWQgWW91bmlzIEFobWFkK3NhaGlsLmxhdy5qbWlAZ21haWwuY29tKzk


ElPTiAxMA==
The Fiscal Responsibility and Budget Management (FRBM) Act aims to

1. Eliminate revenue deficit and reduce fiscal deficit

2. Introduce a more equitable and manageable distribution of the country's debts over the years

Which of the statements given above is/ are correct?

a) 1 only
b) 2 only
c) Both 1 and 2
d) Neither 1 nor 2
Correct Answer: C
Your Answer: A
Explanation

Solution (c)

Explanation:

FRBM Act had the objective of ensuring prudence in fiscal management by –

· eliminating revenue deficit,

· reducing fiscal deficit,

· establishing improved debt management and

· improving transparency in a medium-term framework

- with quantitative targets to be adhered by the state with regard to deficit measures and debt
management.

IASbaba
Score:
Web: http://ilp.iasbaba.com/
69.00 /
Email: ilp@iasbaba.com
Page 10 200
AIPTS/ILP VETERANS -
Exam Title :
2020 T...
Email : sahil.law.jmi@gmail.com
Contact : 9540210113

The act was also expected to give necessary flexibility to Reserve Bank of India (RBI) for
managing inflation in India.

Statement analysis:

Statement 1 Statement 2

Correct Correct

FRBM Act introduces a more equitable and


FRBM Act eliminates revenue deficit and
manageable distribution of the country's
reduces fiscal deficit
debts over the years

QUESTION 12. NjYxMjUrTW9oYW1tYWQgWW91bmlzIEFobWFkK3NhaGlsLmxhdy5qbWlAZ21haWwuY29tKzk


lPTiAxMQ==
Consider the following statements:

1. Government facilitates the private sector’s choice of produced goods and services.

2. Price mechanism is usually not allowed due to welfare reasons.

Which of the statements above is/correct about ‘Market Economy’?

a) 1 only
b) 2 only
c) Both 1 and 2
d) Neither 1 nor 2
Correct Answer: D
Your Answer: B
Explanation

Solution (d)

Statement analysis:

Statement 1 Statement 2

Incorrect Incorrect

In a market economy, it is the private sector, Market forces (i.e. demand and supply)
which decides what to produce. decides the prices of goods and services.

IASbaba
Score:
Web: http://ilp.iasbaba.com/
69.00 /
Email: ilp@iasbaba.com
Page 11 200
AIPTS/ILP VETERANS -
Exam Title :
2020 T...
Email : sahil.law.jmi@gmail.com
Contact : 9540210113

Types of Economic Systems

· In a market economy, also called capitalism, only those consumer goods will be produced that
are in demand, i.e., goods that can be sold profitably either in the domestic or in the foreign
markets.

· In a capitalist society the goods produced are distributed among people not on the basis of
what people need but on the basis of what people can afford and are willing to purchase.

· In a socialist society the government decides what goods are to be produced in accordance
with the needs of society. It is assumed that the government knows what is good for the people
of the country and so the desires of individual consumers are not given much importance. The
government decides how goods are to be produced and how they should be distributed. In
principle, distribution under socialism is supposed to be based on what people need and not on
what they can afford to purchase. Strictly, a socialist society has no private property since
everything is owned by the state.

· Most economies are mixed economies, i.e., the government and the market together answer
the three questions of what to produce, how to produce and how to distribute what is produced.
In a mixed economy, the market will provide whatever goods and services it can produce well,
and the government will provide essential goods and services which the market fails to do.

QUESTION 13. NjYxMjUrTW9oYW1tYWQgWW91bmlzIEFobWFkK3NhaGlsLmxhdy5qbWlAZ21haWwuY29tKzk


ElPTiAxMg==
Consider the following statements and identify the appropriate definition of Ways and Means
Advances (WMA)

a) It is the mechanism through which RBI provides temporary loan facilities to the centre and
state governments.
b) It is the mechanism through which National Banks borrow temporary loan facilities from
the RBI.
c) It is the mechanism conducted by the RBI by way of sale or purchase of government
securities (g-secs) to adjust money supply conditions.
d) It is the mechanism through which RBI requests commercial banks to take certain
measures as per the trend of the economy.
Correct Answer: A
Your Answer: B
Explanation

Solution (a)

Explanation:

Ways and Means Advances

The Reserve Bank of India gives temporary loan facilities to the centre and state governments
as a banker to government. This temporary loan facility is called Ways and Means Advances
(WMA).

IASbaba
Score:
Web: http://ilp.iasbaba.com/
69.00 /
Email: ilp@iasbaba.com
Page 12 200
AIPTS/ILP VETERANS -
Exam Title :
2020 T...
Email : sahil.law.jmi@gmail.com
Contact : 9540210113

The WMA scheme for the Central Government was introduced on April 1, 1997, after putting an
end to the four-decade old system of adhoc (temporary) Treasury Bills to finance the Central
Government deficit. The WMA scheme was designed to meet temporary mismatches in the
receipts and payments of the government. This facility can be availed by the government if it
needs immediate cash from the RBI. The limits for WMA are mutually decided by the RBI and
the Government of India.

QUESTION 14. NjYxMjUrTW9oYW1tYWQgWW91bmlzIEFobWFkK3NhaGlsLmxhdy5qbWlAZ21haWwuY29tKzk


lPTiAxMw==
The rate at which scheduled banks can borrow funds overnight (in an emergency situation
when inter-bank liquidity dries up completely) from the RBI against government securities is
called –

a) Capital Adequacy Ratio


b) Cash flow adequacy facility
c) Marginal standing facility
d) Liquidity adjustment facility
Correct Answer: C
Your Answer: C
Explanation

Solution (c)

Approach:

Marginal Standing Facility is the rate at which scheduled banks could borrow funds overnight
from Reserve Bank of India (RBI) against government securities.

Liquidity Adjustment Facility Marginal Standing Facility

Minimum bidding amount is 5 cr. Minimum bidding amount is 1 cr.

All clients of RBI are eligible to bid. Only Scheduled Commercial Banks can bid.

Banks cannot sell government security to Banks can sell the government security from
RBI that is part of bank's SLR quota. its SLR quota to RBI.

MSF rate is always higher than Repo Rate


LAF lending rate is always repo rate.
as banks need the funds instantly.

Do you know?

IASbaba
Score:
Web: http://ilp.iasbaba.com/
69.00 /
Email: ilp@iasbaba.com
Page 13 200
AIPTS/ILP VETERANS -
Exam Title :
2020 T...
Email : sahil.law.jmi@gmail.com
Contact : 9540210113

· The Marginal Standing Facility rate currently stands at 6.00% following a decline by 25 basis
points on 6 June 2019.

QUESTION 15. NjYxMjUrTW9oYW1tYWQgWW91bmlzIEFobWFkK3NhaGlsLmxhdy5qbWlAZ21haWwuY29tKzk


ElPTiAxNA==
Consider the following statements regarding Cash Reserve Ratio (CRR)

1. It is the amount that all Scheduled Commercial Banks (SCB) excluding Regional Rural Banks
(RRB) are required to maintain without any floor or ceiling rate with RBI with reference to their
total net Demand and Time Liabilities (DTL).

2. When the RBI announces lower cash reserve ratio, it means that the commercial banks have
more money to lend.

Which of the statements given above is/are correct?

a) 1 only
b) 2 only
c) Both 1 and 2
d) Neither 1 nor 2
Correct Answer: C
Your Answer: C
Explanation

Solution (c)

Explanation:

Cash Reserve Ratio (CRR) is the amount of funds that all Scheduled Commercial Banks (SCB)
excluding Regional Rural Banks (RRB) are required to maintain without any floor or ceiling rate
with RBI with reference to their total net Demand and Time Liabilities (DTL) to ensure the
liquidity and solvency of Banks.

In terms of Section 42(1) of the RBI Act 1934, Scheduled Commercial Banks are required to
maintain with RBI an average cash balance, the amount of which shall not be less than 3% and
higher rate not exceeding 20% of the total of the Net Demand and Time Liabilities (NDTL) in
India.

Lower CRR means bank can give more money as loan = lower interest rates = cheap loan =
more people take loan to start business or building house or buying car = boost in economy.
However, can also lead to inflation, if people have more cash in their hands than the items
available for purchase in the market.

Statement Analysis:

Statement 1 Statement 2

IASbaba
Score:
Web: http://ilp.iasbaba.com/
69.00 /
Email: ilp@iasbaba.com
Page 14 200
AIPTS/ILP VETERANS -
Exam Title :
2020 T...
Email : sahil.law.jmi@gmail.com
Contact : 9540210113

Correct Correct

Cash Reserve Ratio (CRR) is the amount


that all Scheduled Commercial Banks (SCB)
When the RBI announces lower cash reserve
excluding Regional Rural Banks (RRB) are
ratio, it means that the commercial banks
required to maintain without any floor or
have more money to lend.
ceiling rate with RBI with reference to their
total net Demand and Time Liabilities (DTL).

Do you know?

· Demand Liabilities are liabilities which are payable on demand and Time Liabilities are those
which are payable otherwise than on demand.

QUESTION 16. NjYxMjUrTW9oYW1tYWQgWW91bmlzIEFobWFkK3NhaGlsLmxhdy5qbWlAZ21haWwuY29tKzk


ElPTiAxNQ==
Which of the following constitute Capital Account?

1. Foreign Loans

2. Foreign Direct Investment

3. Private Remittances

4. Portfolio Investment

Select the correct answer using the codes given below:

a) 1, 2 and 3
b) 1, 2 and 4
c) 2, 3 and 4
d) 1, 3 and 4
Correct Answer: B
Your Answer: Unanswered
Explanation

Solution (b)

Explanation:

Capital Account comprises of Foreign loans, Foreign Direct Investment and Portfolio
Investment. It does not include remittances.

Capital Account is the net result of public and private international investments flowing in and
out of a country. Portfolio investment is the buying of shares and bonds. FDI is the investments
by foreigners in a country or the citizens investing in foreign countries.

IASbaba
Score:
Web: http://ilp.iasbaba.com/
69.00 /
Email: ilp@iasbaba.com
Page 15 200
AIPTS/ILP VETERANS -
Exam Title :
2020 T...
Email : sahil.law.jmi@gmail.com
Contact : 9540210113

The three main components of Balance of Payments (BoP):

1. Current Account

2. Capital Account

3. Official Reserve Transactions

The current account includes all the transactions related to export and import of goods and
services, investment income, and unilateral transfers (remittances, gifts, grants etc.). The
capital account includes all international asset transactions (FDI, FPI etc.). The official reserve
transactions are conducted by central banks like RBI whenever there is BoP deficit or BoP
surplus. These transactions are conducted in the form of international reserve assets, such as
gold and major international currencies.

QUESTION 17. NjYxMjUrTW9oYW1tYWQgWW91bmlzIEFobWFkK3NhaGlsLmxhdy5qbWlAZ21haWwuY29tKzk


ElPTiAxNg==
Which among the following are part of different categories under priority sector?

1. Education

2. Micro, Small and Medium Enterprises

3. Social Infrastructure

4. Housing

5. Renewable Energy

Choose appropriate code:

a) 2, 3 and 4
b) 2, 3 and 5
c) 1, 2 and 3
d) 1, 2, 3, 4 and 5
Correct Answer: D
Your Answer: D
Explanation

Solution (d)

Approach:

PSL is norms set by RBI for the commercial banks in which the banks are suppose to lend 40%
of their total loans to some particular sectors, in a financial year. The below sectors are
mandated by RBI as the priority sectors.

IASbaba
Score:
Web: http://ilp.iasbaba.com/
69.00 /
Email: ilp@iasbaba.com
Page 16 200
AIPTS/ILP VETERANS -
Exam Title :
2020 T...
Email : sahil.law.jmi@gmail.com
Contact : 9540210113

Priority Sector includes the following categories:

(i) Agriculture

(ii) Micro, Small and Medium Enterprises

(iii) Export Credit

(iv) Education

(v) Housing

(vi) Social Infrastructure

(vii) Renewable Energy

(viii) Others

QUESTION 18. NjYxMjUrTW9oYW1tYWQgWW91bmlzIEFobWFkK3NhaGlsLmxhdy5qbWlAZ21haWwuY29tKzk


ElPTiAxNw==
Which of the below statements best describes the term ‘Goldilocks economy’?

a) An economy that is not so hot that it causes inflation, and not so cold that it causes a
recession, which allows a market-friendly monetary policy.
b) An economy that is too hot with inflation and driven by market-friendly monetary policy.
c) An economy that is struck between hot and cold, in other words doesn’t sustain moderate
economic growth and that has high inflation.
d) An economy that’s neither cold enough to support stimulus measures, nor hot enough to
sustain growth.
Correct Answer: A
Your Answer: Unanswered
Explanation

Solution (a)

Explanation:

A Goldilocks economy is an economy that is not too hot or cold, in other words sustains
moderate economic growth, and that has low inflation, which allows a market-friendly monetary
policy.

A Goldilocks Economy describes an economy that is not so hot that it causes inflation, and not
so cold that it causes a recession. The term describes an economy that is operating in an
optimal state by providing full employment and economic stability.

An anti-goldilocks economy, however, is one that’s neither cold enough to support stimulus
measures, nor hot enough to sustain growth.

IASbaba
Score:
Web: http://ilp.iasbaba.com/
69.00 /
Email: ilp@iasbaba.com
Page 17 200
AIPTS/ILP VETERANS -
Exam Title :
2020 T...
Email : sahil.law.jmi@gmail.com
Contact : 9540210113

Is Goldilocks economy a reality?

· Such a state of economy may occur during the recovery phases. For example, the US economy
of the later part of 90s was considered a Goldilocks economy because it was "not too hot, not
too cold, but just right". Because we have business cycles, a Goldilocks economy should be
considered a temporary state.

QUESTION 19. NjYxMjUrTW9oYW1tYWQgWW91bmlzIEFobWFkK3NhaGlsLmxhdy5qbWlAZ21haWwuY29tKzk


ElPTiAxOA==
Which of the following measures would result in an increase in the money supply in the
economy?

1. Purchase of government securities from the public by the Central Bank

2. Deposit of currency in the commercial banks by the public 66125

3. Borrowing by the government from the Central Bank

Select the correct answer using the codes given below:

a) 1 only
b) 2 and 2 only
c) 1 and 3 only
d) 1, 2 and 3
Correct Answer: C
Your Answer: C
Explanation

Solution (c)

Statement Analysis:

Please note that any money that flows out of RBI leads to increase in the money supply.

Statement 1 Statement 2 Statement 3

Correct Incorrect Correct

IASbaba
Score:
Web: http://ilp.iasbaba.com/
69.00 /
Email: ilp@iasbaba.com
Page 18 200
AIPTS/ILP VETERANS -
Exam Title :
2020 T...
Email : sahil.law.jmi@gmail.com
Contact : 9540210113

When the Government


When currency is deposited borrows from RBI, the money
When the RBI purchases by public in commercial again flows out of RBI.
government securities from banks, its mere transfer of
money from public to To fulfil the demand of the
Public, the money flows out
commercial banks. Government, RBI will print
of RBI, this will increase the
new money and this leads to
money supply.
The net effect on the money increase in the money supply
supply is nil. thus statement 3 is also
correct.

QUESTION 20. NjYxMjUrTW9oYW1tYWQgWW91bmlzIEFobWFkK3NhaGlsLmxhdy5qbWlAZ21haWwuY29tKzk


ElPTiAxOQ==
Consider the following statements:

1. The sum total of all the foreign currencies an economy possesses at a particular time is its
foreign currency assets/reserves.

2. Foreign exchange reserves act as the first line of defense for India in case of economic
slowdown.

3. The foreign exchange reserves are managed by the Reserve Bank of India for the Indian
government.

Which of the above statements is/are correct?

a) 1 and 2
b) 1 and 3
c) 2 and 3
d) 1, 2 and 3
Correct Answer: D
Your Answer: C
Explanation

Solution (d)

Explanation:

FOREIGN CURRENCY ASSETS

· The sum total of all the foreign currencies an economy possesses at a particular time is its
foreign currency assets/reserves.

· The total capacity of an economy to manage liquid foreign exchange is its foreign exchange
(Forex) reserve.

· This contains basically three components—the foreign currency assets, the total gold reserves
and the total special drawing rights (SDRs) of an economy in the IMF.

IASbaba
Score:
Web: http://ilp.iasbaba.com/
69.00 /
Email: ilp@iasbaba.com
Page 19 200
AIPTS/ILP VETERANS -
Exam Title :
2020 T...
Email : sahil.law.jmi@gmail.com
Contact : 9540210113

· The reserves are managed by the Reserve Bank of India for the Indian government and the
main component is foreign currency assets.

· Foreign exchange reserves act as the first line of defense for India in case of economic
slowdown. Foreign exchange reserves facilitate external trade and payment and promote
orderly development and maintenance of foreign exchange market in India.

QUESTION 21. NjYxMjUrTW9oYW1tYWQgWW91bmlzIEFobWFkK3NhaGlsLmxhdy5qbWlAZ21haWwuY29tKzk


ElPTiAyMA==
National income include which among the following items –

1. Construction of a new house

2. Winning of a lottery prize

3. Earnings of shareholders from the sale of shares

4. Rent-free house given to an employee by an employer

Choose appropriate code:

a) 2 and 4
b) 1, 3 and 4
c) 1 and 4
d) 1, 2, 3 and 4
Correct Answer: C
Your Answer: C
Explanation

Solution (c)

Statement analysis:

Rent-free house
Earnings of
Construction of a Winning of a lottery given to an
shareholders from
new house prize employee by an
the sale of shares
employer

Correct Incorrect Incorrect Correct

IASbaba
Score:
Web: http://ilp.iasbaba.com/
69.00 /
Email: ilp@iasbaba.com
Page 20 200
AIPTS/ILP VETERANS -
Exam Title :
2020 T...
Email : sahil.law.jmi@gmail.com
Contact : 9540210113

Yes, it will be included No, it will not be No, it will not be


Yes, it is included in
in the national income included in the included in the
the national income
as it is a part of national income as it national income as it
by Income Method
capital formation and does not add to the is a financial claim
since it is a part of
leads to production of flow of goods and and does not
‘wages in kind’ paid
goods and services in services in the contribute to any
to employees.
the economy. economy. productive activity.

QUESTION 22. NjYxMjUrTW9oYW1tYWQgWW91bmlzIEFobWFkK3NhaGlsLmxhdy5qbWlAZ21haWwuY29tKzk


ElPTiAyMQ==
Consider the following statements:

1. Deposit amount on bank balance sheet (account book) is put under liability category.

2. Lent money is put under asset category.

Select the correct option/s:

a) 1 only
b) 2 only
c) Both
d) None
Correct Answer: C
Your Answer: C
Explanation

Solution (c)

Statement analysis:

Statement 1 Statement 2

Correct Correct

Deposit amount on bank balance sheet is Lent money is put under asset category as it
put under liability category as banks have to generates income for the bank in the form
pay that amount back with interest. of interest.

QUESTION 23. NjYxMjUrTW9oYW1tYWQgWW91bmlzIEFobWFkK3NhaGlsLmxhdy5qbWlAZ21haWwuY29tKzk


ElPTiAyMg==
What is the difference between quaternary and quinary activities?

IASbaba
Score:
Web: http://ilp.iasbaba.com/
69.00 /
Email: ilp@iasbaba.com
Page 21 200
AIPTS/ILP VETERANS -
Exam Title :
2020 T...
Email : sahil.law.jmi@gmail.com
Contact : 9540210113

a) While, quaternary activities are related to IT enabled services, quinary activities are related
to financial services.
b) Quaternary activities pertain to information processing. Whereas, quinary activities pertain
to decision making.
c) Both a and b
d) None of the above
Correct Answer: B
Your Answer: Unanswered
Explanation

Solution (b)

Approach/Elimination

Tertiary activities are categorized into quaternary and quinary activities. The former involves
collection, production, processing, and dissemination of information. For example, software
developers, tax consultants etc.

Quinary activities focus on creation, re-arrangement, and interpretation of ideas. It pertains to


high level decision making. Therefore, it is also called ‘gold collar’ professions. For example,
business executives, research scientists etc.

Statement analysis:

Statement (a) Statement (b)

Incorrect Correct

Quaternary activities involves collection,


production, processing, and dissemination
of information. For example, software Quinary activities focus on creation, re-
developers, tax consultants etc. arrangement, and interpretation of ideas. It
pertains to high level decision making.
Financial services is not part of Quinary
activities.

QUESTION 24. NjYxMjUrTW9oYW1tYWQgWW91bmlzIEFobWFkK3NhaGlsLmxhdy5qbWlAZ21haWwuY29tKzk


ElPTiAyMw==
Which of the following are basic tenets of Planning in India?

1. Growth

2. Modernization

3. Self reliance

4. Social justice

IASbaba
Score:
Web: http://ilp.iasbaba.com/
69.00 /
Email: ilp@iasbaba.com
Page 22 200
AIPTS/ILP VETERANS -
Exam Title :
2020 T...
Email : sahil.law.jmi@gmail.com
Contact : 9540210113

Choose the right option:

a) 1, 2 and 3
b) 1 and 3
c) 1, 3 and 4
d) All of the above
Correct Answer: D
Your Answer: D
Explanation

Solution (d)

Approach:

Growth, Modernization, Self reliance and Social justice – All are basic tenets of Planning.

Adoption of new technology is called modernisation. However, modernisation does not refer
only to the use of new technology but also to changes in social outlook such as the recognition
that women should have the same rights as men.

Using its own resources is called self-reliance. A nation can promote economic growth and
modernisation by using its own resources or by using resources imported from other nations.
The first seven five year plans gave importance to self-reliance which means avoiding imports of
those goods which could be produced in India itself. This policy was considered a necessity in
order to reduce our dependence on foreign countries, especially for food.

QUESTION 25. NjYxMjUrTW9oYW1tYWQgWW91bmlzIEFobWFkK3NhaGlsLmxhdy5qbWlAZ21haWwuY29tKzk


ElPTiAyNA==
Which among the following statements is/are true with reference to depreciation of a currency?

1. It is a fall in the value of domestic currency with respect to other currencies

2. It is done by deliberate government intervention

Choose appropriate answer:

a) 1 only
b) 2 only
c) Both 1 and 2
d) Neither 1 nor 2
Correct Answer: A
Your Answer: A
Explanation

Solution (a)

Approach:

IASbaba
Score:
Web: http://ilp.iasbaba.com/
69.00 /
Email: ilp@iasbaba.com
Page 23 200
AIPTS/ILP VETERANS -
Exam Title :
2020 T...
Email : sahil.law.jmi@gmail.com
Contact : 9540210113

Currency depreciation is the loss of value of a country's currency with respect to one or more
foreign reference currencies, typically in a floating exchange rate system in which no official
currency value is maintained. In a floating exchange rate system, a currency's value goes up (or
down) if the demand for it goes up more (or less) than the supply does.

On the other hand, when the government lowers the exchange rate of its currency, it is called
Devaluation, it is intentional and is done to promote the exports of the country.

Statement analysis:

Statement (1) Statement (2)

Correct Incorrect

Currency depreciation is the loss of value of


It is not done by deliberate government
a country's currency with respect to one or
intervention.
more foreign reference currencies.

QUESTION 26. NjYxMjUrTW9oYW1tYWQgWW91bmlzIEFobWFkK3NhaGlsLmxhdy5qbWlAZ21haWwuY29tKzk


ElPTiAyNQ==
From the below, identify the correct statement about disposable income -

a) It is the money available with households after deduction of income tax.


b) It is the liquid money available with households.
c) It is the gross money earned by a household.
d) It is the money available to the household once savings are made.
Correct Answer: A
Your Answer: B
Explanation

Solution (a)

Explanation:

Disposable income, also known as disposable personal income (DPI), is the amount of money
that households have available for spending and saving after income taxes have been accounted
for. Disposable personal income is often monitored as one of the many key economic indicators
used to gauge the overall state of the economy.

Economists use DPI as a starting point to gauge households' rates of savings and spending.

Disposable income minus all payments for necessities, such as mortgage, health insurance, food
and transportation, equals discretionary income. This portion of disposable income can be spent

IASbaba
Score:
Web: http://ilp.iasbaba.com/
69.00 /
Email: ilp@iasbaba.com
Page 24 200
AIPTS/ILP VETERANS -
Exam Title :
2020 T...
Email : sahil.law.jmi@gmail.com
Contact : 9540210113

on what the income earner chooses or, alternatively, it can be saved. The personal savings rate
is the percentage of disposable income that goes into savings for retirement or use at a later
date. Marginal propensity to consume represents the percentage of each additional dollar of
disposable income that gets spent, while marginal propensity to save denotes the percentage
that gets saved.

QUESTION 27. NjYxMjUrTW9oYW1tYWQgWW91bmlzIEFobWFkK3NhaGlsLmxhdy5qbWlAZ21haWwuY29tKzk


ElPTiAyNg==
Graphical curve which advocates a relationship between inflation and unemployment in an
economy is called

a) Laffer's curve
b) Phillips curve
c) Friedman curve
d) Rahn curve
Correct Answer: B
Your Answer: A
Explanation

Solution (b)

Analysis:

Phillips’s curve

· Graphical curve which advocates a relationship between inflation and unemployment in an


economy.

· It means decreased unemployment, (i.e., increased levels of employment) in an economy will


correlate with higher rates of inflation. While there is a short run tradeoff between
unemployment and inflation, it has not been observed in the long run. In

· 1968, Milton Friedman asserted that the Phillips curve was only applicable in the short-run
and that in the long-run, inflationary policies will not decrease unemployment.

IASbaba
Score:
Web: http://ilp.iasbaba.com/
69.00 /
Email: ilp@iasbaba.com
Page 25 200
AIPTS/ILP VETERANS -
Exam Title :
2020 T...
Email : sahil.law.jmi@gmail.com
Contact : 9540210113

· Friedman then correctly predicted that in the 1973–75 recession, both inflation and
unemployment would increase. The long-run Phillips curve is now seen as a vertical line at the
natural rate of unemployment, where the rate of inflation has no effect on unemployment.

Laffer’s curve

· It illustrates a theoretical relationship between rates of taxation and the resulting levels of
government revenue. It illustrates the concept of taxable income elasticity—i.e., taxable income
changes in response to changes in the rate of taxation.

Friedman Curve

· There is no curve as such

Rahn Curve

· The Rahn curve is a graph used to illustrate an economic theory, proposed in 1996 by
American economist Richard W. Rahn, which indicates that there is a level of government
spending that maximises economic growth. The theory is used by classical liberals to argue for
a decrease in overall government spending and taxation.

· There are two interlinked aspects of the Perspectives on the appropriate amount of
Government Debt. One is whether government debt is a burden and two is regarding the issue
of financing the debt.

IASbaba
Score:
Web: http://ilp.iasbaba.com/
69.00 /
Email: ilp@iasbaba.com
Page 26 200
AIPTS/ILP VETERANS -
Exam Title :
2020 T...
Email : sahil.law.jmi@gmail.com
Contact : 9540210113

· It has often been argued that public ‘debt does not matter because we owe it to ourselves’.
This is because although there is a transfer of resources between generations, purchasing
power remains within the nation. However, any debt that is owed to foreigners involves a
burden since we have to send goods abroad corresponding to the interest payments.

· If the government invests in infrastructure, future generations may be better off, provided the
return on such investments is greater than the rate of interest. The actual debt could be paid off
by the growth in output. The debt should not then be considered burdensome. The growth in
debt will have to be judged by the growth of the economy as a whole. Hence Debt is
burdensome if it reduces future growth in output.

QUESTION 28. NjYxMjUrTW9oYW1tYWQgWW91bmlzIEFobWFkK3NhaGlsLmxhdy5qbWlAZ21haWwuY29tKzk


ElPTiAyNw==
Which of the statements given below is/are correct about cash reserve ratio (CRR)?

1. RBI keeps the reserve of the banks like cash reserve ratio (CRR) with it.

2. CRR hike will have negative impact over Equity Market, Bond Market, Real estate and
Automobile sector.

Choose appropriate code:

a) 1 only
b) 2 only
c) Both 1 and 2
d) Neither 1 nor 2
Correct Answer: A
Your Answer: B
Explanation

Solution (a)

Approach:

Cash Reserve Ratio (CRR) is the amount of funds that all Scheduled Commercial Banks (SCB)
excluding Regional Rural Banks (RRB) are required to maintain without any floor or ceiling rate
with RBI with reference to their total net Demand and Time Liabilities (DTL) to ensure the
liquidity and solvency of Banks.

Whenever the RBI hikes the CRR rate, a lot of excess liquidity is sucked out of the markets.
Banks have lesser cash available with them to deploy as loans.

Statement analysis:

Statement 1 Statement 2

IASbaba
Score:
Web: http://ilp.iasbaba.com/
69.00 /
Email: ilp@iasbaba.com
Page 27 200
AIPTS/ILP VETERANS -
Exam Title :
2020 T...
Email : sahil.law.jmi@gmail.com
Contact : 9540210113

Correct Incorrect

Cash Reserve Ratio (CRR) is the amount of CRR hike will have negative impact over
funds that all Scheduled Commercial Banks Equity Market, Real estate and Automobile
are required to maintain with RBI. sector (but not on Bond Market)

Impact of CRR hike on various segments and sectors –

· On Equity Market: CRR hike will bring less liquidity into the banking system. If banks have
less cash, they will lend less money or will not invest in equities.

· On Commodity Market: CRR hike will bring less liquidity in the market and people will
invest less in Commodity market.

· On Bond Market: CRR hike will help Bond market as long term yield will increase.

· On Banking: Increased interest rates lead to lower credit growth and further bring about the
overall profitability. Thus, banking stocks might face selling pressure.

· On Real estate: CRR hike will reduce liquidity in the markets. Consumers will be less inclined
to borrow home loans at higher interest rates resulting in a fall in demand. Also construction
companies will have to borrow at higher rates thus increasing their overheads.

· On Auto: The same applies to autos. The demand for automobiles will decrease due to
increased lending rates and the cost to company will rise sharply due to increased borrowing
rates. Hence, auto stocks can also see a downturn.

QUESTION 29. NjYxMjUrTW9oYW1tYWQgWW91bmlzIEFobWFkK3NhaGlsLmxhdy5qbWlAZ21haWwuY29tKzk


ElPTiAyOA==
Consider the following statements:

1. Revaluation : : government increasing the exchange rate of its currency against any foreign
currency

2. Appreciation : : if a free floating domestic currency increases its value against the value of a
foreign currency

Which of the above is/are correctly matched?

a) 1 only
b) 2 only
c) Both 1 and 2
d) Neither 1 nor 2
Correct Answer: C
Your Answer: C
Explanation

IASbaba
Score:
Web: http://ilp.iasbaba.com/
69.00 /
Email: ilp@iasbaba.com
Page 28 200
AIPTS/ILP VETERANS -
Exam Title :
2020 T...
Email : sahil.law.jmi@gmail.com
Contact : 9540210113

Solution (c)

Statement Analysis:

Statement 1 Statement 2

Correct Correct

APPRECIATION : : In foreign exchange


market, if a free floating domestic currency
increases its value against the value of a
REVALUATION : : A term used in foreign foreign currency, it is appreciation.
exchange market which means a
In domestic economy, if a fixed asset has
government increasing the exchange rate of
seen increase in its value it is also known as
its currency against any foreign currency.
appreciation. Appreciation rates for
different assets are not fixed by any
government as they depend upon many
factors which are unseen.

QUESTION 30. NjYxMjUrTW9oYW1tYWQgWW91bmlzIEFobWFkK3NhaGlsLmxhdy5qbWlAZ21haWwuY29tKzk


ElPTiAyOQ==
Which of the statements given below is/are correct about Central Statistics Office (CSO)?

1. It releases Consumer Price Indices (CPI) for all-India and states/union territories separately
for rural, urban, industrial workers and combined (rural plus urban).

2. It also disseminates Energy Statistics, Social and Environment Statistics and prepares the
National Industrial Classification.

Choose the correct answer:

a) 1 only
b) 2 only
c) Both 1 and 2
d) Neither 1 nor 2
Correct Answer: B
Your Answer: A
Explanation

Solution (b)

Statement analysis:

IASbaba
Score:
Web: http://ilp.iasbaba.com/
69.00 /
Email: ilp@iasbaba.com
Page 29 200
AIPTS/ILP VETERANS -
Exam Title :
2020 T...
Email : sahil.law.jmi@gmail.com
Contact : 9540210113

Statement 1 Statement 2

Incorrect Correct

Central Statistics Office (CSO) coordinates


the statistical activities in the country and
evolves statistical standards.

Consumer Price Indices (CPI) for Industrial Its activities include compilation of National
Workers are released by Labour Bureau, Accounts, Index of Industrial Production,
Government of India. (Not by CSO) Consumer Price Indices for Urban/Rural/
Combined, Human Development Statistics,
Hence, statement (1) is wrong. including Gender Statistics in the states and
union territories and disseminates Energy
Statistics, Social and Environment Statistics
and prepares the National Industrial
Classification.

QUESTION 31. NjYxMjUrTW9oYW1tYWQgWW91bmlzIEFobWFkK3NhaGlsLmxhdy5qbWlAZ21haWwuY29tKzk


ElPTiAzMA==
What is the term used in the foreign exchange market which denotes the currency that is easily
available in any economy in its forex market?

a) Hard currency
b) Hot currency
c) Soft currency
d) Cheap currency
Correct Answer: C
Your Answer: B
Explanation

Solution (c)

Analysis:

Hard Currency Hot currency Soft Currency Cheap currency

Incorrect Incorrect Correct Incorrect

IASbaba
Score:
Web: http://ilp.iasbaba.com/
69.00 /
Email: ilp@iasbaba.com
Page 30 200
AIPTS/ILP VETERANS -
Exam Title :
2020 T...
Email : sahil.law.jmi@gmail.com
Contact : 9540210113

It is the international
currency in which the
highest faith is shown
and is needed by
every economy.
A term first used by
The strongest the economist J. M.
currency of the world A term of the forex Keynes (1930s). If a
is one which has a market and is a A term used in the government starts re-
high level of liquidity. temporary name for foreign exchange purchasing its bonds
any hard currency. market which denotes before their
Basically, the Due to certain the currency that is maturities (at full-
economy with the reasons, if a hard easily available in any maturity prices) the
highest as well as currency is exiting an economy in its forex money which flows
highly diversified economy at a fast market. For example, into the economy is
exports that are pace for the time, the rupee is a soft known as the cheap
compulsive imports hard currency is currency in the Indian currency, also called
for other countries (as known to be hot. As in forex market. It is cheap money.
of high-level the case of the SE basically the opposite
technology, defence Asian crisis, the US term for the hard In banking industry, it
products, lifesaving dollar had become currency. means a period of
medicines and hot. comparatively lower/
petroleum products) softer interest rates
will also create high regime.
demand for its
currency in the world
and become the hard
currency. It is always
scarce.

QUESTION 32. NjYxMjUrTW9oYW1tYWQgWW91bmlzIEFobWFkK3NhaGlsLmxhdy5qbWlAZ21haWwuY29tKzk


ElPTiAzMQ==
Consider the following statements

1. Measuring the aggregate values of spending that the firms receive for the final goods and
services which they produce is called Expenditure method.

2. Method of measuring the aggregate values of final goods and services produced by all the
firms is called Product method.

3. Method of measuring the sum total of all factor payments are called Income method

Choose the incorrect statement/s

a) 1 and 2 only
b) 2 and 3 only
c) 1 and 3 only
d) None of the above

IASbaba
Score:
Web: http://ilp.iasbaba.com/
69.00 /
Email: ilp@iasbaba.com
Page 31 200
AIPTS/ILP VETERANS -
Exam Title :
2020 T...
Email : sahil.law.jmi@gmail.com
Contact : 9540210113

Correct Answer: D
Your Answer: A
Explanation

Solution (d)

Explanation/Statement analysis:

All the statements are correct.

Statement 1 Statement 2 Statement 3

Correct Correct Correct

Measuring the aggregate


Method of measuring the
values of spending, that the Method of measuring the
aggregate values of final
firms receive for the final sum total of all factor
goods and services produced
goods and services which payments are called Income
by all the firms is called
they produce is called method.
Product method.
Expenditure method.

QUESTION 33. NjYxMjUrTW9oYW1tYWQgWW91bmlzIEFobWFkK3NhaGlsLmxhdy5qbWlAZ21haWwuY29tKzk


ElPTiAzMg==
Consider the following statements:

1. Savings bank account has both time and demand liabilities.

2. Current Bank account has time liabilities but no demand liabilities.

3. Fixed deposit has demand liabilities but no time liabilities.

Which of the above statements are correct?

a) 1 only
b) 2 and 3
c) 1 and 3
d) All of the above
Correct Answer: A
Your Answer: A
Explanation

Solution (a)

Statement Analysis:

IASbaba
Score:
Web: http://ilp.iasbaba.com/
69.00 /
Email: ilp@iasbaba.com
Page 32 200
AIPTS/ILP VETERANS -
Exam Title :
2020 T...
Email : sahil.law.jmi@gmail.com
Contact : 9540210113

Statement 1 Statement 2 Statement 3

Correct Incorrect Incorrect

A savings account is a
deposit account held at a
retail bank that pays interest
but cannot be used directly
as money in the narrow sense
of a medium of exchange (for
example, by writing a
cheque).
Current bank account is
These accounts let customers opened by businessmen who
set aside a portion of their have a higher number of
liquid assets while earning a regular transactions with the Fixed deposit is 100% time
monetary return. For the bank. It includes deposits, liability. You can not
bank, money in a savings withdrawals, and contra withdraw money from a fixed
account may not be callable transactions. deposit before a stipulated
immediately and, in some
time. If you have to withdraw
jurisdictions, does not incur a It is also known as Demand
money from it, bank will
reserve requirement. Cash in Deposit Account.
charge you a penalty.
the bank's vaults may thus be
used, for example, to fund For this the bank does not
interest-paying loans. pay an interest. Rather it
takes service charges.
The limit of money which you
can withdraw at any time is
the demand Liability of the
bank which it has to maintain
with itself. Rest of the
amount is a time liability for
which you will have to give a
prior notice to the bank.

QUESTION 34. NjYxMjUrTW9oYW1tYWQgWW91bmlzIEFobWFkK3NhaGlsLmxhdy5qbWlAZ21haWwuY29tKzk


ElPTiAzMw==
Which of the following has the largest share in the Gross Domestic Savings in India?

a) House Hold savings


b) Private Corporate sector
c) Public Sector
d) None of the above
Correct Answer: A
Your Answer: A
Explanation

IASbaba
Score:
Web: http://ilp.iasbaba.com/
69.00 /
Email: ilp@iasbaba.com
Page 33 200
AIPTS/ILP VETERANS -
Exam Title :
2020 T...
Email : sahil.law.jmi@gmail.com
Contact : 9540210113

Solution (a)

Approach:

Gross Domestic Savings and Gross Domestic Capital Formation

The data refers to Gross Domestic Savings and Gross Domestic Capital Formation. It gives
details regarding private corporate sector, public sector and households sector, individually.
Gross Domestic Savings indicates the financial state and growth of the country, as household
saving is the main source of government borrowing to fund public services.

As per RBI’s report, House Hold savings are highest followed by Private Corporate sector and
Public Sector.

QUESTION 35. NjYxMjUrTW9oYW1tYWQgWW91bmlzIEFobWFkK3NhaGlsLmxhdy5qbWlAZ21haWwuY29tKzk


ElPTiAzNA==
Gross National Product is calculated

a) By measuring the sum total of gross value added by all the firms in the economy including
depreciation.
b) By measuring the sum total of gross value added by all the firms in the economy excluding
depreciation.
c) By measuring the sum total of gross value added by all the firms in the economy including
depreciation and net factor income from abroad.
d) By measuring the sum total of gross value added by all the firms in the economy excluding
depreciation and net factor income from abroad
Correct Answer: C
Your Answer: C
Explanation

Solution (c)

Explanation:

The macroeconomic variable which takes into account such additions and subtractions is known
as Gross National Product (GNP).

GNP measures the total monetary value of the total output produced by a country's residents.
Therefore, any output produced by foreign residents within the country's borders must be
excluded in calculations of GNP, while any output produced by the country's residents outside of
its borders must be counted.

GNP is related to another important economic measure called gross domestic product (GDP),
which takes into account all output produced within a country's borders regardless of who owns
the means of production. GNP starts with GDP, adds residents' investment income from
overseas investments, and subtracts foreign residents' investment income earned within a
country.

IASbaba
Score:
Web: http://ilp.iasbaba.com/
69.00 /
Email: ilp@iasbaba.com
Page 34 200
AIPTS/ILP VETERANS -
Exam Title :
2020 T...
Email : sahil.law.jmi@gmail.com
Contact : 9540210113

GNP = GDP + Factor income earned by the domestic factors of production employed in the rest
of the world – Factor income earned by the factors of production of the rest of the world
employed in the domestic economy.

Hence, GNP = GDP + Net factor income from abroad

(Net factor income from abroad = Factor income earned by the domestic factors of production
employed in the rest of the world – Factor income earned by the factors of production of the
rest of the world employed in the domestic economy).

QUESTION 36. NjYxMjUrTW9oYW1tYWQgWW91bmlzIEFobWFkK3NhaGlsLmxhdy5qbWlAZ21haWwuY29tKzk


ElPTiAzNQ==
Consider the following with regard to Liquidity Adjustment Facility (LAF)

1. It allows banks to borrow money through repurchase agreements.

2. LAF consists of two instruments - Statutory Liquidity Ratio and Cash Reserve Ratio.

Which of the statements given above is/are correct?

a) 1 only
b) 2 only
c) Both 1 and 2
d) Neither 1 nor 2
Correct Answer: A
Your Answer: B
Explanation

Solution (a)

Approach:

Liquidity adjustment facility (LAF) is a monetary policy tool which allows banks to borrow
money through repurchase agreements. LAF is used to aid banks in adjusting the day to day
mismatches in liquidity. LAF consists of repo and reverse repo operations.

Statutory Liquidity Ratio and Cash Reserve Ratio are credit control instruments which come
under Variable Reserve Ratio. These are the reserves that scheduled banks have to maintain
and are varied to control the credit creation.

Statement analysis:

Statement 1 Statement 2

Correct Incorrect

IASbaba
Score:
Web: http://ilp.iasbaba.com/
69.00 /
Email: ilp@iasbaba.com
Page 35 200
AIPTS/ILP VETERANS -
Exam Title :
2020 T...
Email : sahil.law.jmi@gmail.com
Contact : 9540210113

Liquidity Adjustment Facility (LAF) allows Liquidity Adjustment Facility (LAF) consists
banks to borrow money through repurchase of two instruments - repo and reverse repo
agreements. operations.

QUESTION 37. NjYxMjUrTW9oYW1tYWQgWW91bmlzIEFobWFkK3NhaGlsLmxhdy5qbWlAZ21haWwuY29tKzk


ElPTiAzNg==
Which of the following is not a method to calculate the National income of a country?

a) Interest Method
b) Income method
c) Expenditure method
d) Value – Added Method
Correct Answer: A
Your Answer: A
Explanation

Solution (a)

Approach

For calculating national income- an economy is looked upon from three different angles:

1. Production units in an economy are classified into primary, secondary, and tertiary sectors.
On the basis of this classification, value-added method is used to measure national income.

2. Economy is also viewed as a combination of individuals and households owing different kinds
of factors of production. On the basis of this combination, income method is used for estimating
national income.

3. Economy is viewed as a collection of units used for consumption, saving, and investment. On
the basis of this collection, final expenditure method is used for calculating national income.

Final Expenditure Value-added


Interest Method Income Method
Method Method

IASbaba
Score:
Web: http://ilp.iasbaba.com/
69.00 /
Email: ilp@iasbaba.com
Page 36 200
AIPTS/ILP VETERANS -
Exam Title :
2020 T...
Email : sahil.law.jmi@gmail.com
Contact : 9540210113

Final expenditure
method, also known
as final product
method, is used to Value added method,
Income method, also
measure final also called net output
known as factor
expenditures incurred method, is used to
income method, is
by production units measure the
used to calculate all
for producing final contribution of an
income accrued to the
goods and services economy’s production
basic factors of
within an economic units to the GDPmp.
production used in
territory during a In other words, value-
producing national
given time period. added method
product. Traditionally,
measures value added
Interest Method is not there are four factors
These expenditures by each industry in an
a method to calculate of production, namely
are incurred on economy. For
the National income land, labor, capital,
consumption and calculating national
of a country. and organization.
investment. This income through
Accordingly there are
method is the value-added method,
four factor payments,
opposite of the value- it is necessary to first
namely rent,
added method. This is calculate gross value
compensation of
because value-added added at market price
employees, interest,
method estimates (GVAmp), net value
and profit. There is
national income from added at market price
another category of
the sales side, (NVAmp), and net
factor payment called
whereas the value added at factor
mixed income.
expenditure method cost (NVAfc).
calculates national
income from the
purchase side.

QUESTION 38. NjYxMjUrTW9oYW1tYWQgWW91bmlzIEFobWFkK3NhaGlsLmxhdy5qbWlAZ21haWwuY29tKzk


ElPTiAzNw==
Agriculture income in India is exempted from taxation. Which of the following incomes do not
come under Agricultural income in India?

a) Any rent received from land which is used for agriculture purpose.
b) The income obtained from sale of replanted trees and growing flowers and creepers.
c) Income obtained from the sale of seeds.
d) Income generated through poultry or bee keeping.
Correct Answer: D
Your Answer: B
Explanation

Solution (d)

Explanation:

IASbaba
Score:
Web: http://ilp.iasbaba.com/
69.00 /
Email: ilp@iasbaba.com
Page 37 200
AIPTS/ILP VETERANS -
Exam Title :
2020 T...
Email : sahil.law.jmi@gmail.com
Contact : 9540210113

The income earned from agricultural land is exempt from taxes under Section 10(1) of the
Income Tax Act 1961. Agricultural income is computed in the same manner as Business income.

What Is Defined As Agricultural Income In India?

· Any rent received from land which is used for agricultural purposes.

· Agricultural operations which mainly include processing of farm produce which is then sold in
the market as agricultural produce.

· Income obtained from a farmhouse which is in proximity to the land you own. And which may
be used as a storehouse or a dwelling unit.

· It is very necessary that land exists and it is used for agricultural operations. Agricultural
operations means the seeds should be planted and effort made to sprout seeds from the land
and cultivate the land.

· Let us consider that you receive rental income from your land. It is very necessary that
agricultural activities are carried out on this land. Only then can you state that rental income
obtained from your land is agricultural income.

· The income obtained from sale of replanted trees and growing flowers and creepers

· Income obtained from the sale of seeds.

· The profits received from your partner who is engaged in agricultural produce or activities.

· Income earned from nursery operations such as selling of plants and flowers.

What Is Not Considered As Agricultural Income In India?

· Let us consider that you have poultry or manage bee hives. This is not considered as
agricultural income in India.

· If you have income from dairy and spontaneously grown trees.

· Income earned by producing salt by flooding the land with sea water.

· Royalty income from mines and purchase of standing crop.

· Income from butter and cottage cheese making.

· TV Serial shooting in farmhouses is not considered as agricultural Income and is taxed.

· Land located near a busy thoroughfare surrounded by industrial areas or estates and
commercial buildings and vegetables are grown on this land then this land does not constitute
as agricultural land and income is taxed.

QUESTION 39. NjYxMjUrTW9oYW1tYWQgWW91bmlzIEFobWFkK3NhaGlsLmxhdy5qbWlAZ21haWwuY29tKzk


ElPTiAzOA==
Total demand for money in an economy is composed of

IASbaba
Score:
Web: http://ilp.iasbaba.com/
69.00 /
Email: ilp@iasbaba.com
Page 38 200
AIPTS/ILP VETERANS -
Exam Title :
2020 T...
Email : sahil.law.jmi@gmail.com
Contact : 9540210113

a) Transaction demand which is directly proportional to real GDP and speculative demand
which is inversely related to the market rate of interest.
b) Transaction demand which is inversely proportional to real GDP and speculative demand
which is directly related to the market rate of interest Narmada
c) Transaction demand which is directly proportional to market rate of interest and
speculative demand which is inversely related to the real GDP.
d) Transaction demand which is inversely proportional to market rate of interest and
speculative demand which is directly related to the real GDP.
Correct Answer: A
Your Answer: Unanswered
Explanation

Solution (a)

Explanation:

Total demand for money in an economy is composed of transaction demand which is directly
proportional to real GDP and speculative demand which is inversely related to the market rate
of interest. The former is directly proportional to real GDP and price level, whereas the latter is
inversely related to the market rate of interest.

QUESTION 40. NjYxMjUrTW9oYW1tYWQgWW91bmlzIEFobWFkK3NhaGlsLmxhdy5qbWlAZ21haWwuY29tKzk


ElPTiAzOQ==
Bankruptcy and insolvency is a subject on which –

a) only the Parliament can make law


b) only State Legislatures can make law
c) both the Parliament and State Legislatures can make law
d) the Parliament can make law with the consent of State Legislatures through resolutions
Correct Answer: C
Your Answer: A
Explanation

Solution (c)

Explanation/Elimination:

Bankruptcy and insolvency is a subject on which both the Parliament and State Legislatures can
make law.

The Concurrent List or List-III (Seventh Schedule) is a list of 52 items given in the Seventh
Schedule to the Constitution of India.It includes the power to be considered by both the central
and state government.

The 52 items currently on the list include - Bankruptcy and insolvency.

IASbaba
Score:
Web: http://ilp.iasbaba.com/
69.00 /
Email: ilp@iasbaba.com
Page 39 200
AIPTS/ILP VETERANS -
Exam Title :
2020 T...
Email : sahil.law.jmi@gmail.com
Contact : 9540210113

QUESTION 41. NjYxMjUrTW9oYW1tYWQgWW91bmlzIEFobWFkK3NhaGlsLmxhdy5qbWlAZ21haWwuY29tKzk


lPTiA0MA==
Consider the following pairs:

(Type of NBFCs/Activities) : : (Regulator)

1. Venture Capital Fund : : SEBI

2. Pension funds : : Insurance Regulatory and Development Authority (IRDA)

3. Merchant Banking Companies : : RBI

4. Housing Finance Companies : : Ministry of Urban Affairs

Which of the above pairs is/are correctly matched?

a) 1, 2 and 3 only
b) 3 and 4 only
c) 1, 2, 3 and 4
d) 1 and 2 only
Correct Answer: D
Your Answer: B
Explanation

Solution (d)

Elimination:

Look at the third pair. It can be easily eliminated, as a Ministry cannot be a regulatory body for
Housing Finance Companies. Hence, you can straight away eliminate three options and arrive
at the answer.

Statement Analysis:

Statement 1 Statement 2 Statement 3 Statement 4

Correct Correct Incorrect Incorrect

Housing Finance
Merchant Banking
Venture Capital Funds Pension Funds are Companies are
Companies are
are regulated by SEBI regulated by IRDA regulated by National
regulated by SEBI
Housing Bank.

Do you know?

Not all the NBFCs are regulated by RBI. Only those NBFCs that satisfy the 'Principal
Business criteria' defined by the RBI will get themselves registered with the RBI and these
entities will be regulated and supervised by RBI.

IASbaba
Score:
Web: http://ilp.iasbaba.com/
69.00 /
Email: ilp@iasbaba.com
Page 40 200
AIPTS/ILP VETERANS -
Exam Title :
2020 T...
Email : sahil.law.jmi@gmail.com
Contact : 9540210113

Additional Information:

A company, which fulfills both the following criteria, will be registered as NBFC by the RBI.

· Financial activity as “principal business” is when a company's financial assets constitute more
than 50% of the total assets

· Income from financial assets constitutes more than 50% of the gross income.

QUESTION 42. NjYxMjUrTW9oYW1tYWQgWW91bmlzIEFobWFkK3NhaGlsLmxhdy5qbWlAZ21haWwuY29tKzk


lPTiA0MQ==
As per Reserve Bank of India's guidelines, a bank can have which of the following components
to fulfill its Statutory Liquidity Ratio (SLR) quota?

1. Government Securities

2. Gold

3. Vault cash

Which of the statements given above is/are correct?

a) 1 and 2 only
b) 1 and 3 only
c) 2 and 3 only
d) 1, 2 and 3
Correct Answer: D
Your Answer: A
Explanation

Solution (d)

All the three (Government Securities, Gold, Vault Cash) can be used to fulfill SLR quota.

Additional Information:

As per December 10, 2015 notification by the RBI, for Scheduled Commercial Banks, the SLR
should be in the form of:

· In cash, or

· In gold valued at a price not exceeding the current market price, or

· Unencumbered investment in any of the following instruments, namely:-

o Dated securities of the Government of India or

o Treasury Bills of the Government of India; or

o State Development Loans (SDLs) of the State Governments

IASbaba
Score:
Web: http://ilp.iasbaba.com/
69.00 /
Email: ilp@iasbaba.com
Page 41 200
AIPTS/ILP VETERANS -
Exam Title :
2020 T...
Email : sahil.law.jmi@gmail.com
Contact : 9540210113

· The deposit and unencumbered approved securities required, [ under sub-section (2) of
section 11 of the Banking Regulation Act, 1949] to be made with the Reserve Bank by a banking
company incorporated outside India;

· Any balance maintained by a scheduled bank with the Reserve Bank in excess of the
balance required to be maintained by it under section 42 of the Reserve Bank of India Act,1934.

QUESTION 43. NjYxMjUrTW9oYW1tYWQgWW91bmlzIEFobWFkK3NhaGlsLmxhdy5qbWlAZ21haWwuY29tKzk


ElPTiA0Mg==
Which of the following are included in the Liabilities of RBI Balance sheet?

1. Foreign exchange held by RBI

2. Vault cash held by commercial banks with RBI

3. Treasury deposits of Government of India with RBI

4. Government Securities – Loan given to the Government of India by RBI

Select the correct answer using the code given below:

a) 1, 2 and 3 only
b) 2, 3 and 4 only
c) 1, 2 and 4 only
d) 1, 2, 3 and 4
Correct Answer: A
Your Answer: A
Explanation

Solution (a)

Elimination:

Statement 4 says that the Government securities (in this case) are loans given by RBI which
indicates that it as an asset of RBI. By ruling out 4th statement, there will be only one
possibility (option A) which is the correct answer.

IASbaba
Score:
Web: http://ilp.iasbaba.com/
69.00 /
Email: ilp@iasbaba.com
Page 42 200
AIPTS/ILP VETERANS -
Exam Title :
2020 T...
Email : sahil.law.jmi@gmail.com
Contact : 9540210113

Image Source: NCERT Class-12 Introductory Macroeconomics

QUESTION 44. NjYxMjUrTW9oYW1tYWQgWW91bmlzIEFobWFkK3NhaGlsLmxhdy5qbWlAZ21haWwuY29tKzk


lPTiA0Mw==
Consider the following statements regarding Repo Rate

1. The (fixed) interest rate at which the Reserve Bank provides overnight liquidity to banks

2. No collateral is required by the borrowing bank

3. Only Scheduled commercial banks can avail this facility

4. Borrowing Banks can use SLR quota securities as collateral

Which of the statements given above is/are correct?

a) 1 and 2 only
b) 1 and 3 only
c) 1, 2 and 3 only
d) 1, 2, 3 and 4
Correct Answer: B
Your Answer: B
Explanation

Solution (b)

Elimination:

Collaterals are required if commercial banks borrow money from RBI under Repo Rate. With
this fact one can eliminate the statement 2, thus Option b is the correct answer.

Statement Analysis:

IASbaba
Score:
Web: http://ilp.iasbaba.com/
69.00 /
Email: ilp@iasbaba.com
Page 43 200
AIPTS/ILP VETERANS -
Exam Title :
2020 T...
Email : sahil.law.jmi@gmail.com
Contact : 9540210113

Statement 1 Statement 2 Statement 3 Statement 4

Correct Incorrect Correct Incorrect

Repo Rate is the


(fixed) interest rate at Borrowing Banks can
Only Scheduled
which the Reserve Collateral is NOT use SLR quota
commercial banks can
Bank provides required securities as
avail this facility
overnight liquidity to collateral
banks

Additional Information:

The (fixed) interest rate at which the Reserve Bank provides overnight liquidity to banks against
the collateral of government and other approved securities under the Liquidity Adjustment
Facility (LAF).

QUESTION 45. NjYxMjUrTW9oYW1tYWQgWW91bmlzIEFobWFkK3NhaGlsLmxhdy5qbWlAZ21haWwuY29tKzk


ElPTiA0NA==
The total liability of the monetary authority of a country (RBI) is called High Powered Money.
The High Powered Money consists of –

1. Currency notes and coins in circulation with the public

2. Deposits held by commercial banks with RBI

3. Vault cash of public banks

4. Deposits held by the Government of India with RBI

Which of the statements given above is/are correct?

a) 1 and 3 only
b) 2 and 4 only
c) 1, 2 and 3 only
d) 1, 2, 3 and 4
Correct Answer: D
Your Answer: B
Explanation

Solution (d)

Do you know?

IASbaba
Score:
Web: http://ilp.iasbaba.com/
69.00 /
Email: ilp@iasbaba.com
Page 44 200
AIPTS/ILP VETERANS -
Exam Title :
2020 T...
Email : sahil.law.jmi@gmail.com
Contact : 9540210113

· High Powered Money is also called Monetary Base or Reserve Money or Base Money.

· All the options given are part of High Powered Money.

Additional Information:

Reserve Money (M0) forms the basis for the creation of liquid money in the economy.

A low (or high) Bank Rate reduces (or increases) Reserve Deposit Ratio (rdr) and hence
increases (or decreases) the value of the money multiplier, which is (1 + cdr)/(cdr + rdr). Thus,
for any given amount of high powered money (H), total money supply goes up.

QUESTION 46. NjYxMjUrTW9oYW1tYWQgWW91bmlzIEFobWFkK3NhaGlsLmxhdy5qbWlAZ21haWwuY29tKzk


ElPTiA0NQ==
Which of the following may be the consequence of rupee depreciation against the US Dollar?

a) Exports become attractive whereas Imports become an unattractive costly affair


b) Imports become attractive whereas Exports become an unattractive affair
c) Both Imports and Exports become unattractive
d) Both Imports and Exports become attractive
Correct Answer: A
Your Answer: B
Explanation

Solution (a)

Statement Analysis:

Option A Option B

Correct Incorrect

Exports may become attractive as rupee


Imports may become attractive when rupee
depreciates.
appreciates.
An exporter would comparatively earn
An importer would comparatively pay less (in
more rupees If he exchanges the earned
terms of rupees) while importing.
dollars.

Additional Information:

· Most of the international trade happens in US dollars. Therefore, as rupee depreciates,


exports become more profitable, because the exporter earns more rupees for exchanging dollar.

IASbaba
Score:
Web: http://ilp.iasbaba.com/
69.00 /
Email: ilp@iasbaba.com
Page 45 200
AIPTS/ILP VETERANS -
Exam Title :
2020 T...
Email : sahil.law.jmi@gmail.com
Contact : 9540210113

· On the other hand, imports become expensive as the importer needs to pay more rupees for
the dollars billed.

· Industries linked to exports like Pharmaceuticals and IT benefit with depreciation.

· Similarly, the industries linked to imports (or having vital components of their product
imported) have to bear higher input cost, which is ultimately passed on to the end users.

QUESTION 47. NjYxMjUrTW9oYW1tYWQgWW91bmlzIEFobWFkK3NhaGlsLmxhdy5qbWlAZ21haWwuY29tKzk


ElPTiA0Ng==
The difference between Broad Money and Narrow Money is based on?

a) Broad Money is generally used by RBI to lend money to the general public whereas Narrow
Money used by Union Government for its deficit financing
b) Broad Money is generally used by RBI to lend money to the Union Government whereas
Narrow Money used by Union Government to lend money to the general public
c) The main difference is between their liquidity
d) There is no difference between them as both are one and the same
Correct Answer: C
Your Answer: C
Explanation

Solution (c)

Additional Information:

· M1 and M2 are known as Narrow Money whereas M3 and M4 are known as Broad Money.

· M1 = Currency (Notes and Coins) held by the Public + Net demand Deposits held by the
commercial banks

· M2 = M1 + Savings deposits with Post Office savings banks

· M3 = M1 + Net Time deposits of commercial banks

· M4 = M3 + total deposits with Post Office savings organizations (excluding National Savings
Certificates)

· Liquidity – M1 > M2 > M3 > M4

QUESTION 48. NjYxMjUrTW9oYW1tYWQgWW91bmlzIEFobWFkK3NhaGlsLmxhdy5qbWlAZ21haWwuY29tKzk


ElPTiA0Nw==
Double coincidence of wants is a characteristic feature of?

a) Market Economy
b) Barter system

IASbaba
Score:
Web: http://ilp.iasbaba.com/
69.00 /
Email: ilp@iasbaba.com
Page 46 200
AIPTS/ILP VETERANS -
Exam Title :
2020 T...
Email : sahil.law.jmi@gmail.com
Contact : 9540210113

c) Both Market Economy and Barter system


d) None of the above
Correct Answer: B
Your Answer: B
Explanation

Solution (b)

Double coincidence of wants means that both the parties (buyer and seller) have to agree to sell
and buy each commodity.

Additional Information:

The problem is caused by the improbability of the wants, needs, or events that cause or
motivate a transaction occurring at the same time and the same place.

QUESTION 49. NjYxMjUrTW9oYW1tYWQgWW91bmlzIEFobWFkK3NhaGlsLmxhdy5qbWlAZ21haWwuY29tKzk


ElPTiA0OA==
“It is not from the benevolence of the butcher, the brewer, of the baker, that we expect our
dinner, but from their regard to their own interest. We address ourselves, not to their humanity
but to their self-love, and never talk to them of our own necessities but of their advantage”. This
is a passage from the Book “An Enquiry into the Nature and Cause of the Wealth of Nations”
which denotes?

a) Market Economy
b) Command Economy
c) Mixed Economy
d) Closed Economy
Correct Answer: A
Your Answer: A
Explanation

Solution (a)

Additional Information:

Adam Smith is regarded as the founding father of modern economics (it was known as political
economy at that time).

His well-known work An Enquiry into the Nature and Cause of the Wealth of Nations (1776) is
regarded as the first major comprehensive book on the subject.

The passage from the book. ‘It is not from the benevolence of the butcher, the brewer, of the
baker, that we expect our dinner, but from their regard to their own interest. We address
ourselves, not to their humanity but to their self-love, and never talk to them of our own
necessities but of their advantage’ is often cited as an advocacy for free market economy.

IASbaba
Score:
Web: http://ilp.iasbaba.com/
69.00 /
Email: ilp@iasbaba.com
Page 47 200
AIPTS/ILP VETERANS -
Exam Title :
2020 T...
Email : sahil.law.jmi@gmail.com
Contact : 9540210113

QUESTION 50. NjYxMjUrTW9oYW1tYWQgWW91bmlzIEFobWFkK3NhaGlsLmxhdy5qbWlAZ21haWwuY29tKzk


ElPTiA0OQ==
Consider the following statements regarding Foreign Currency Non-Resident (Bank) Account
(FCNR (B) Account)

1. FCNR (B) accounts can be opened in the form of termed deposits

2. FCNR (B) accounts can be opened not only by Non-Resident Indians (NRI) but also by
resident Indians

Which of the statements given above is/are incorrect ?

a) 1 Only
b) 2 Only
c) Both 1 and 2
d) Neither 1 nor 2
Correct Answer: B
Your Answer: Unanswered
Explanation

Solution (b)

Statement Analysis:

Statement 1 Statement 2

Correct Incorrect

FCNR (B) accounts can be opened in the FCNR (B) accounts can only be opened by
form of termed deposits NRIs or Persons of Indian Origin (PIO)

Additional Information:

The Foreign Currency Non-Resident (FCNR(B)) scheme was introduced with effect from May
15, 1993 to replace the then prevailing FCNR(A) scheme introduced in 1975, where the foreign
exchange risk was borne by RBI and subsequently by the Govt. of India.

QUESTION 51. NjYxMjUrTW9oYW1tYWQgWW91bmlzIEFobWFkK3NhaGlsLmxhdy5qbWlAZ21haWwuY29tKzk


lPTiA1MA==
Which one of the following is correct about ‘Depreciation’?

a) It is total value of wear and tear that happened in the assets.


b) Ministry of Finance in India decides and announces the rates of depreciation.

IASbaba
Score:
Web: http://ilp.iasbaba.com/
69.00 /
Email: ilp@iasbaba.com
Page 48 200
AIPTS/ILP VETERANS -
Exam Title :
2020 T...
Email : sahil.law.jmi@gmail.com
Contact : 9540210113

c) In the external sector when the value of domestic currency increases against a said foreign
currency it is called depreciation.
d) All of the above.
Correct Answer: A
Your Answer: A
Explanation

Solution (a)

Elimination:

· Statement (a) is quite easy, if one is aware of common meaning of depreciation.

· Statement (b) is quite confusing or we might be not aware of which Ministry decides.

· Statement (c) – If we use some common sense, this statement can be eliminated easily. Rupee/
currency depreciation means fall in value and appreciation means increase in value. Therefore,
eliminating this statement, we are left with either option (a) or (b). And safer side, you can opt
for statement (a) as the answer.

Statement Analysis:

Statement 1 Statement 2 Statement 3

Correct Incorrect Incorrect

In the external sector, if the


The governments of the
value of the domestic
economies decide and
currency falls (not when it
announce the rates by which
‘Depreciation’ deals with increases) following market
assets depreciate (done in
total value of wear and tear mechanism in front of a
India by the Ministry of
that happened in the assets. foreign currency, it is the
Commerce and Industry, not
situation of ‘depreciation’ in
Every asset (except human Ministry of Finance) and a
the domestic currency.
beings) go for depreciation in list is published.
the process of their uses, Currency or Rupee
The list is used by the
which means they ‘wear and depreciation is when rupee
different sections of the
tear’. value decreases (not
economy to determine the
increases) and more rupees
real levels of depreciations in
can buy one unit of foreign
different assets.
currency.

QUESTION 52. NjYxMjUrTW9oYW1tYWQgWW91bmlzIEFobWFkK3NhaGlsLmxhdy5qbWlAZ21haWwuY29tKzk


lPTiA1MQ==
Which among the following statements are true with regard to use of the concept of Net
Domestic Product (NDP)?

IASbaba
Score:
Web: http://ilp.iasbaba.com/
69.00 /
Email: ilp@iasbaba.com
Page 49 200
AIPTS/ILP VETERANS -
Exam Title :
2020 T...
Email : sahil.law.jmi@gmail.com
Contact : 9540210113

1. To show the achievements of the economy in research and development to cut down levels of
depreciation.

2. It is used in comparative analysis to compare the economies of the world.

Choose the correct answer:

a) 1 only
b) 2 only
c) Both 1 and 2
d) Neither 1 nor 2
Correct Answer: A
Your Answer: C
Explanation

Solution (a)

Elimination/Approach:

· Since, Net Domestic Product (NDP) is the GDP calculated after adjusting the weight of the
value of ‘depreciation’, statement (1) seems correct.

· Statement (2) can be eliminated as we always read about GDP in newspapers or any studies
when they compare the economies of the world.

The different uses of the concept of NDP are as given below:

· For domestic use only – to understand the historical situation of the loss due to depreciation to
the economy. Also used to understand and analyse the sectoral situation of depreciation in
industry and trade in comparative periods.

· To show the achievements of the economy in the area of research and development which have
tried cutting the levels of depreciation in a historical time period.

Statement Analysis:

Statement 1 Statement 2

Correct Incorrect

The concept of NDP is used to show the NDP is not used in comparative economics,
achievements of the economy in the area of i.e., to compare the economies of the world.
research and development which have tried
This is due to different rates of depreciation
cutting the levels of depreciation in a
which is set by the different economies of
historical time period.
the world.

IASbaba
Score:
Web: http://ilp.iasbaba.com/
69.00 /
Email: ilp@iasbaba.com
Page 50 200
AIPTS/ILP VETERANS -
Exam Title :
2020 T...
Email : sahil.law.jmi@gmail.com
Contact : 9540210113

QUESTION 53. NjYxMjUrTW9oYW1tYWQgWW91bmlzIEFobWFkK3NhaGlsLmxhdy5qbWlAZ21haWwuY29tKzk


ElPTiA1Mg==
Consider the following statements about Gross Domestic Product (GDP):

1. GDP is the total market value of all final goods and services.

2. It takes into consideration the goods produced within the boundaries of a country.

3. It indicates the qualitative aspects of the goods and services.

Which of the above statements is/are incorrect ?

a) 1 and 2 only
b) 2 and 3 only
c) 3 only
d) All of the above
Correct Answer: C
Your Answer: C
Explanation

Solution (c)

Statement Analysis:

Statement 1 and 2 Statement 3

Correct Incorrect

Gross Domestic Product (GDP) is the value


It is a ‘quantitative’ concept and its volume/
of the all final goods and services produced
size indicates the ‘internal’ strength of the
within the boundary of a nation during one
economy.
year.
But it does not say anything about the
The gross domestic product (GDP) is one of
‘qualitative’ aspects of the produced goods
the primary indicators used to gauge the
and services by the economy.
health of a country's economy.

QUESTION 54. NjYxMjUrTW9oYW1tYWQgWW91bmlzIEFobWFkK3NhaGlsLmxhdy5qbWlAZ21haWwuY29tKzk


lPTiA1Mw==
Consider the following statements about Gross National Product (GNP).

1. GNP is the total market value of all final goods and services produced by the nationals within
the boundaries of the country.

2. GNP is the GDP of the country added with its income from abroad.

Choose the incorrect answer:

IASbaba
Score:
Web: http://ilp.iasbaba.com/
69.00 /
Email: ilp@iasbaba.com
Page 51 200
AIPTS/ILP VETERANS -
Exam Title :
2020 T...
Email : sahil.law.jmi@gmail.com
Contact : 9540210113

a) 1 only
b) 2 only
c) Both 1 and 2
d) Neither 1 nor 2
Correct Answer: A
Your Answer: A
Explanation

Solution (a)

Statement Analysis:

Statement 1 Statement 2

Incorrect Correct

GNP is the total value of goods and services


produced by the people of a nation.
GNP is the GDP of a country added with its
It does not matter whether the goods/ ‘income from abroad’.
services are produced within or outside of
that nation's geographic boundaries.

QUESTION 55. NjYxMjUrTW9oYW1tYWQgWW91bmlzIEFobWFkK3NhaGlsLmxhdy5qbWlAZ21haWwuY29tKzk


ElPTiA1NA==
Which of the following is/are counted in the segment of ‘Income from Abroad’ w.r.t. Gross
National Product?

1. Private remittances

2. Interest on external loans

3. External grants

Choose the correct answer:

a) 1 only
b) 1 and 2 only
c) 1 and 3 only
d) All of the above
Correct Answer: D
Your Answer: B
Explanation

Solution (d)

IASbaba
Score:
Web: http://ilp.iasbaba.com/
69.00 /
Email: ilp@iasbaba.com
Page 52 200
AIPTS/ILP VETERANS -
Exam Title :
2020 T...
Email : sahil.law.jmi@gmail.com
Contact : 9540210113

Explanation:

Gross National Product (GNP) is the GDP of a country added with its ‘income from abroad’. The
items which are counted in the segment ‘Income from Abroad’ are:

1. Trade Balance

2. Interest of External Loans

3. Private Remittances

QUESTION 56. NjYxMjUrTW9oYW1tYWQgWW91bmlzIEFobWFkK3NhaGlsLmxhdy5qbWlAZ21haWwuY29tKzk


ElPTiA1NQ==
Consider the following about GDP and GNP as comparative concepts:

1. In case of India GDP is always lower than its GNP.

2. GDP is a quantitative measure only while GNP indicates qualitative and quantitative aspects
of the economy.

3. GNP takes into account the trans-boundary economic activities of an economy.

Choose the correct answer:

a) 1 and 2 only
b) 1 and 3 only
c) 2 and 3 only
d) All of the above
Correct Answer: C
Your Answer: C
Explanation

Solution (c)

Elimination:

· Statement 1 can be easily eliminated if you are aware of definition of GDP and GNP. (Explained
below in table)

Statement Analysis:

Statement 1 Statement 2 Statement 3

Incorrect Correct Correct

IASbaba
Score:
Web: http://ilp.iasbaba.com/
69.00 /
Email: ilp@iasbaba.com
Page 53 200
AIPTS/ILP VETERANS -
Exam Title :
2020 T...
Email : sahil.law.jmi@gmail.com
Contact : 9540210113

In case of India, GNP is


always lower than its GDP.
GNP is more exhaustive
The normal formula is GNP = concept of national income
GDP + Income from Abroad. than the GDP as it indicates
While measuring GNP, the
about the ‘quantitative’ as
But it becomes GNP = GDP + trans-boundary economic
well as the ‘qualitative’
(– Income from Abroad) = activities of an economy is
aspect of the economy, i.e.,
GDP – Income from Abroad, also taken into account.
the ‘internal’ as well as the
in the case of India. ‘external’ strength of the
economy.
This means that India’s GNP
is always lower than its GDP.

QUESTION 57. NjYxMjUrTW9oYW1tYWQgWW91bmlzIEFobWFkK3NhaGlsLmxhdy5qbWlAZ21haWwuY29tKzk


ElPTiA1Ng==
Which one is/are considered as a 'Lender of last resort'?

a) Scheduled Banks
b) Commercial Banks
c) NABARD
d) Reserve Bank of India
Correct Answer: D
Your Answer: D
Explanation

Solution (d)

Explanation:

Reserve Bank of
Scheduled Banks Commercial banks NABARD
India

IASbaba
Score:
Web: http://ilp.iasbaba.com/
69.00 /
Email: ilp@iasbaba.com
Page 54 200
AIPTS/ILP VETERANS -
Exam Title :
2020 T...
Email : sahil.law.jmi@gmail.com
Contact : 9540210113

A Central Bank is
known by different
names in different
countries.

In India, RBI is
considered as
Scheduled Banks: 'Lender of last
refers to a bank which Commercial banks are Unlike commercial resort'.
is listed in the 2nd banks which are banks, development
Schedule of the directly dealing with banks like NABARD do When a
Reserve Bank of India customers - opening not have deposit or commercial bank
Act, 1934. deposit accounts, loan accounts and they faces financial
lending loans, issuing will simply process the crisis and fails to
Banks not under this demand drafts, loan proposals received obtain funds from
Schedule are called sending online from the customers, other sources then
non-scheduled banks. remittances etc., and sanction the loans and the central bank
the customers can the amount will be plays a vital role of
Scheduled banks are approach these bank disbursed through a lender of last
usually private, foreign people and get their commercial bank like resort. It provides
and nationalised banks services fulfilled. SBI. the financial
operating in India. assistance in form
of credit.

This role of
Central Bank
saves the
commercial bank
from bankruptcy.

QUESTION 58. NjYxMjUrTW9oYW1tYWQgWW91bmlzIEFobWFkK3NhaGlsLmxhdy5qbWlAZ21haWwuY29tKzk


ElPTiA1Nw==
Consider the following terms related to Exchange rates and identify the correctly matched pair/
s:

1. Flexible Exchange rate : : It is determined by the forces of market demand and supply.

2. Nominal Exchange rate : : Exchange rates are quoted in money terms.

3. Real Exchange rate : : Ratio of domestic to foreign prices measured in same currency.

Choose the appropriate answer:

a) 1 and 2
b) 2 and 3
c) 1 and 3
d) All of the above
Correct Answer: A

IASbaba
Score:
Web: http://ilp.iasbaba.com/
69.00 /
Email: ilp@iasbaba.com
Page 55 200
AIPTS/ILP VETERANS -
Exam Title :
2020 T...
Email : sahil.law.jmi@gmail.com
Contact : 9540210113

Your Answer: C
Explanation

Solution (a)

Statement Analysis:

Statement 1 Statement 2 Statement 3

Correct Correct Incorrect

Real Exchange Rate - The


Nominal Exchange Rate is
ratio of foreign to domestic
Flexible Exchange Rates is the rate at which one
prices, measured in the same
also known as Floating currency will be exchanged
currency.
Exchange rate which is for another.
determined by the forces of It is the purchasing power of
It is also regarded as the
market demand and supply. a currency relative to
value of one country’s
another at current exchange
In a completely flexible currency in relation to
rates and prices. It is the
system, the central banks another currency.
ratio of the number of units
follow a simple set of rules – of a given country's currency
It is also called as Bilateral
they do nothing to directly necessary to buy a market
nominal exchange rate,
affect the level of the basket of goods in the other
bilateral in the sense that
exchange rate, in other country, after acquiring the
they are exchange rates for
words they do not intervene other country's currency in
one currency against another
in the foreign exchange the foreign exchange market,
and they are nominal
market (and therefore, there to the number of units of the
because they quote the
are no official reserve given country's currency that
exchange rate in money
transactions). would be necessary to buy
terms, i.e. so many rupees
per dollar or per pound. that market basket directly in
the given country.

QUESTION 59. NjYxMjUrTW9oYW1tYWQgWW91bmlzIEFobWFkK3NhaGlsLmxhdy5qbWlAZ21haWwuY29tKzk


ElPTiA1OA==
Which organisation is meant to ensure exports from India?

a) EXIM Bank
b) ECGC
c) Ministry of Commerce
d) RBI
Correct Answer: B
Your Answer: C
Explanation

Solution (b)

IASbaba
Score:
Web: http://ilp.iasbaba.com/
69.00 /
Email: ilp@iasbaba.com
Page 56 200
AIPTS/ILP VETERANS -
Exam Title :
2020 T...
Email : sahil.law.jmi@gmail.com
Contact : 9540210113

Explanation:

ECGC Ltd. (Formerly Export Credit Guarantee Corporation of India Ltd.), wholly owned by
Government of India, was set up in 1957 with the objective of promoting exports from the
country by providing Credit Risk Insurance and related services for exports.

It functions under the administrative control of Ministry of Commerce and Industry, and is
managed by a Board of Directors comprising representatives of the Government, Reserve Bank
of India, banking, and insurance and exporting community.

Over the years it has designed different export credit risk insurance products to suit the
requirements of Indian exporters and commercial banks extending export credit.

ECGC is essentially an export promotion organization, seeking to improve the competitiveness


of the Indian exporters by providing them with credit insurance covers. ECGC keeps its
premium rates at the optimal level.

Additional Information

What does ECGC do?

· Provides a range of credit risk insurance covers to exporters against loss in export of goods
and services

· Offers Export Credit Insurance covers to banks and financial institutions to enable exporters
to obtain better facilities from them

· Provides Overseas Investment Insurance to Indian companies investing in joint ventures


abroad in the form of equity or loan

How does ECGC help exporters?

ECGC

· Offers insurance protection to exporters against payment risks

· Provides guidance in export-related activities

· Makes available information on different countries with it's own credit ratings

· Makes it easy to obtain export finance from banks/financial institutions

· Assists exporters in recovering bad debts

· Provides information on credit-worthiness of overseas buyers

QUESTION 60. NjYxMjUrTW9oYW1tYWQgWW91bmlzIEFobWFkK3NhaGlsLmxhdy5qbWlAZ21haWwuY29tKzk


ElPTiA1OQ==
National Income estimates in India is prepared by:

a) Finance Minister

IASbaba
Score:
Web: http://ilp.iasbaba.com/
69.00 /
Email: ilp@iasbaba.com
Page 57 200
AIPTS/ILP VETERANS -
Exam Title :
2020 T...
Email : sahil.law.jmi@gmail.com
Contact : 9540210113

b) National Sample Survey Office


c) Central Statistical Office
d) GDP Survey of India
Correct Answer: C
Your Answer: B
Explanation

Solution (c)

Explanation:

The Central Statistical Office which is one of the two wings of the National Statistical
Organisation (NSO) is responsible for coordination of statistical activities in the country and for
evolving and maintaining statistical standards. Its activities include compilation of National
Accounts; conduct of Annual Survey of Industries and Economic Censuses, compilation of Index
of Industrial Production, as well as Consumer Price Indices. It also deals with various social
statistics, training, international cooperation, Industrial Classification etc.

The CSO is headed by a Director-General who is assisted by 5 Additional Director-Generals


looking after the National Accounts Division, Social Statistics Division, Economic Statistics
Division, Training Division and the Coordination and Publication Division.

QUESTION 61. NjYxMjUrTW9oYW1tYWQgWW91bmlzIEFobWFkK3NhaGlsLmxhdy5qbWlAZ21haWwuY29tKzk


lPTiA2MA==
Consider the below statements with reference to institution of Banking Ombudsman in India
and identify the incorrect statement:

a) The orders passed by the Banking Ombudsman, are not final and are not binding on the
parties concerned.
b) The Banking Ombudsman can receive and consider complaints from NRIs having accounts
in India.
c) The Banking Ombudsman is appointed by the RBI.
d) The services provided by the Banking Ombudsman are chargeable.
Correct Answer: D
Your Answer: A
Explanation

Solution (d)

Statement Analysis:

Statement (a) Statement (b) Statement (c) Statement (d)

Correct Correct Correct Incorrect

IASbaba
Score:
Web: http://ilp.iasbaba.com/
69.00 /
Email: ilp@iasbaba.com
Page 58 200
AIPTS/ILP VETERANS -
Exam Title :
2020 T...
Email : sahil.law.jmi@gmail.com
Contact : 9540210113

The Banking
Ombudsman can
receive and consider
any complaint
The Banking
relating to a number
Ombudsman is a
of deficiencies related
senior official
to banking operations
The orders passed by appointed by the
including internet
the Banking Reserve Bank of
banking. (including The service provided
Ombudsman, are not India.
from NRIs) by the Banking
final and are not Ombudsman is free
He has the
binding on the parties RBI has mentioned a from any charges.
responsibility to
concerned. large number of
redress customer
service deficiencies
complaints against
by banks to
deficiency in certain
customers where the
banking services.
customers (including
NRIs) can approach
the Ombudsman
through a complaint.

QUESTION 62. NjYxMjUrTW9oYW1tYWQgWW91bmlzIEFobWFkK3NhaGlsLmxhdy5qbWlAZ21haWwuY29tKzk


lPTiA2MQ==
Gross Domestic Product (GDP) of a country is called ‘Gross’ because it does not include

a) Intermediary products
b) Depreciation of capital
c) Indians working outside
d) Income generated by primary activities
Correct Answer: B
Your Answer: A
Explanation

Solution (b)

Explanation:

· GDP does not include the depreciation in the capital Goods.

· The net domestic product (NDP) equals the gross domestic product (GDP) minus depreciation
on a country's capital goods.

QUESTION 63. NjYxMjUrTW9oYW1tYWQgWW91bmlzIEFobWFkK3NhaGlsLmxhdy5qbWlAZ21haWwuY29tKzk


ElPTiA2Mg==
Which of the following grant(s) direct credit assistance to rural households?

IASbaba
Score:
Web: http://ilp.iasbaba.com/
69.00 /
Email: ilp@iasbaba.com
Page 59 200
AIPTS/ILP VETERANS -
Exam Title :
2020 T...
Email : sahil.law.jmi@gmail.com
Contact : 9540210113

1. Regional Rural Banks

2. National Bank for Agriculture and Rural Development

3. Land Development Banks

Select the correct answer using the codes given below:

a) 1 and 2 only
b) 2 only
c) 1 and 3 only
d) 1, 2 and 3
Correct Answer: C
Your Answer: C
Explanation

Solution (c)

Analysis:

Options 1 and 3 Option 2

Correct Incorrect

NABARD doesn’t give “direct” credit


Regional Rural Banks and Land
assistance. It provides credit via
Development Banks grants direct credit
intermediaries such as Microfinance
assistance to rural households.
companies, cooperative society, RRB.

QUESTION 64. NjYxMjUrTW9oYW1tYWQgWW91bmlzIEFobWFkK3NhaGlsLmxhdy5qbWlAZ21haWwuY29tKzk


lPTiA2Mw==
Which of the statements given below is/are correct with regard to Regional Rural Banks
(RRBs)?

1. RRBs are regulated by National Bank for Agriculture and Rural Development (NABARD)

2. RRBs operate only in rural areas.

Choose appropriate code:

a) 1 only
b) 2 only
c) Both 1 and 2
d) Neither 1 nor 2
Correct Answer: A

IASbaba
Score:
Web: http://ilp.iasbaba.com/
69.00 /
Email: ilp@iasbaba.com
Page 60 200
AIPTS/ILP VETERANS -
Exam Title :
2020 T...
Email : sahil.law.jmi@gmail.com
Contact : 9540210113

Your Answer: C
Explanation

Solution (a)

Statement Analysis:

Statement 1 Statement 2

Correct Incorrect

Regional Rural Banks are regulated by


Regional Rural Banks – Their area of
National Bank for Agriculture and Rural
operation may include urban areas too.
Development (NABARD).

Additional information:

Regional Rural Banks (RRB)

· Regional Rural Banks are scheduled commercial banks (Government bank) operating at
regional level in different States of India.

· They have been created with a view to serve primarily the rural areas of India with basic
banking and financial services. However, RRBs may have branches set up for urban operations
and their area of operation may include urban areas too.

· The area of operation of RRBs is limited to the area as notified by Government of India
covering one or more districts in the State. RRBs also perform a variety of different functions.

RRBs perform various functions in following heads:

· Providing banking facilities to rural and semi-urban areas.

· Carrying out government operations like disbursement of wages of MGNREGA workers,


distribution of pensions etc.

· Providing Para-Banking facilities like locker facilities, debit and credit cards.

· The rural banks had the legislative backing of the Regional Rural Banks Act 1976. This act
allowed the government to set up banks from time to time wherever it considered necessary.

Do you know?

The RRBs are owned by three entities with their respective shares as follows:

· Central Government → 50%

· State government → 15%

· Sponsor bank → 35%

IASbaba
Score:
Web: http://ilp.iasbaba.com/
69.00 /
Email: ilp@iasbaba.com
Page 61 200
AIPTS/ILP VETERANS -
Exam Title :
2020 T...
Email : sahil.law.jmi@gmail.com
Contact : 9540210113

QUESTION 65. NjYxMjUrTW9oYW1tYWQgWW91bmlzIEFobWFkK3NhaGlsLmxhdy5qbWlAZ21haWwuY29tKzk


ElPTiA2NA==
Which of the following terms correctly explains the term Bancassurance?

a) It is term refers to the insurance provided to demand deposits in a commercial bank.


b) It is an assurance given to the customers of a National bank, by RBI, to instil faith of
customers in the bank.
c) It refers to the selling of life insurance and other insurance products and services by a
bank.
d) None of the above
Correct Answer: C
Your Answer: Unanswered
Explanation

Solution (c)

Explanation:

Bancassurance means selling insurance product through banks. Banks and insurance company
come up in a partnership wherein the bank sells the tied insurance company's insurance
products to its clients.

Bancassurance arrangement benefits both the firms. On the one hand, the bank earns fee
amount (non interest income) from the insurance company apart from the interest income
whereas on the other hand, the insurance firm increases its market reach and customers. The
bank acts as an intermediary, helping insurance firm reach its target customer in order to
increase its market share.

QUESTION 66. NjYxMjUrTW9oYW1tYWQgWW91bmlzIEFobWFkK3NhaGlsLmxhdy5qbWlAZ21haWwuY29tKzk


ElPTiA2NQ==
The International Monetary Fund (IMF) ranks nations on the basis of ‘national income’
according to:

a) Gross Domestic Product (GDP)


b) Gross National Product (GNP)
c) Gross National Product adjusted for Purchasing power Parity
d) Gross Domestic Product adjusted for Purchasing power Parity
Correct Answer: D
Your Answer: A
Explanation

Solution (d)

Explanation:

IASbaba
Score:
Web: http://ilp.iasbaba.com/
69.00 /
Email: ilp@iasbaba.com
Page 62 200
AIPTS/ILP VETERANS -
Exam Title :
2020 T...
Email : sahil.law.jmi@gmail.com
Contact : 9540210113

According to IMF, purchasing power Parity (PPP) adjusts the national income for standard of
living in every nation so that an accurate estimate of national wealth may be arrived at.

India is currently ranked as the 3rd largest economy in terms of GDP adjusted for PPP by IMF
after USA and Japan.

QUESTION 67. NjYxMjUrTW9oYW1tYWQgWW91bmlzIEFobWFkK3NhaGlsLmxhdy5qbWlAZ21haWwuY29tKzk


ElPTiA2Ng==
Which of the following can be called as 'factors of production'?

1. Human labour

2. Land

3. Consumer goods

4. Capital goods

Select the correct answer using the codes below.

a) 1, 2 and 3 only
b) 2, 3 and 4 only
c) 1 only
d) 1, 2 and 4 only
Correct Answer: D
Your Answer: D
Explanation

Solution (d)

Explanation:

Options 1 and 2 Option 3 Option 4

Correct Incorrect Correct

Consumer goods are finished


Capital goods are machines
products. They are not
Human labour and land which are used to produce
factors of production, but
contribute to production. other goods. So, statement 4
products. So, statement 3 is
is also correct.
wrong.

Additional information:

· The factors of production include land, labor, capital and entrepreneurship.

IASbaba
Score:
Web: http://ilp.iasbaba.com/
69.00 /
Email: ilp@iasbaba.com
Page 63 200
AIPTS/ILP VETERANS -
Exam Title :
2020 T...
Email : sahil.law.jmi@gmail.com
Contact : 9540210113

· Capital can be further divided into fixed (machines etc) and working capital (money, raw
material etc).

QUESTION 68. NjYxMjUrTW9oYW1tYWQgWW91bmlzIEFobWFkK3NhaGlsLmxhdy5qbWlAZ21haWwuY29tKzk


ElPTiA2Nw==
The Reserve Bank of India supervises the functioning of banks in which of the following ways?

1. RBI monitors the banks so that they maintain prescribed cash balance.

2. RBI by its mandate prohibits banks from earning excessive profits.

3. RBI supervises the credit activities of lenders in the informal sector.

4. RBI ensures that bank loans are diverted towards priority sectors to achieve inclusive
growth.

Select the correct answer using the codes below.

a) 2, 3 and 4 only
b) 2 and 3 only
c) 1 and 4 only
d) 1, 2, 3 and 4
Correct Answer: C
Your Answer: C
Explanation

Solution (c)

Statement analysis:

Statement 1 Statement 2 Statement 3 Statement 4

Correct Incorrect Incorrect Correct

IASbaba
Score:
Web: http://ilp.iasbaba.com/
69.00 /
Email: ilp@iasbaba.com
Page 64 200
AIPTS/ILP VETERANS -
Exam Title :
2020 T...
Email : sahil.law.jmi@gmail.com
Contact : 9540210113

Similarly the RBI sees


that the banks give
loans not just to
profit-making
businesses and
Banks maintain a
Statement 2 is very There is no traders but also to
minimum cash
extreme. organisation which small cultivators,
balance out of the
supervises the credit small scale industries,
deposits they receive. There is no such activities of lenders in to small borrowers
mandate which the informal sector. etc.
The RBI monitors the
prohibits banks from
banks in actually
earning excessive They lend at arbitrary Periodically, banks
maintaining cash
profits. interest rates. have to submit
balance.
information to the
RBI on how much
they are lending, to
whom, at what
interest rate, etc.

QUESTION 69. NjYxMjUrTW9oYW1tYWQgWW91bmlzIEFobWFkK3NhaGlsLmxhdy5qbWlAZ21haWwuY29tKzk


ElPTiA2OA==
With reference to economy, stagflation occurs when there is:

1. Decline in economic growth

2. High unemployment

3. High inflation

Select the correct answer using the code given below.

a) 1 and 2 only
b) 1 and 3 only
c) 2 and 3 only
d) 1, 2 and 3
Correct Answer: D
Your Answer: D
Explanation

Solution (d)

Approach/explanation:

Stagflation is a condition of slow economic growth and relatively high unemployment -


economic stagnation - accompanied by rising prices, or inflation, or inflation and a decline in
Gross Domestic Product (GDP).

IASbaba
Score:
Web: http://ilp.iasbaba.com/
69.00 /
Email: ilp@iasbaba.com
Page 65 200
AIPTS/ILP VETERANS -
Exam Title :
2020 T...
Email : sahil.law.jmi@gmail.com
Contact : 9540210113

Stagflation can prove to be a particularly tough problem for governments to deal with due to
the fact that most policies designed to lower inflation tend to make it tougher for the
unemployed, and policies designed to ease unemployment raise inflation.

QUESTION 70. NjYxMjUrTW9oYW1tYWQgWW91bmlzIEFobWFkK3NhaGlsLmxhdy5qbWlAZ21haWwuY29tKzk


ElPTiA2OQ==
Which of the following is incorrectly matched w.r.t. inflation?

a) Core Inflation – Includes the price rise in all goods and services including food and fuel.
b) Headline Inflation – Includes persistent rise in prices of goods and services, including
commodities such as food and energy prices.
c) Structural Inflation – Inflation built into the economy due to government’s monetary policy
d) All of the above
Correct Answer: A
Your Answer: C
Explanation

Solution (a)

Statement Analysis:

Statement (a) Statement (b) Statement (c)

Incorrect Correct Correct

Structural inflation arises out


of shortfalls in the supply of
goods, a general crisis of a
developing economy, rising
Headline inflation is a demand but lack of investible
measure of the total inflation capital to produce the
Core Inflation shows price within an economy, including required level of goods.
rise in all goods and services commodities such as food
excluding energy and food and energy prices (e.g., oil Whenever the Government
articles. and gas), which tend to be managed to go for higher
much more volatile and growths by managing higher
prone to inflationary spikes. investible capital it had
inflationary pressures on the
economy and growth was
sacrificed at the altar of
lower inflation.

IASbaba
Score:
Web: http://ilp.iasbaba.com/
69.00 /
Email: ilp@iasbaba.com
Page 66 200
AIPTS/ILP VETERANS -
Exam Title :
2020 T...
Email : sahil.law.jmi@gmail.com
Contact : 9540210113

QUESTION 71. NjYxMjUrTW9oYW1tYWQgWW91bmlzIEFobWFkK3NhaGlsLmxhdy5qbWlAZ21haWwuY29tKzk


lPTiA3MA==
Which of the following is not helpful in controlling money supply?

a) Free market policy


b) CRR
c) Bank rate
d) Change in margin requirement
Correct Answer: A
Your Answer: D
Explanation

Solution (a)

Explanation/Approach

Central Bank, i.e. RBI in India, controls the quantity of money in an economy through measures
like - reserve requirement, CRR, Bank rate, Open Market Operations, Quantitative Easing
Program etc. (not Free market policy)

Central banks work hard to ensure that a nation's economy remains healthy. One way central
banks do this is by controlling the amount of money circulating in the economy. They can do this
by influencing interest rates, setting reserve requirements, and employing open market
operation tactics, among other approaches. Having the right quantity of money in circulation is
crucial to ensuring a healthy and sustainable economy.

QUESTION 72. NjYxMjUrTW9oYW1tYWQgWW91bmlzIEFobWFkK3NhaGlsLmxhdy5qbWlAZ21haWwuY29tKzk


lPTiA3MQ==
Which of the following entities can issue a ‘Certificate of Deposit’?

1. Commercial Banks

2. Financial Institutions

3. Private companies

4. Post Offices

Select the code from following:

a) 1 and 2
b) 3 only
c) 1, 2 and 4
d) All of the above
Correct Answer: A
Your Answer: D
Explanation

IASbaba
Score:
Web: http://ilp.iasbaba.com/
69.00 /
Email: ilp@iasbaba.com
Page 67 200
AIPTS/ILP VETERANS -
Exam Title :
2020 T...
Email : sahil.law.jmi@gmail.com
Contact : 9540210113

Solution (a)

Explanation:

Certificate of Deposit (CD) is a negotiable money market instrument and issued in


dematerialised form or as a Usance Promissory Note against funds deposited at a bank or other
eligible financial institution for a specified time period.

Guidelines for issue of CDs are presently governed by various directives issued by the Reserve
Bank of India (RBI), as amended from time to time.

CDs can be issued by –

· Scheduled commercial banks (excluding Regional Rural Banks and Local Area Banks); and

· Select All-India Financial Institutions (FIs) that have been permitted by RBI to raise short-term
resources within the umbrella limit fixed by RBI.

Options 1 and 2 Options 3 and 4

Correct Incorrect

CDs can be issued by –

· Scheduled commercial banks (excluding


Regional Rural Banks and Local Area Guidelines for issue of CDs are presently
Banks); and governed by various directives issued by the
Reserve Bank of India (RBI) and doesn’t
· Select All-India Financial Institutions (FIs) include private companies or post offices.
that have been permitted by RBI to raise
short-term resources within the umbrella
limit fixed by RBI.

QUESTION 73. NjYxMjUrTW9oYW1tYWQgWW91bmlzIEFobWFkK3NhaGlsLmxhdy5qbWlAZ21haWwuY29tKzk


ElPTiA3Mg==
Which of the following is incorrect about ‘National Income’ (NI)?

a) Net National Product at factor cost is NI.


b) National Income divided by the population of the country is Per Capita Income (PCI).
c) India is calculating NI based on factor cost since 2015.
d) In India CSO calculates the NI statistics.
Correct Answer: C
Your Answer: C
Explanation

Solution (c)

IASbaba
Score:
Web: http://ilp.iasbaba.com/
69.00 /
Email: ilp@iasbaba.com
Page 68 200
AIPTS/ILP VETERANS -
Exam Title :
2020 T...
Email : sahil.law.jmi@gmail.com
Contact : 9540210113

Statement Analysis:

Statement 1 Statement 2 Statement 3 Statement 4

Correct Correct Incorrect Correct

India officially used to


Per Capita Income calculate it's national
National Income (NI) (PCI) can be income at factor cost
In India CSO
can be measured in measured by dividing . Since January 2015,
calculates the NI
terms of Net National National Income by the CSO has switched
statistics.
Product at factor cost. population of the over to calculating it
country. at market price or
market cost.

QUESTION 74. NjYxMjUrTW9oYW1tYWQgWW91bmlzIEFobWFkK3NhaGlsLmxhdy5qbWlAZ21haWwuY29tKzk


lPTiA3Mw==
In an economy a situation of liquidity trap is characterised by which of the following?

1. High savings rate

2. High demand for bonds

3. Expansionary monetary policy

Select the correct answer using the code given below:

a) 1 and 2 only
b) 2 and 3 only
c) 1 and 3 only
d) 1, 2 and 3
Correct Answer: C
Your Answer: D
Explanation

Solution (c)

Explanation:

Liquidity trap is a situation when expansionary monetary policy (increase in money supply) does
not increase the interest rate, income and hence does not stimulate economic growth.

It is the situation in which prevailing interest rates are low and savings rates are high, making
monetary policy ineffective.

IASbaba
Score:
Web: http://ilp.iasbaba.com/
69.00 /
Email: ilp@iasbaba.com
Page 69 200
AIPTS/ILP VETERANS -
Exam Title :
2020 T...
Email : sahil.law.jmi@gmail.com
Contact : 9540210113

In a liquidity trap, consumers choose to avoid bonds and keep their funds in savings, because of
the prevailing belief that interest rates will soon rise. Because bonds have an inverse
relationship to interest rates, many consumers do not want to hold an asset with a price that is
expected to decline.

QUESTION 75. NjYxMjUrTW9oYW1tYWQgWW91bmlzIEFobWFkK3NhaGlsLmxhdy5qbWlAZ21haWwuY29tKzk


ElPTiA3NA==
Which of the following statement correctly defines a ‘Nostro’ Account?

a) It is an account which a foreign bank will have in an Indian back in foreign currency.
b) It is an account which an Indian bank has to maintain in RBI.
c) It is an account which an Indian bank holds in an overseas bank in foreign currency.
d) It is an account which Central government has to maintain in RBI.
Correct Answer: C
Your Answer: Unanswered
Explanation

Solution (c)

Explanation:

Nostro account refers to an account that a bank holds in a foreign currency in another bank.
Nostros, a term derived from the Latin word for "ours," are frequently used to facilitate foreign
exchange and trade transactions. The opposite term vostro accounts, derived from the Latin
word for "yours," is how a bank refers to the accounts that other banks have on its books in its
home currency.

Nostro account and vostro account refer to the same thing from a different perspective. For
example, Bank X has an account with Bank Y in Bank Y's home currency. To Bank X, that is a
nostro, meaning "our account on your books," while to Bank Y, it is a vostro, meaning "your
account on our books." These accounts are used to facilitate international transactions and to
settle transactions that hedge exchange rate risk.

QUESTION 76. NjYxMjUrTW9oYW1tYWQgWW91bmlzIEFobWFkK3NhaGlsLmxhdy5qbWlAZ21haWwuY29tKzk


ElPTiA3NQ==
Consider the following statements regarding Regional rural Banks (RRBs):

1. They provide direct loans to small and marginal farmers.

2. They are co-sponsored by the Reserve Bank of India.

3. They also perform other banking operations.

4. State Governments are share holders in RRBs.

Which of the above statements are correct about RRBs?

IASbaba
Score:
Web: http://ilp.iasbaba.com/
69.00 /
Email: ilp@iasbaba.com
Page 70 200
AIPTS/ILP VETERANS -
Exam Title :
2020 T...
Email : sahil.law.jmi@gmail.com
Contact : 9540210113

a) 1, 2 and 3
b) 2, 3 and 4
c) 1, 3 and 4
d) All of the above
Correct Answer: C
Your Answer: C
Explanation

Solution (c)

Explanation:

Regional Rural Banks (also RRBs) are local level banking organizations operating in different
States of India. They have been created with a view to serve primarily the rural areas of India
with basic banking and financial services. However, RRBs may have branches set up for urban
operations and their area of operation may include urban areas too.

The area of operation of RRBs is limited to the area as notified by Government of India covering
one or more districts in the State. RRBs also perform a variety of different functions.

RRBs perform various functions in following heads

· Providing banking facilities to rural and semi-urban areas. Carrying out government
operations like disbursement of wages of MGNREGA workers, distribution of pensions etc.

· Providing Para-Banking facilities like locker facilities, debit and credit cards.

The Regional Rural Bank were owned by the Central Government, the State Government and
the Sponsor Bank (There were five commercial banks, Punjab National Bank, State Bank of
India, Syndicate Bank, United Bank of India and United Commercial Bank, which sponsored the
regional rural banks) who held shares in the ratios as follows Central Government-50%, State
Government- 15% and Sponsor Banks- 35%. Earlier, Reserve Bank of India had laid down
ceilings on the rate of interest to be charged by these RRBs.

Statement 1 Statement 2 Statement 3 Statement 4

Correct Incorrect Correct Correct

IASbaba
Score:
Web: http://ilp.iasbaba.com/
69.00 /
Email: ilp@iasbaba.com
Page 71 200
AIPTS/ILP VETERANS -
Exam Title :
2020 T...
Email : sahil.law.jmi@gmail.com
Contact : 9540210113

Regional Rural Banks


(also RRBs) are local Share holders ratio
level banking
The Regional Rural
organizations. · Central
Banks are owned and RRBs provide other
sponsored by Central Para-Banking Government-50%
They serve primarily
the rural areas of Government, the facilities like locker
· State Government-
India with basic State Government facilities, debit and
15% and
banking and financial and the Sponsor credit cards.
services. They provide Bank. · Sponsor Banks-
direct loans to small 35%.
and marginal farmers.

QUESTION 77. NjYxMjUrTW9oYW1tYWQgWW91bmlzIEFobWFkK3NhaGlsLmxhdy5qbWlAZ21haWwuY29tKzk


ElPTiA3Ng==
‘Purchasing Power Parity (PPP)’ is a type of ‘Exchange Rate System’. Which of the following
statements is/are correct about PPP?

1. Exchange rates of various currencies are determined on the basis of their relative purchasing
powers in the respective countries.

2. This exchange rate can be used to buy and sell currencies in FOREX market.

Select the code from below:

a) 1 only
b) 2 only
c) Both 1 and 2
d) Neither 1 nor 2
Correct Answer: A
Your Answer: A
Explanation

Solution (a)

Statement 1 Statement 2

Correct Incorrect

IASbaba
Score:
Web: http://ilp.iasbaba.com/
69.00 /
Email: ilp@iasbaba.com
Page 72 200
AIPTS/ILP VETERANS -
Exam Title :
2020 T...
Email : sahil.law.jmi@gmail.com
Contact : 9540210113

The concept of purchasing power parity


allows one to estimate what the exchange
‘Purchasing Power Parity (PPP)’ exchange
rate between two currencies would have to
rate system cannot be used to buy and sell
be in order for the exchange to be at par
currencies in FOREX market.
with the purchasing power of the two
countries' currencies.

Additional Information

Using the PPP rate for hypothetical currency conversions, a given amount of one currency thus
has the same purchasing power whether used directly to purchase a market basket of goods or
used to convert at the PPP rate to the other currency and then purchase the market basket
using that currency. Observed deviations of the exchange rate from purchasing power parity
are measured by deviations of the real exchange rate from its PPP value of 1.

PPP exchange rates help to minimize misleading international comparisons that can arise with
the use of market exchange rates. For example, suppose that two countries produce the same
physical amounts of goods as each other in each of two different years. Since market exchange
rates fluctuate substantially, when the GDP of one country measured in its own currency is
converted to the other country's currency using market exchange rates, one country might be
inferred to have higher real GDP than the other country in one year but lower in the other; both
of these inferences would fail to reflect the reality of their relative levels of production. But if
one country's GDP is converted into the other country's currency using PPP exchange rates
instead of observed market exchange rates, the false inference will not occur.

Essentially GDP PPP controls for the different costs of living and price level, usually relative to
the United States Dollar, that would make an accurate depiction of a given nation's gross
income.

QUESTION 78. NjYxMjUrTW9oYW1tYWQgWW91bmlzIEFobWFkK3NhaGlsLmxhdy5qbWlAZ21haWwuY29tKzk


ElPTiA3Nw==
A rapid decrease in the rate of inflation is sometimes attributed to the base effect. What is base
effect?

a) It is the impact of the price levels of previous year on the calculation of inflation rate
b) It is impact of drastic deficiency in supply due to failure of crops
c) It is the impact of the surge in demand due to rapid economic growth
d) None of the statements above are correct
Correct Answer: A
Your Answer: A
Explanation

Solution (a)

Explanation:

IASbaba
Score:
Web: http://ilp.iasbaba.com/
69.00 /
Email: ilp@iasbaba.com
Page 73 200
AIPTS/ILP VETERANS -
Exam Title :
2020 T...
Email : sahil.law.jmi@gmail.com
Contact : 9540210113

The Base effect relates to inflation in the corresponding period of the previous year, if the
inflation rate was too low in the corresponding period of the previous year, even a smaller rise
in the Price Index will arithmetically give a high rate of inflation now. On the other hand, if the
price index had risen at a high rate in the corresponding period of the previous year and
recorded high inflation rate, a similar absolute increase in the Price index now will show a
lower inflation rate now.

QUESTION 79. NjYxMjUrTW9oYW1tYWQgWW91bmlzIEFobWFkK3NhaGlsLmxhdy5qbWlAZ21haWwuY29tKzk


ElPTiA3OA==
Which of the following statements is/are correct about Peer-to-peer lending (P2P) platforms?

1. They are treated as non-banking financial companies (NBFCs)

2. They are regulated by the Reserve Bank of India (RBI)

3. They act as intermediaries between borrower, lender, and partner bank

Choose appropriate code:

a) 3 only
b) 1 and 3
c) 1 and 2
d) 1, 2 and 3
Correct Answer: D
Your Answer: A
Explanation

Solution (d)

Explanation:

P2P lending is a crowd-funding model (largely online) where people looking to invest their
money with people who want to borrow can do so. The concept is centred around savers getting
higher interest by lending their money instead of saving and borrowers get comparatively lower
interest rates.

Borrowers are either individuals or small businesses. But unlike a traditional savings account,
one can lose money if the borrower defaults.

Peer-to-peer lending (P2P) platforms would be treated as non-banking financial companies


(NBFCs) and regulated by the Reserve Bank of India (RBI)

Do you know?

An NBFC-P2P shall

· act as an intermediary providing an online marketplace or platform to the participants


involved in Peer to Peer lending;

IASbaba
Score:
Web: http://ilp.iasbaba.com/
69.00 /
Email: ilp@iasbaba.com
Page 74 200
AIPTS/ILP VETERANS -
Exam Title :
2020 T...
Email : sahil.law.jmi@gmail.com
Contact : 9540210113

· not raise deposits as defined by or under Section 45I(bb) of the Act or the Companies Act,
2013;

· not lend on its own;

· not provide or arrange any credit enhancement or credit guarantee;

· not facilitate or permit any secured lending linked to its platform; i.e. only clean loans will be
permitted;

· not hold, on its own balance sheet, funds received from lenders for lending, or funds received
from borrowers for servicing loans;

· not cross sell any product except for loan specific insurance products;

· not permit international flow of funds;

· ensure adherence to legal requirements applicable to the participants as prescribed under


relevant laws.

· store and process all data relating to its activities and participants on hardware located within
India.

· undertake due diligence on the participants;

· undertake credit assessment and risk profiling of the borrowers and disclose the same to their
prospective lenders;

· require prior and explicit consent of the participant to access its credit information;

· undertake documentation of loan agreements and other related documents;

· provide assistance in disbursement and repayments of loan amount;

· render services for recovery of loans originated on the platform.

QUESTION 80. NjYxMjUrTW9oYW1tYWQgWW91bmlzIEFobWFkK3NhaGlsLmxhdy5qbWlAZ21haWwuY29tKzk


ElPTiA3OQ==
Which of the following agencies regulates the Non Banking Financial Company (NBFC)?

a) RBI
b) SEBI
c) Finance Ministry
d) State Governments
Correct Answer: A
Your Answer: B
Explanation

Solution (a)

IASbaba
Score:
Web: http://ilp.iasbaba.com/
69.00 /
Email: ilp@iasbaba.com
Page 75 200
AIPTS/ILP VETERANS -
Exam Title :
2020 T...
Email : sahil.law.jmi@gmail.com
Contact : 9540210113

Explanation:

A Non Banking Financial Company (NBFC) is a company registered under the Companies Act,
1956 of India, engaged in the business of loans and advances, acquisition of shares, stock,
bonds hire-purchase, insurance business or chit business but does not include any institution
whose principal business includes agriculture, industrial activity or the sale, purchase or
construction of immovable property.

The working and operations of NBFCs are regulated by the Reserve Bank of India (RBI) within
the framework of the Reserve Bank of India Act, 1934 (Chapter III B) and the directions issued
by it.

QUESTION 81. NjYxMjUrTW9oYW1tYWQgWW91bmlzIEFobWFkK3NhaGlsLmxhdy5qbWlAZ21haWwuY29tKzk


lPTiA4MA==
Through Open Market Operations, the RBI purchases and sells

1. Government securities

2. Gold

3. Forex

4. Shares of PSUs

Select the code from following:

a) 1 only
b) 2 and 3
c) 1,2 and 3
d) All of the above
Correct Answer: A
Your Answer: C
Explanation

Solution (a)

Approach/explanation:

Open Market Operations refer to the purchase and sale of the Government securities (G-Secs)
by RBI from / to market. The objective of Open Market Operations is to adjust the rupee
liquidity conditions in the economy on a durable basis.

When RBI sells government security in the markets, the banks purchase them. When the banks
purchase Government securities, they have a reduced ability to lend to the industrial houses or
other commercial sectors. This reduced surplus cash, contracts the rupee liquidity and
consequently credit creation / credit supply. When RBI purchases the securities, the commercial
banks find them with more surplus cash and this would create more credit in the system. Thus,
in the case of excess liquidity, RBI resorts to sale of G-secs to suck out rupee from system.

IASbaba
Score:
Web: http://ilp.iasbaba.com/
69.00 /
Email: ilp@iasbaba.com
Page 76 200
AIPTS/ILP VETERANS -
Exam Title :
2020 T...
Email : sahil.law.jmi@gmail.com
Contact : 9540210113

Similarly, when there is a liquidity crunch in the economy, RBI buys securities from the market,
thereby releasing liquidity. It’s worth note here that the market for government securities is not
well developed in India but still OMO plays very important role.

QUESTION 82. NjYxMjUrTW9oYW1tYWQgWW91bmlzIEFobWFkK3NhaGlsLmxhdy5qbWlAZ21haWwuY29tKzk


lPTiA4MQ==
The Indian rupee is a legal tender in which of the following countries?

1. Bangladesh

2. Pakistan

3. Nepal

4. Bhutan

Select the code from following:

a) 1 and 2
b) 3 and 4
c) 1, 3 and 4
d) None of the above
Correct Answer: B
Your Answer: D
Explanation

Solution (b)

Explanation:

Indian currency is considered as a Legal tender in Nepal and Bhutan.

Legal tender is any official medium of payment recognized by law that can be used to
extinguish a public or private debt, or meet a financial obligation.

The national currency is legal tender in practically every country. A creditor is obligated to
accept legal tender toward repayment of a debt. Legal tender can only be issued by the national
body that is authorized to do so.

QUESTION 83. NjYxMjUrTW9oYW1tYWQgWW91bmlzIEFobWFkK3NhaGlsLmxhdy5qbWlAZ21haWwuY29tKzk


ElPTiA4Mg==
Consider the following

1. Creeping – When inflation is between 1 % and 5 %

2. Trotting – When inflation is between 6 % and 10 %

IASbaba
Score:
Web: http://ilp.iasbaba.com/
69.00 /
Email: ilp@iasbaba.com
Page 77 200
AIPTS/ILP VETERANS -
Exam Title :
2020 T...
Email : sahil.law.jmi@gmail.com
Contact : 9540210113

3. Galloping – When inflation is between 11 % and 20 %

4. Runaway – When inflation is between 21 % and 50 %

5. Hyperinflation – When inflation is very high and is above 50 %

Which of the above is/are correctly matched?

a) 1, 3 and 5
b) 1, 2, 4 and 5
c) 1, 2, 3, 4 and 5
d) 2, 3, 4 and 5
Correct Answer: C
Your Answer: A
Explanation

Solution (c)

Types of inflation with respect to rate

Inflation is divided based on the rate, but there are no clear lines of demarcation and hence you
can see them being used often interchangeably

· Creeping – When inflation is between 1 % and 5 %

· Trotting – When inflation is between 6 % and 10 %

· Galloping – When inflation is between 11 % and 20 %

· Runaway – When inflation is between 21 % and 50 %

· Hyperinflation – When inflation is very high and is above 50 %

QUESTION 84. NjYxMjUrTW9oYW1tYWQgWW91bmlzIEFobWFkK3NhaGlsLmxhdy5qbWlAZ21haWwuY29tKzk


lPTiA4Mw==
Consider the following statements

1. Currency Deposit Ratio is the ratio of money held by the public in currency to that they hold
in bank deposits

2. Reserve Deposit Ratio is the proportion of the total deposits, commercial banks keep as
reserves.

Which of the above is/are true?

a) 1 only
b) 2 only
c) Both
d) None

IASbaba
Score:
Web: http://ilp.iasbaba.com/
69.00 /
Email: ilp@iasbaba.com
Page 78 200
AIPTS/ILP VETERANS -
Exam Title :
2020 T...
Email : sahil.law.jmi@gmail.com
Contact : 9540210113

Correct Answer: C
Your Answer: C
Explanation

Solution (c)

Statement Analysis:

Statement 1 Statement 2

Currency Deposit Ratio (CDR) Reserve Deposit Ratio (RDR)

Correct Correct

It is the proportion of the total deposits,


commercial banks keep as reserves.

The deposits by people in banks are used by


banks to lend others but not whole of the
As name suggests it is the ratio of money
deposits are lend, part of it is kept as
held by the public in currency to that they
reserve money.
hold in bank deposits.
Reserve money is divided into 2 parts – vault
CDR = CU/DD
cash in banks and deposits of commercial
Mostly people prefer to keep cash instead of banks with RBI.
deposits and it also depends on seasonal
The first part i.e. vault cash in banks is used
pattern of expenditure. For example -
to meet the cash demands of depositors.
During the festivals people keep more cash
holdings, as the expenditures are more. Keeping reserves is costly for banks as they
can lend this part as well and can earn
interest but RBI ensures that they keep
reserves so they can meet the cash demands
of account holders without any problem.

QUESTION 85. NjYxMjUrTW9oYW1tYWQgWW91bmlzIEFobWFkK3NhaGlsLmxhdy5qbWlAZ21haWwuY29tKzk


ElPTiA4NA==
This term is used to refer the situation where measures are taken to curb deflation.

a) Hyperinflation
b) Reflation
c) Disinflation
d) None
Correct Answer: B
Your Answer: D
Explanation

IASbaba
Score:
Web: http://ilp.iasbaba.com/
69.00 /
Email: ilp@iasbaba.com
Page 79 200
AIPTS/ILP VETERANS -
Exam Title :
2020 T...
Email : sahil.law.jmi@gmail.com
Contact : 9540210113

Solution (b)

Terms in Inflation:

1. Stagflation: Situation when inflation and unemployment is very high but has slow economic
growth.

2. Deflation: Simply negative inflation (below zero) is called Deflation. Where prices are falling.

· Low money or less demand à Lowering of prices à Lesser production at factories à Lesser
investment

· It goes in a spiral if not controlled which is also known as deflationary spiral. Similarly when it
happens in the positive direction with inflation, it’s known as Inflator-y spiral or wage-price
spiral.

3. Disinflation: Considerable slowdown of inflation rate with respect to previous period but
still is in positive. (Remember this is different from deflation as deflation is inflation below zero)

4. Reflation: This term is used to refer the situation where measures are taken to curb
deflation. Steps can be like fiscal policy (reducing taxes) or monetary policy (increasing money
supply or reducing interest rates)

5. Slowdown – Recession & Depression: These three terms are inter-linked. Generally first
slowdown occurs, then recession comes and finally depression occurs

· Slowdown: A decline in economy of a country – Link it with GDP, not inflation

· Recession: If slowdown occurs for 2 successive quarters i.e. GDP falls for 2 successive
quarters, it is known as recession. Common indicators are fall in GDP and investments

· Depression: Generally if recession lasts long, it is said that the economy is in depression.
Main indicators are huge fall in demand of goods and services with a sharp decline in GDP and
investments. Ex: Great Depression of 1930

QUESTION 86. NjYxMjUrTW9oYW1tYWQgWW91bmlzIEFobWFkK3NhaGlsLmxhdy5qbWlAZ21haWwuY29tKzk


ElPTiA4NQ==
Under the flexible exchange rate system, the exchange rate is determined

a) By the Central Bank


b) By the World Trade Organisation
c) Predominantly by market mechanism
d) As a weighted index of a group of currencies
Correct Answer: C
Your Answer: C
Explanation

Solution (c)

IASbaba
Score:
Web: http://ilp.iasbaba.com/
69.00 /
Email: ilp@iasbaba.com
Page 80 200
AIPTS/ILP VETERANS -
Exam Title :
2020 T...
Email : sahil.law.jmi@gmail.com
Contact : 9540210113

Explanation:

Flexible exchange rate

Flexible exchange rates can be defined as exchange rates determined by global supply and
demand of currency. In other words, they are prices of foreign exchange determined by the
market, that can rapidly change due to supply and demand, and are not pegged nor controlled
by central banks. The opposite scenario, where central banks intervene in the market with
purchases and sales of foreign and domestic currency in order to keep the exchange rate within
limits, also known as bands, is called fixed exchange rate.

QUESTION 87. NjYxMjUrTW9oYW1tYWQgWW91bmlzIEFobWFkK3NhaGlsLmxhdy5qbWlAZ21haWwuY29tKzk


ElPTiA4Ng==
Which one of the following in not a part of service sector in India?

a) Transport
b) Construction
c) Hotels and restaurants
d) Insurance
Correct Answer: B
Your Answer: D
Explanation

Solution (b)

Secondary activities add value to natural resources by transforming raw materials into valuable
products. Secondary activities, therefore, are concerned with manufacturing, processing and
construction (infrastructure) industries.

People engaged in secondary activities are called blue collar workers.

QUESTION 88. NjYxMjUrTW9oYW1tYWQgWW91bmlzIEFobWFkK3NhaGlsLmxhdy5qbWlAZ21haWwuY29tKzk


ElPTiA4Nw==
Which among the below is not correct in regard to Capital Expenditure?

a) It includes those expenditure that create permanent assets and yield periodical income.
b) It is essential that the concrete assets should be productive or revenue producing in
character.
c) Repayment of loan, investment in shares, loans by Central government to state
government/s, foreign government/s and government companies – are all part of capital
expenditure.
d) Expenditure on a temporary asset cannot ordinarily be considered as expenditure of capital
nature.
Correct Answer: B

IASbaba
Score:
Web: http://ilp.iasbaba.com/
69.00 /
Email: ilp@iasbaba.com
Page 81 200
AIPTS/ILP VETERANS -
Exam Title :
2020 T...
Email : sahil.law.jmi@gmail.com
Contact : 9540210113

Your Answer: B
Explanation

Solution (b)

Statement Analysis:

Statement (a) Statement (b) Statement (c) Statement (d)

Correct Incorrect Correct Correct

Capital expenditure
may be generally
defined as
expenditure incurred
with the object of
either increasing
concrete assets of a
material (eg
construction of roads,
industrial buildings or
equipments) and
permanent character
or of reducing Repayment of loan,
recurring liabilities investment in shares,
(repayment of loan). It is not essential that loans by Central Expenditure on a
the concrete assets government to state temporary asset
A productive asset
should be productive government/s, foreign cannot ordinarily be
may be considered as
in character or that government/s and considered as
one which produces
they should even be government expenditure of capital
sufficient revenue to
revenue producing. companies – are all nature.
afford a surplus over
part of capital
all charges relevant
expenditure.
to its functioning.

It may on rare
occasions be
necessary and
justifiable to treat as
capital a scheme not
commercially
remunerative but
involving large
expenditure, say for
the construction of a
new city.

IASbaba
Score:
Web: http://ilp.iasbaba.com/
69.00 /
Email: ilp@iasbaba.com
Page 82 200
AIPTS/ILP VETERANS -
Exam Title :
2020 T...
Email : sahil.law.jmi@gmail.com
Contact : 9540210113

QUESTION 89. NjYxMjUrTW9oYW1tYWQgWW91bmlzIEFobWFkK3NhaGlsLmxhdy5qbWlAZ21haWwuY29tKzk


ElPTiA4OA==
With reference to Market Stabilisation Scheme (MSS), consider the following statements:

1. It is a tool used by the RBI to suck out excess liquidity from the market through issue of
securities on behalf of the government.

2. The money raised under MSS is kept in a separate account called MSS Account and not
parked in the government account or utilized to fund its expenditures.

Which of the statements given above is/are true?

a) 1 only
b) 2 only
c) Both 1 and 2
d) Neither 1 nor 2
Correct Answer: C
Your Answer: C
Explanation

Solution (c)

Statement Analysis:

Statement 1 Statement 2

Correct Correct

The money raised under MSS is kept in a


separate account called MSS Account and
Market Stabilisation Scheme or MSS is a not parked in the government account or
tool used by the Reserve Bank of India to utilized to fund its expenditures.
suck out excess liquidity from the market
The money obtained under MSS should be
through issue of securities like Treasury
kept with the RBI. It should not be
Bills, Dated Securities etc. on behalf of the
transferred to the government. This is
government.
because, if it is transferred, government will
spend the money in the economy thereby
adding to liquidity.

Additional Information:

· The MSS was first introduced in April 2004. Main thing about MSS is that it is used to
withdraw excess liquidity or money from the system by selling government bonds.

· After demonetisation, huge deposits were put into the banking system. At the same time,
banks can’t lend it to customers as it is just temporary money. The RBI has instructed banks to

IASbaba
Score:
Web: http://ilp.iasbaba.com/
69.00 /
Email: ilp@iasbaba.com
Page 83 200
AIPTS/ILP VETERANS -
Exam Title :
2020 T...
Email : sahil.law.jmi@gmail.com
Contact : 9540210113

keep all the additional deposits as CRR. But here, the banks will suffer losses as they have to
pay interest to the depositors.

· To compensate banks, the MSS policy is revived. Here, banks can put the excess money
obtained from deposits in Market Stabilisation Bonds (MSBs). They can get an interest payment
as well.

QUESTION 90. NjYxMjUrTW9oYW1tYWQgWW91bmlzIEFobWFkK3NhaGlsLmxhdy5qbWlAZ21haWwuY29tKzk


ElPTiA4OQ==
Which of the following is used for the measurement of distribution of income?

a) Laffer Curve
b) Philip Curve
c) Engel’s Law
d) Gini-Lorenz Curve
Correct Answer: D
Your Answer: D
Explanation

Solution (d)

Explanation:

The distribution of income in an economy is represented by a Lorenz Curve and the degree of
income inequality is measured through the Gini Coefficient. One of the five major and common
macroeconomic goals of a government is the equitable (fair) distribution of income.

QUESTION 91. NjYxMjUrTW9oYW1tYWQgWW91bmlzIEFobWFkK3NhaGlsLmxhdy5qbWlAZ21haWwuY29tKzk


lPTiA5MA==
Which of the following are part/consequences of RBI’s Contractionary monetary policy?

1. It restricts the volume of credit available in the economy

2. Decreasing the Bank rate is the main component

3. The objective is to counter inflation

Which of the statements given above is/are correct?

a) 2 Only
b) 2 and 3 Only
c) 1, 2 and 3
d) 1 and 3 Only
Correct Answer: D
Your Answer: D
Explanation

IASbaba
Score:
Web: http://ilp.iasbaba.com/
69.00 /
Email: ilp@iasbaba.com
Page 84 200
AIPTS/ILP VETERANS -
Exam Title :
2020 T...
Email : sahil.law.jmi@gmail.com
Contact : 9540210113

Solution (d)

Do you know?

· Contractionary monetary policy is also called Dear Money policy.

Statement Analysis:

Statement 1 Statement 2 Statement 3

Correct Incorrect Correct

Contractionary monetary Decreasing Bank Rate will


Contractionary monetary
policy restricts the volume of not help the cause of
policy is used to counter
credit available in the contractionary monetary
inflation.
economy. policy.

Additional Information:

Effects of Contractionary monetary policy:

· Reduced Inflation

· May increase unemployment (as it brings down growth rates)

· Drop in business expansions owing to a high cost of credit

Tools for Contractionary monetary policy

· Increase Interest rates

· Increase reserve requirements such SLR and/or CRR of the banks

QUESTION 92. NjYxMjUrTW9oYW1tYWQgWW91bmlzIEFobWFkK3NhaGlsLmxhdy5qbWlAZ21haWwuY29tKzk


lPTiA5MQ==
Consider the following pairs

Book Author

1. An Enquiry into the Nature and Cause of the Wealth of Nations: Amartya Sen

2. The Economic Consequences of the Peace : Adam Smith

3. General Theory of Employment, Interest and Money : J M Keynes

4. Inequality Reexamined : Karl Marx

IASbaba
Score:
Web: http://ilp.iasbaba.com/
69.00 /
Email: ilp@iasbaba.com
Page 85 200
AIPTS/ILP VETERANS -
Exam Title :
2020 T...
Email : sahil.law.jmi@gmail.com
Contact : 9540210113

Which of the pairs given above is/are correctly matched?

a) 3 Only
b) 3 and 4 Only
c) 1, 2 and 3 Only
d) 4 Only
Correct Answer: A
Your Answer: Unanswered
Explanation

Solution (a)

Approach:

It’s easy to eliminate option C here. But the other three options are so closely given that it is
tough to eliminate the options unless one knows it for sure.

Generally, seeing the name of Karl Marx (deliberately framed), many might think that it might
be true that Marx would have written a book on Inequality. But avoid taking such excessive
risks.

Statement Analysis:

Statement 1 Statement 2 Statement 3 Statement 4

Incorrect Incorrect Correct Incorrect

An Enquiry into the General Theory of


The Economic Inequality
Nature and Cause of Employment, Interest
Consequences of the Reexamined is
the Wealth of and Money is
Peace is authored by J authored by Amartya
Nations is authored authored by J M
M Keynes Sen.
by Adam Smith. Keynes

QUESTION 93. NjYxMjUrTW9oYW1tYWQgWW91bmlzIEFobWFkK3NhaGlsLmxhdy5qbWlAZ21haWwuY29tKzk


ElPTiA5Mg==
What is meant by Merchant Discount Rate (MDR)

a) It is a fee charged by a merchant from a customer for accepting payments from customers
through credit and debit cards in their establishments
b) It is a fee charged from a merchant by a bank for accepting payments from customers
through credit and debit cards in their establishments
c) It is a fee born by a merchant for providing discounts on the sale in their establishments to
the customers
d) None of the above
Correct Answer: B

IASbaba
Score:
Web: http://ilp.iasbaba.com/
69.00 /
Email: ilp@iasbaba.com
Page 86 200
AIPTS/ILP VETERANS -
Exam Title :
2020 T...
Email : sahil.law.jmi@gmail.com
Contact : 9540210113

Your Answer: B
Explanation

Solution (b)

Merchant Discount Rate is a fee charged from a merchant by a bank for accepting payments
from customers through credit and debit cards in their establishment.

Additional Information:

· The MDR compensates the bank issuing the card, the bank which puts up the swiping
machine (Point-of-Sale terminal) and network providers such as Mastercard or Visa for their
services

· RBI specifies the maximum MDR charges that can be levied on every card transaction.

QUESTION 94. NjYxMjUrTW9oYW1tYWQgWW91bmlzIEFobWFkK3NhaGlsLmxhdy5qbWlAZ21haWwuY29tKzk


lPTiA5Mw==
Reserve Bank of India acts as a banker to which of the following entities?

a) To Union Government only


b) To Union Government, all the State Governments and local Governments also
c) To Union Government and all the State Governments except Jammu and Kashmir
d) To Union Government and all the State Governments except Sikkim
Correct Answer: D
Your Answer: A
Explanation

Solution (d)

Currently, the Reserve Bank acts as banker to all the State Governments in India (including
Union Territory of Puducherry), except Sikkim.

Additional Information:

Since RBI is a government’s banker, it provides short-term credit to the government to meet
any shortfalls in its receipts over its disbursements.

RBI also provides short-term credit to state governments as ways and means advances.

QUESTION 95. NjYxMjUrTW9oYW1tYWQgWW91bmlzIEFobWFkK3NhaGlsLmxhdy5qbWlAZ21haWwuY29tKzk


ElPTiA5NA==
Which of the following statements are true about “Section 7” of the RBI Act 1934?

1. It is a provision under which the government can give directions to the RBI to take certain
actions in the public interest

2. To issue such directions the Government need not consult the Governor of RBI.

IASbaba
Score:
Web: http://ilp.iasbaba.com/
69.00 /
Email: ilp@iasbaba.com
Page 87 200
AIPTS/ILP VETERANS -
Exam Title :
2020 T...
Email : sahil.law.jmi@gmail.com
Contact : 9540210113

Choose the correct answer using the codes given below

a) 1 Only
b) 2 Only
c) Both 1 and 2
d) Neither 1 nor 2
Correct Answer: A
Your Answer: A
Explanation

Solution (a)

The Central Government may from time to time give such directions to the Bank as it may, after
consultation with the Governor of the Bank , consider necessary in the public interest.

Hence Statement 1 is correct and Statement 2 is incorrect.

QUESTION 96. NjYxMjUrTW9oYW1tYWQgWW91bmlzIEFobWFkK3NhaGlsLmxhdy5qbWlAZ21haWwuY29tKzk


ElPTiA5NQ==
Consider the following pairs

Committee Chaired by

1. High Level Task Force on Public Credit : Bimal Jalan

Registry for India

2. High Level Committee on : Nandan Nilekani

Deepening of Digital Payments

3. Committee on restructuring stressed assets and : Sunil Mehta

creating more value for public sector banks

Which of the above pairs is/are correctly matched?

a) 1 and 2 Only
b) 2 and 3 Only
c) 1, 2 and 3
d) 2 Only
Correct Answer: B
Your Answer: Unanswered
Explanation

Solution (b)

Statement Analysis:

IASbaba
Score:
Web: http://ilp.iasbaba.com/
69.00 /
Email: ilp@iasbaba.com
Page 88 200
AIPTS/ILP VETERANS -
Exam Title :
2020 T...
Email : sahil.law.jmi@gmail.com
Contact : 9540210113

Pair 1 Pair 2 Pair 3

Incorrect Correct Correct

The high-level committee on


High Level Task Force High Level Committee on
restructuring stressed assets
on Public Credit Registry for Deepening of Digital
and creating more value for
India is chaired by Payments is chaired by
public sector banks is headed
Y.M.Deosthalee. Nandan Nilekani.
by Sunil Mehta.

QUESTION 97. NjYxMjUrTW9oYW1tYWQgWW91bmlzIEFobWFkK3NhaGlsLmxhdy5qbWlAZ21haWwuY29tKzk


ElPTiA5Ng==
Consider the following statements

1. The Reserve Bank has discretionary powers to approve the voluntary amalgamation of two
banking companies

2. The Reserve Bank has discretionary powers to approve the voluntary amalgamation of a
banking company with a non-banking company

Which of the statements given above is/are correct?

a) 1 only
b) 2 only
c) Both 1 and 2
d) Neither 1 nor 2
Correct Answer: A
Your Answer: D
Explanation

Solution (a)

Statement Analysis:

Statement 1 Statement 2

Correct Incorrect

IASbaba
Score:
Web: http://ilp.iasbaba.com/
69.00 /
Email: ilp@iasbaba.com
Page 89 200
AIPTS/ILP VETERANS -
Exam Title :
2020 T...
Email : sahil.law.jmi@gmail.com
Contact : 9540210113

These powers do not extend to the voluntary


The Reserve Bank has discretionary powers amalgamation of a banking company with a
to approve the voluntary amalgamation of non-banking company where amalgamations
two banking companies under the provisions are governed by sections 391 to 394 of the
of Section 44A of the Banking Regulation Companies Act, 1956 in terms of which, the
Act, 1949. scheme of amalgamation has to be approved
by the High Court.

Additional Information:

The Banking Companies (Acquisition and Transfer of Undertakings)Acts of 1970 and 1980
provide that the Central Government, in consultation with the Reserve Bank of India (RBI), may
make a scheme, inter alia, for the amalgamation of any nationalised bank with any other
nationalised bank or any other banking institution

QUESTION 98. NjYxMjUrTW9oYW1tYWQgWW91bmlzIEFobWFkK3NhaGlsLmxhdy5qbWlAZ21haWwuY29tKzk


ElPTiA5Nw==
Justice BN Srikrishna committee recently seen in news deals with which of the following?

a) Data protection
b) Surplus reserves of RBI
c) Surplus reserves of SEBI
d) Foreign Sovereign Bonds
Correct Answer: A
Your Answer: A
Explanation

Solution (a)

The Justice BN Srikrishna committee submitted its report on the data protection law recently
(in July 2019)

On July 31, 2017, the Government of India appointed a 10-member Committee of Experts
headed by Justice B.N. Srikrishna, former Judge of the Supreme Court of India "to identify key
data protection issues in India and recommend methods of addressing them".

IASbaba
Score:
Web: http://ilp.iasbaba.com/
69.00 /
Email: ilp@iasbaba.com
Page 90 200
AIPTS/ILP VETERANS -
Exam Title :
2020 T...
Email : sahil.law.jmi@gmail.com
Contact : 9540210113

Pic: https://economictimes.indiatimes.com/img/65171932/Master.jpg

QUESTION 99. NjYxMjUrTW9oYW1tYWQgWW91bmlzIEFobWFkK3NhaGlsLmxhdy5qbWlAZ21haWwuY29tKzk


ElPTiA5OA==
Consider the following statements about Economic Census

1. It is being conducted for first time in country to ascertain the size of informal sector.

2. Ministry of Statistics and Program implementation is conducting this Census.

3. All farm and non-farm activities are considered in this census.

IASbaba
Score:
Web: http://ilp.iasbaba.com/
69.00 /
Email: ilp@iasbaba.com
Page 91 200
AIPTS/ILP VETERANS -
Exam Title :
2020 T...
Email : sahil.law.jmi@gmail.com
Contact : 9540210113

Which of the above statements is/are correct?

a) 1 and 2 only
b) 2 only
c) 1 and 3 only
d) 1, 2 and 3
Correct Answer: B
Your Answer: Unanswered
Explanation

Solution (b)

Economic Census was in news recently.

Statement 1 Statement 2 Statement 3

Incorrect Correct Incorrect

Economic Census is not


being conducted for first time
in country.

The government recently


started off the 7th Economic It is being conducted by
Census (EC) from Tripura. It Ministry of Statistics and
Economic Census is the
will be launched in other Program implementation
complete count of all
states and UT in August and (MoSPI)
establishments/units located
September.
MoSPI has partnered with within the geographical
Six Economic Censuses have Common Service Centres, boundaries of India.
been conducted till date: CSC e-Governance Service
Agriculture is not covered in
1977,1980,1990,1998,2005 India Ltd(SPV of Ministry of
Economic Census.
and 2013 Electronics & IT) as the
implementing agency for EC
The EC is the only source of
information on the
significantly large
unorganized sector in the
economy.

QUESTION 100. NjYxMjUrTW9oYW1tYWQgWW91bmlzIEFobWFkK3NhaGlsLmxhdy5qbWlAZ21haWwuY29tKz


ElPTiA5OQ==
Which of the following statements are correct about ‘Asset Reconstruction Companies’?

1. ARCs are established under SARFAECI Act and are regulated by RBI.

IASbaba
Score:
Web: http://ilp.iasbaba.com/
69.00 /
Email: ilp@iasbaba.com
Page 92 200
AIPTS/ILP VETERANS -
Exam Title :
2020 T...
Email : sahil.law.jmi@gmail.com
Contact : 9540210113

2. The aim of ARCs is to reconstruct Government structures and renovation of buildings of


Historical importance.

Select the code from below:

a) 1 only
b) 2 only
c) Both 1 and 2
d) Neither 1 nor 2
Correct Answer: A
Your Answer: A
Explanation

Solution (a)

The SARFAESI Act also provides for the establishment of Asset Reconstruction Companies
(ARCs) regulated by RBI to acquire assets from banks and financial institutions. The Act
provides for sale of financial assets by banks and financial institutions to asset reconstruction
companies (ARCs). RBI has issued guidelines to banks on the process to be followed for sales of
financial assets to ARCs.

Asset Reconstruction Companies (ARCs) have been created to bring about a system for
recovering Non Performing Assets (NPAs) from the books of secured lenders and unlocking the
value of Non-Performing Assets (NPA). Reserve Bank of India (RBI) provides license for ARCs
and ARCs are empowered by the SARFAESI Act.

For more details of ARCs you can go through the following link: https://www.indiafilings.com/
learn/asset-reconstruction-company-arc-india/

IASbaba
Score:
Web: http://ilp.iasbaba.com/
69.00 /
Email: ilp@iasbaba.com
Page 93 200
AIPTS/ILP VETERANS -
Exam Title :
2020 T...
Email : sahil.law.jmi@gmail.com
Contact : 9540210113

Review in Hindi
QUESTION 1.
नीचे��दए�गए�कथन��से�, सां�व�धक�तरलता�अनुपात�( SLR) क��उ�चत�प�रभाषा�क��पहचान�कर�:

a) यह��रज़व��का�एक�उपाय�है��जसम��वा�ण��यक�ब�क��को�सरकारी�बॉ�ड�, सोना�, और�समान�तरल�संप���के��प�म��रखना�आव�यक�है।�


b) यह��रज़व��का�एक�उपाय�है��जसे�ब�क��को�नकद��के��प�म��रखने�क��आव�यकता�होती�है।�
c) यह�अ�य�धक�तरल�संप���का�एक�उपाय�है��जसे�आसानी�से�नकद��म��प�रव�त�त��कया�जा�सकता�है��जसे�ब�क��को�रखना�आव�यक�है।�
d) यह��ाहक��के�कुल�जमा�का�एक��नधा��रत��यूनतम�अंश�है�, �जसे�वा�ण��यक�ब�क��को�या�तो�नकद��म��या�क���य�ब�क�के�पास�जमा�के
�प�म��रखना�होता�है।�
Correct Answer: A
Your Answer:
Explanation

Solution (a)

कथन��व�ेषण:

कथन�( a) कथन�( b) कथन�( c) कथन�( d)

स�य� अस�य� अस�य� अस�य�

नकद�आर��त
अनुपात�( CRR)
एसएलआर�, या�सां�व�धक एलसीआर�, या�तरलता�कवरेज �ाहक��क��कुल
तरलता�अनुपात�, �रज़व��का सीआरआर�, या�नकद�आर��त अनुपात�, अ�य�धक�तरल जमा�रा�श�का�एक
एक�उपाय�है�जो�वा�ण��यक अनुपात�, �रज़व��का�एक�उपाय संप���का�एक�उपाय�है��जसे �यूनतम��यूनतम
ब�क��को�सरकारी�बॉ�ड�, सोना है��जसे�ब�क��को�नकद��के��प आसानी�से�नकद��म��प�रव�त�त अंश�है�, �जसे
और�समान�तरल�संप���के म��रखने�क��आव�यकता�होती �कया�जा�सकता�है��जसे�ब�क� वा�ण��यक�ब�क�
�प�म��रखने�क��आव�यकता है।� को�रखने�क��आव�यकता�होती को�या�तो�नकद��म�
होती�है।� है।� या�क���य�ब�क�के
पास�जमा�के��प
म��रखना�होता�है।�

�या�आप�जानते�ह��?

• एसएलआर�, सीआरआर�और�एलसीआर�- तीन��ही�भारतीय��रज़व��ब�क�(आरबीआई) �ारा��योग��कए�जाने�वाले�नी�तगत�उपाय�ह��,


जो��कसी�भी�समय�ब�क���ारा�रखे�गए�कुल�भंडार�को��भा�वत�करते�ह�।�
• ये�आर��त�(�रज़व�) आव�यकताएं�उन�ऋण��क��मा�ा�को��भा�वत�करती�ह��जो�ब�क�उधारकता���तक�बढ़ा�सकते�ह�।�

QUESTION 2.
�या�होता�है�जब�RBI �रज़व��आव�यकता��को�कठोर�करता�है�?

1. ब�क�ऋण�दे ने�म��कटौती�करने�को�मजबूर�होते�ह��
2. अथ��व�था�म��मु�ा�क��आपू�त��म��कमी�होती�है�

IASbaba
Score:
Web: http://ilp.iasbaba.com/
69.00 /
Email: ilp@iasbaba.com
Page 94 200
AIPTS/ILP VETERANS -
Exam Title :
2020 T...
Email : sahil.law.jmi@gmail.com
Contact : 9540210113

3. अथ��व�था�म��मु�ा�क��आपू�त��बढ़�जाती�है�
4. मु�ा�फ��त�क��दर�घटने�क��उ�मीद�होती�है�

सही�उ�र�चुने:

a) केवल�2 और�4
b) 1, 2 और�4
c) केवल�3
d) केवल�3 और�4
Correct Answer: B
Your Answer:
Explanation

Solution (b)

जब�आरबीआई��रज़व��आव�यकता��को�कठोर�करता�है�, तो�ब�क��को�ऋण�दे ने�म��कटौती�करने�के��लए�मजबूर��कया�जाता�है�तथा�इसके


कारण�अथ��व�था�म��मु�ा�क��आपू�त��कम�हो�जाती�है।�

आरबीआई��ारा��रज़व��आव�यकता��को�कम��कए�जाने�पर�मु�ा�क��आपू�त��बढ़�जाती�है।�इससे�रा��क��मु�ा�आपू�त��बढ़ती�है�तथा
अथ��व�था�का��व�तार�होता�है।�ले�कन�बढ़ती��य�ग�त�व�ध�मु�ा�फ��त�को�बढ़ाने�का�भी�काय��कर�सकती�है।�

कथन��व�ेषण:

कथन�1, 2 और�4 कथन�3

स�य� अस�य�

जब�आरबीआई��रज़व��आव�यकता��को�कठोर�करता�है�,
तो�ब�क��को�ऋण�दे ने�म��कटौती�करने�के��लए�मजबूर��कया
जाता�है�और�इसके�कारण�अथ��व�था�म��मु�ा�क��आपू�त�
जब�RBI �रज़व��आव�यकता��को�कठोर�करता�है�, तो�अथ��व�था
कम�हो�जाती�है।�
म��मु�ा�क��आपू�त��नह��बढ़ती�है�, ब��क�यह��सकुड़�जाती�है।�
चूं�क�मु�ा�आपू�त��कम�हो�जाती�है�, तो�मु�ा�फ��त�क��दर
भी�कम�हो�जाती�है।�

QUESTION 3.
�न�न�ल�खत�म��से�कौन�“ �ाउ�ड�ग�आउट�" ( crowding out ) श�द�को�सही�ढं ग�से�प�रभा�षत�करता�है�?

a) यह�एक�आ�थ�क��स�ांत�है�जो�मानता�है��क�सामा�जक�सुर�ा�म��अ�धक�खच��आ�थ�क��वकास�को�बढ़ावा�दे ता�है।�
b) यह�एक��नवेश����या�है��जसके�तहत�सामा�य�जनता�को�बाहर�रखा�जाता�है।�
c) यह�एक�आ�थ�क��स�ांत�है�जो�मानता�है��क�साव�ज�नक��े��के��य�म��वृ����नजी��े��के��य�को�कम�करती�है�या�समा�त�करती�है।�
d) यह�एक�आ�थ�क��स�ांत�है�जो�सुझाव�दे ता�है��क�सरकारी�ऋण�वा�तव�म��रोजगार�पैदा�करके�मांग�को�बढ़ा�सकता�है�, �जससे��नजी��य
को�बढ़ावा��मलता�है।�
Correct Answer: C
Your Answer:

IASbaba
Score:
Web: http://ilp.iasbaba.com/
69.00 /
Email: ilp@iasbaba.com
Page 95 200
AIPTS/ILP VETERANS -
Exam Title :
2020 T...
Email : sahil.law.jmi@gmail.com
Contact : 9540210113

Explanation

Solution (c)

Explanation:

Crowding out effect ( �ाउ�ड�ग�आउट��भाव) Crowding in effect ( �ाउ�ड�ग�इन��भाव�)

�ाउ�ड�ग�आउट��भाव�एक�आ�थ�क��स�ांत�है�जो�यह�कहता
है��क�साव�ज�नक��े��का�बढ़ता��य��नजी��े��के��य�को
�सरी�ओर�, �ाउ�ड�ग�इन�, सरकार�का�ऋण�वा�तव�म��रोजगार
कम�करता�है�या�समा�त�करता�है।�
पैदा�करके�माँग�को�बढ़ा�सकता�है�, �जससे��नजी��य�म��वृ��
�ाउ�ड�ग�आउट��भाव�से�पता�चलता�है��क�साव�ज�नक��े� होगी।�
का��य��नजी��े��के��य�को�कम�करता�है।�

QUESTION 4.
�न�न�ल�खत�म��से�कौन�सा�कथन�" राजकोषीय��ो�साहन�" ( fiscal stimulus ) का�उ�चत��प�से�वण�न�करता�है�?

a) ती��आ�थ�क��वकास�के�कारण�मांग�म��वृ���को�पूरा�करने�के��लए�माल�क��आपू�त��सु�न��त�करने�हेतु��व�नमा�ण��े��म��सरकार��ारा�यह
एक�बड़ा��नवेश�है�
b) यह�दे श�म��आ�थ�क�ग�त�व�ध�को�बढ़ावा�दे ने�के��लए�सरकार�क��एक�गहन�सकारा�मक�कार�वाई�है�
c) कृ�ष�और�संब���े���के��लए�ऋण�का�सं�वतरण�सु�न��त�करने�तथा�अ�धक�खा��उ�पादन�को�बढ़ावा�दे ने�और�खा��मु�ा�फ��त�को
�नयं��त�करने�के��लए��व�ीय�सं�थान��पर�सरकार�क��यह�गहन�कार�वाई�है�
d) यह��व�ीय�समावेशन�क��अपनी�नी�त�को�आगे�बढ़ाने�के��लए�सरकार��ारा�क��गयी�एक�चरम�सकारा�मक�कार�वाई�है�
Correct Answer: B
Your Answer:
Explanation

Solution (b)

Approach:

' �ो�साहन�' ( stimulus ) उपाय��के�एक�पैकेज�के�मा�यम�से�नी�त��नमा�ता���ारा�सु�त�अथ��व�था�को��कक�टाट� �करने�का�एक��यास


है।�

मौ��क��ो�साहन�( Monetary stimulus ) राज�व��ो�साहन�( Fiscal stimulus )

IASbaba
Score:
Web: http://ilp.iasbaba.com/
69.00 /
Email: ilp@iasbaba.com
Page 96 200
AIPTS/ILP VETERANS -
Exam Title :
2020 T...
Email : sahil.law.jmi@gmail.com
Contact : 9540210113

एक�राजकोषीय��ो�साहन�सरकार��ारा�अपने��वयं�के�खजाने�से
अ�धक��य�करके�या�कर�दर��म��कमी�करके�उपभो�ा��के
एक�मौ��क��ो�साहन�म��क���य�ब�क��ारा�मु�ा�आपू�त��का हाथ��म��अ�धक�पैसा�प�ंचाया�जाता�है।�
�व�तार�करने�या�मु�ा�(�याज�दर�) को�कम�करके
उपभो�ा��य�को�बढ़ाया�जाता�है।� राजकोषीय��ो�साहन�आ�थ�क��वकास�को��ो�सा�हत�करने�और
समथ�न�करने�के��लए�एक�सरकार��ारा�क��गई�एक�सकारा�मक
कार�वाई�है।�

QUESTION 5.
�व�ीय�पय�वे�ण�बोड��( Board for Financial Supervision- BFS) के�संदभ��म���न�न�ल�खत�कथन��पर��वचार�कर��-

1. यह�एक��वशेष��वतं��एज�सी�है�जो�क���सरकार�क��आंत�रक�और�बाहरी�दे नदा�रय��को�सम���प�से��बं�धत�करती�है�तथा�शु�क�के
लाभ�जैसे�मामल��पर�सलाह�दे ती�है।�
2. इसे�सरकार�का��नवेश�ब�कर�या�मच�ट�ब�कर�भी�कहा�जाता�है।�

ऊपर��दए�गए�कथन��म��से�कौन�सा�सही�है�/ ह��?

a) केवल�1
b) केवल�2
c) दोन��1 और�2
d) न�तो�1 और�न�ही�2
Correct Answer: D
Your Answer:
Explanation

Solution (d)

कथन��व�ेषण:

कथन�1 कथन�2

अस�य� अस�य�

IASbaba
Score:
Web: http://ilp.iasbaba.com/
69.00 /
Email: ilp@iasbaba.com
Page 97 200
AIPTS/ILP VETERANS -
Exam Title :
2020 T...
Email : sahil.law.jmi@gmail.com
Contact : 9540210113

बोड��फॉर�फाइन��शयल�सुपर�वजन�( BFS), �जसका “ प��लक�डेट�मैनेजम�ट�एज�सी�” (PDMA) को�सरकार�का��नवेश


गठन�नवंबर�1994 म���कया�गया�था�, RBI के�तहत ब�कर�या�मच�ट�ब�कर�भी�कहा�जाता�है।�
एक��वाय���नकाय�के��प�म��है।�
बोड��फॉर�फाइन��शयल�सुपर�वजन�( BFS) दे श�म��मु�ा�बाजार
भारतीय��रज़व��ब�क�, बोड��फॉर�फाइन��शयल�सुपर�वजन सं�थान��क��दे खरेख�करता�है।�
(बीएफएस) के�माग�दश�न�म��पय�वे�ी�काय��करता�है।�
बीएफएस�का��ाथ�मक�उ�े �य�अनुसू�चत�वा�ण��यक�और�सहकारी
��ता�वत�" प��लक�डेट�मैनेजम�ट�एज�सी�" (PDMA) ए ब�क��, अ�खल�भारतीय��व�ीय�सं�थान��, �थानीय��े�ीय�ब�क��, लघु
क��वशेष��वतं��एज�सी�है�जो�सम���प�से�क���सरकार �व��ब�क��, भुगतान�ब�क��, �े�डट�सूचना�कंप�नय��, गैर-ब��क�ग��व�
क��आंत�रक�और�बाहरी�दे नदा�रय��का��बंधन�करती�है कंप�नय��और��ाथ�मक�डीलर�स�हत��व�ीय��े��क��समे�कत�पय�वे�ण
और�शु�क�के�लाभ�जैसे�मामल��पर�सलाह�दे ती�है।� करना�है।�

QUESTION 6.
�न�न�ल�खत�म��से�कौन�सा��रजव��ब�क�का�मूल�काय��है�?

1. ब�क�नोट��के��नग�मन�को��व�नय�मत�करना�
2. अपने�लाभ�के��लए�दे श�क��मु�ा�और�ऋण��णाली�को�संचा�लत�करना�
3. मू�य���थरता�बनाए�रखना�

उपयु��कूट�चुन�:

a) केवल�1
b) केवल�2
c) 1 और�2
d) 1, 2 और�3
Correct Answer: D
Your Answer:
Explanation

Solution (d)

Explanation:

भारतीय��रज़व��ब�क�क����तावना�म���रज़व��ब�क�के�मूल�काय��का�वण�न��कया�गया�है:

" भारत�म��मौ��क���थरता�को�सुर��त�रखने�के��लए�ब�क�नोट��के��रज़व��को�सुर��त�रखने�और�आम�तौर�पर�अपने�लाभ�के��लए�दे श�क��मु�ा


और�ऋण��णाली�को�संचा�लत�करने�के��लए�; एक�आधु�नक�मौ��क�नी�त�ढांचे�क��चुनौती�को�पूरा�करने�के��लए�; बढ़ती�ज�टल�अथ��व�था�,
�वकास�के�उ�े �य�को��यान�म��रखते��ए�मू�य���थरता�को�बनाए�रखना।�”

QUESTION 7.
�न�न�म��से�कौन�सा�कथन�गैर-अनुसू�चत�ब�क��के�संबंध�म��सही�है�/ ह��?

1. वे�आरबीआई��ारा�बनाए�गए��नयम��का�पालन�नह��करते�ह�।�
2. वे��दन-��त�दन�क��ग�त�व�धय��के��लए�RBI से�ऋण�लेने�के��लए�पा��नह��ह�।�
3. नकद�आर��त�अनुपात�( CRR) क��शत��उन�पर�लागू�नह��होती�ह�।�

उपयु��कूट�चुन�:

a) केवल�1

IASbaba
Score:
Web: http://ilp.iasbaba.com/
69.00 /
Email: ilp@iasbaba.com
Page 98 200
AIPTS/ILP VETERANS -
Exam Title :
2020 T...
Email : sahil.law.jmi@gmail.com
Contact : 9540210113

b) केवल�2
c) 1 और�2
d) 1, 2 और�3
Correct Answer: C
Your Answer:
Explanation

Solution (c)

कथन��व�ेषण:

कथन�1 कथन�2 कथन�3

स�य� स�य� अस�य�

गैर-अनुसू�चत�ब�क��को�सीआरआर�शत��का
गैर-अनुसू�चत�ब�क�भी��दन-��त�दन�क� पालन�करना�होगा।�
ग�त�व�धय��के��लए�RBI से�ऋण�लेने�के
गैर-अनुसू�चत�ब�क�RBI �ारा�बनाए
�लए�पा��नह��ह��, ले�कन�आपातकालीन हालां�क�, इन�ब�क��के�पास��वयं�के�साथ�CR
गए��नयम��का�पालन�नह��करते�ह�।�
प�र��थ�तय��म��RBI उ�ह��ऋण��दान�कर R फंड�हो�सकता�है��य��क�RBI �ारा�इसे�R
सकता�है।� BI म��जमा�करने�के��लए�कोई�बा�यता�नह��क�
गई�है।�

�या�आप�जानते�ह��?

अनुसू�चत�ब�क��और�गैर-अनुसू�चत�ब�क��के�बीच�मु�य�अंतर�ह��

1. अनुसू�चत�ब�क�RBI �ारा�बनाए�गए��नयम��का�पालन�करते�ह��जब�क�गैर-अनुसू�चत�ब�क�RBI �ारा�बनाए�गए��नयम��का�पालन�नह�


करते�ह�।�
2. अनुसू�चत�ब�क�भारतीय��रज़व��ब�क�अ�ध�नयम�, 1934 क���सरी�अनुसूची�म��शा�मल�करने�के��लए�पा��ह��, जब�क�गैर-अनुसू�चत
ब�क��सरी�अनुसूची�म��शा�मल�नह��ह�।�
3. अनुसू�चत�ब�क��को��नय�मत�ब��क�ग�उ�े �य��के��लए�RBI से�धन�उधार�लेने�क��अनुम�त�है�, जब�क�गैर-अनुसू�चत�ब�क��को�अनुम�त
नह��है।�
4. अनुसू�चत�ब�क�समाशोधन�गृह�( clearing house ) के�सद�य�बन�सकते�ह��जब�क�गैर-अनुसू�चत�ब�क�नह��बन�सकते।�
5. अनुसू�चत�ब�क��और�गैर-अनुसू�चत�ब�क��दोन��को�नकद�आर��त�अनुपात�बनाए�रखने�क��आव�यकता�है�, ले�कन�अनुसू�चत�ब�क��को
यह�रा�श�RBI के�पास�जमा�करनी�होगी�जब�क�गैर-अनुसू�चत�ब�क�इस�रा�श�को��वयं�के�पास�जमा�कर�सकते�ह�।�

QUESTION 8.
�ापा�रक�नेता��और�सरकारी�अ�धका�रय��के�म�य�पार�प�रक��प�से�लाभ�द�संबंध��को��न�न�के��प�म��जाना�जाता�है�-

a) कॉरपोरेट�लोकतं��( corporatocracy)
b) अंधरा�ीयता�( Jingoism )
c) भाई-भतीजावाद�( Nepotism)
d) �ोनी�कै�पट�ल�म�( Crony capitalism)

IASbaba
Score:
Web: http://ilp.iasbaba.com/
69.00 /
Email: ilp@iasbaba.com
Page 99 200
AIPTS/ILP VETERANS -
Exam Title :
2020 T...
Email : sahil.law.jmi@gmail.com
Contact : 9540210113

Correct Answer: D
Your Answer:
Explanation

Solution (d)

Approach:

कॉरपोरेटो�ेसी�( Corporatocracy ) एक�समान�श�द�लगता�है�, ले�कन�इस�त�य�म���भ�ता�है��क�यहां�बड़े��वसाय�और�इसके�नेता


सरकार��को��नयं��त�करते�ह�।�

�व�ेषण/ �ा�या:

कॉरपोरेटो�ेसी� अंधरा�ीयता� भाई-भतीजावाद� घोर�पूंजीवाद�

अंधरा�ीयता�(�ज�गोइज़म)
आ�ामक��वदे श�नी�त�के��प�म�
रा�वाद�है�, जैसे��क�एक�दे श�के
�ोनी�कै�पट�ल�म�एक�ऐसी
कॉरपोरेटो�ेसी�एक�श�द�है �ारा�खतर��या�वा�त�वक�बल�के
नेपो�ट�म��वसाय�, राजनी�त अथ��व�था�है��जसम���वसाय
�जसका�उपयोग�कॉप�रेट�या उपयोग�, शां�तपूण��संबंध��के
राजनी�त�, मनोरंजन�, खेल�, जो�खम�के�प�रणाम�व�प�नह��,
कॉप�रेट��हत���ारा��नयं��त �वपरीत�, जो�इसे�अपने�रा�ीय
धम��और�अ�य�ग�त�व�धय� ब��क�एक��ापारी�वग��और
एक�आ�थ�क�और�राजनी�तक �हत��के��प�म��मानता�है।�
स�हत��व�भ���े���म���कसी�के राजनी�तक�वग��के�बीच�सांठगांठ
�णाली�को�संद�भ�त�करता�है।� �र�तेदार��को�रोज़गार�दे ने�से�है। के�मा�यम�से�धन�के�लाभ�के��प
�ज�गोइज़म�क��प�रभाषा�चरम
और�आ�ामक�दे शभ���है�, �ज म��बढ़ता�है।�
सके�प�रणाम�व�प�आ�ामक
�वदे श�नी�त�है।�

QUESTION 9.
�न�न�ल�खत�म��से�कौन�सा�/ से�ऋण��नयं�ण�( Credit Control ) का�मा�ा�मक�उपाय�( Quantitative measure ) है�?

1. नै�तक�दबाव�( Moral Suasion )


2. ब�क�दर�नी�त�( Bank rate policy )
3. ऋण�का�यु��करण�( Rationing of credit )
4. ओपन�माक�ट�आपरेशन�( Open Market Operations )

उपयु��कूट�चुन�:

a) केवल�2
b) 2 और�4
c) 2, 3 और�4
d) 1, 2 और�4
Correct Answer: B
Your Answer:
Explanation

Solution (b)

IASbaba
Score:
Web: http://ilp.iasbaba.com/
69.00 /
Email: ilp@iasbaba.com
Page 100 200
AIPTS/ILP VETERANS -
Exam Title :
2020 T...
Email : sahil.law.jmi@gmail.com
Contact : 9540210113

Approach/Elimination:

य�द�आप��े�डट�(ऋण) को��नयं��त�करने�के��लए�RBI �ारा��नयो�जत��व�भ��तरीक��से�अवगत�ह��, तो�आप�इस����का�उ�र�आसानी�से�दे


सकते�ह�।�

आरबीआई��ारा�उधार�क��मा�ा�को��नयं��त�करने�के��लए�दो�मह�वपूण��तरीके�ह��- मा�ा�मक��नयं�ण�और�गुणा�मक��नयं�ण।�

• मा�ा�मक��नयं�ण��को�ब��क�ग��णाली��ारा�सृ�जत�क��गयी�ऋण�क��मा�ा�को��व�नय�मत�करने�के��लए��डज़ाइन��कया�गया�है।��व�श�
उपयोग��म���े�डट�के��वाह�को��व�नय�मत�करने�के��लए�गुणा�मक�उपाय��या�चयना�मक�तरीक��को��डज़ाइन��कया�गया�है।�
• �े�डट��नयं�ण�क��मा�ा�मक�या�पारंप�रक��व�धय��म��ब�क�दर�नी�त�, खुले�बाजार�संचालन�और�प�रवत�नीय�( variable ) आर��त
अनुपात�शा�मल�ह�।��े�डट��नयं�ण�के�गुणा�मक�या�चयना�मक�तरीक��म��मा�ज�न�आव�यकता�, �े�डट�राश�न�ग�, नै�तक�दबाव�( sua
sion) ( �े�डट�मॉ�नट�र�ग��व�था) , उपभो�ा��े�डट�के��व�नयमन�और���य��कार�वाई�के��व�नयमन�शा�मल�ह�।�

कथन��व�ेषण:

कथन�1 और�3 कथन�2 और�4

अस�य� स�य�

गुणा�मक�उपाय��म��शा�मल�ह��-
मा�ा�मक��नयं�ण�म��शा�मल�ह��-
1. मा�ज�न�आव�यकता��म��प�रवत�न�
1. ब�क�दर�नी�त�
2. ऋण��का�यु��करण�
2. खुला�बाजार�प�रचालन�
3. उपभो�ा�ऋण�का��व�नयमन�
3. प�रवत�नीय��रजव��अनुपात�
4. नै�तक�दबाव�और�ऋण��नगरानी��व�था�

QUESTION 10.
�न�न�ल�खत�म��से�कौन�सा�श�द�" अवसर�लागत�" ( opportunity cost ) को�सही�ढं ग�से�प�रभा�षत�करता�है�?

a) यह�अवसर�क��खोज�म���य�क��गई�लागत�या�रा�श�है।�
b) यह�उन�लाभ��का���त�न�ध�व�करता�है�जो�एक�����, �नवेशक�या��वसाय�खो�दे ते�ह��जब�एक��वक�प�पर��सरे��वक�प�को�चुनते�ह�।�
c) यह�वह�रा�श�है�जो�वत�मान�क��संप���, भ�व�य�क��लागत�पर�होगी।�
d) वे��न��त�होते�ह��, जो�माल�के�उ�पादन�म��या�सेवा��के���तपादन�म��उपयोग��कए�गए�भू�म�, भवन�, �नमा�ण�और�उपकरण��क��खरीद
पर�एक�बार�का��य�होता�है।�
Correct Answer: B
Your Answer:
Explanation

Solution (b)

Explanation:

अवसर�लागत�( Opportunity costs ) उन�लाभ��का���त�न�ध�व�करती�है�जो�एक�����, �नवेशक�या��वसाय�खो�दे ते�ह��जब�एक


�वक�प�पर��सरे�को�चुनते�ह�।�

IASbaba
Score:
Web: http://ilp.iasbaba.com/
69.00 /
Email: ilp@iasbaba.com
Page 101 200
AIPTS/ILP VETERANS -
Exam Title :
2020 T...
Email : sahil.law.jmi@gmail.com
Contact : 9540210113

�कसी�व�तु�का�लाभ�, मुनाफ़ा�या�मू�य�, जो��कसी�अ�य�व�तु�को��ा�त�करने�या��ा�त�करने�के��लए��दया�जाना�चा�हए।�

�या�आप�जानते�ह��?

• अवसर�लागत�अथ�शा���म��मूलभूत�लागत�होती�है�, तथा�इसका�उपयोग��कसी�प�रयोजना�क��लागत�लाभ��व�ेषण�के��लए��कया
जाता�है।�
• इस�तरह�क��लागत�, हालां�क�, खाता-बही�म��दज��नह��क��जाती�है�, ले�कन�नकद��क��गणना�और�उनके�प�रणाम�व�प�लाभ�या�हा�न
क��गणना�करके��नण�य�लेने�म��मा�यता��ा�त�है।�

QUESTION 11.
राजकोषीय�उ�रदा�य�व�और�बजट��बंधन�( FRBM) अ�ध�नयम�का�उ�े �य�है�

1. राज�व�घाटा�समा�त�करना�तथा�राजकोषीय�घाटा�कम�करना�
2. वष��दर�वष��दे श�के�ऋण��का�अ�धक��यायसंगत�और��बंधनीय��वतरण���तुत�करना�

ऊपर��दए�गए�कथन��म��से�कौन�सा�सही�है�/ ह��?

a) केवल�1
b) केवल�2
c) दोन��1 और�2
d) न�तो�1 और�न�ही�2
Correct Answer: C
Your Answer:
Explanation

Solution (c)

Explanation:

एफआरबीएम�अ�ध�नयम�का�उ�े �य��व�ीय��बंधन�म���ववेक�( prudence ) को�सु�न��त�करना�था�-

• राज�व�घाटे �को�ख�म�करना�,
• राजकोषीय�घाटा�कम�करना�,
• ऋण��बंधन�म��सुधार��था�पत�करना�और�
• म�यम�अव�ध�के�ढांचे�क��पारद�श�ता�म��सुधार�
• रा�य��ारा�घाटे �के�उपाय��और�ऋण��बंधन�के�संबंध�म���नधा��रत�मा�ा�मक�ल�य��का�पालन�करना।�

अ�ध�नयम�म��भारतीय��रजव��ब�क�( RBI) को�भारत�म��मु�ा�फ��त�के��बंधन�के��लए�आव�यक�लचीलापन�दे ने�क��भी�अपे�ा�क��गई�थी।�

कथन��व�ेषण:

कथन�1 कथन�2

स�य� स�य�

IASbaba
Score:
Web: http://ilp.iasbaba.com/
69.00 /
Email: ilp@iasbaba.com
Page 102 200
AIPTS/ILP VETERANS -
Exam Title :
2020 T...
Email : sahil.law.jmi@gmail.com
Contact : 9540210113

एफआरबीएम�अ�ध�नयम�राज�व�घाटे �को�समा�त�करता एफआरबीएम�अ�ध�नयम�वष��दर�वष��दे श�के�ऋण��के�अ�धक


है�और�राजकोषीय�घाटे �को�कम�करता�है� �यायसंगत�और��बंधनीय��वतरण�पर�ज़ोर�दे ता�है�

QUESTION 12.
�न�न�ल�खत�कथन��पर��वचार�कर�:

1. सरकार��नजी��े��क��पसंद�के�उ�पा�दत�व�तु��और�सेवा��को�सु�वधा��दान�करती�है।�
2. मू�य�तं��को�आमतौर�पर�क�याणकारी�कारण��के�कारण�अनुम�त�नह��द��जाती�है।�

ऊपर��दए�गए�कथन��म��से�कौन�' बाजार�अथ��व�था�' के�बारे�म��सही�है�/ है�?

a) केवल�1
b) केवल�2
c) दोन��1 और�2
d) न�तो�1 और�न�ही�2
Correct Answer: D
Your Answer:
Explanation

Solution (d)

कथन��व�ेषण:

कथन�1 कथन�2

अस�य� अस�य�

एक�बाजार�अथ��व�था�म��, यह��नजी��े��है�, जो�तय बाजार�क��श���(यानी�मांग�और�आपू�त�) व�तु��और�सेवा��क�


करता�है��क��या�उ�पादन�करना�है।� क�मत��को�तय�करती�है।�

आ�थ�क��णा�लय��के��कार�

• एक�बाजार�अथ��व�था�म��, �जसे�पूंजीवाद�भी�कहा�जाता�है�, केवल�उन�उपभो�ा�व�तु��का�उ�पादन��कया�जाता�है�, जो�मांग�म��ह�


, अथा�त�, ऐसे�सामान��ज�ह��या�तो�घरेलू�या��वदे शी�बाजार��म��लाभ�द��प�से�बेचा�जा�सकता�है।�
• एक�पूंजीवाद��समाज�म��उ�पा�दत�व�तु��को�लोग��के�बीच��वत�रत��कया�जाता�है�जो��क�लोग��क��आव�यकता�के�आधार�पर�नह��,
ब��क�इस�आधार�पर�होता�है��क�लोग��या�खरीद�सकते�ह��और�खरीदने�के�इ�छु क�ह�।�
• एक�समाजवाद��समाज�म��सरकार�यह�तय�करती�है��क�समाज�क��ज़�रत��के�मुता�बक��या�सामान�तैयार��कया�जाना�है।�यह�माना
जाता�है��क�सरकार�को�पता�है��क�दे श�के�लोग��के��लए��या�अ�छा�है�और�इस�लए����गत�उपभो�ा��क��इ�छा��को��यादा
मह�व�नह���दया�जाता�है।�सरकार�तय�करती�है��क�माल�का�उ�पादन�कैसे��कया�जाए�और�उ�ह��कैसे��वत�रत��कया�जाए।��स�ांत��प
म��, समाजवाद�के�तहत��वतरण�के�आधार�पर�माना�जाता�है��क�लोग��को��या�ज�रत�है�और��या�वे�खरीद�नह��सकते�ह��, पर
आधा�रत�है।�कठोरता�से�, एक�समाजवाद��समाज�के�पास�कोई��नजी�संप���नह��है��य��क�सब�कुछ�रा�य�के��वा�म�व�म��होता�है।�
• अ�धकांश�अथ��व�था��म��त�अथ��व�थाएं�ह��, अथा�त�, सरकार�और�बाजार��मलकर�तीन���ो�के�उ�र�दे ते�ह���क��या�उ�पादन
करना�है�, कैसे�उ�पादन�करना�है�और�कैसे��वतरण�करना�है।�एक��म��त�अथ��व�था�म��, बाजार�जो�भी�व�तु�और�सेवाएं��दान�कर

IASbaba
Score:
Web: http://ilp.iasbaba.com/
69.00 /
Email: ilp@iasbaba.com
Page 103 200
AIPTS/ILP VETERANS -
Exam Title :
2020 T...
Email : sahil.law.jmi@gmail.com
Contact : 9540210113

सकता�है�वह�अ�छ��तरह�से�उ�पादन�कर�सकता�है�, और�सरकार�आव�यक�व�तु�और�सेवाएं��दान�करेगी�जो�बाजार��दान�करने�म�
�वफल�रहता�है।�

QUESTION 13.
�न�न�ल�खत�कथन��पर��वचार�कर��तथा�तरीके�और�साधन�अ��म�( Ways and Means Advances- WMA) क��उपयु��प�रभाषा
क��पहचान�कर��

a) यह�वह�तं��है��जसके�मा�यम�से�RBI, क���और�रा�य�सरकार��को�अ�थायी�ऋण�सु�वधाएं��दान�करता�है।�
b) यह�वह�तं��है��जसके�मा�यम�से�रा�ीय�ब�क�, आरबीआई�से�अ�थायी�ऋण�सु�वधाएं�लेते�ह�।�
c) यह�RBI �ारा�सरकारी���तभू�त�( g-sec) क���ब���या�खरीद�के�मा�यम�से�धन�आपू�त��क����थ�त�को�समायो�जत�करने�के��लए��कया
जाता�है।�
d) यह�वह�तं��है��जसके�मा�यम�से�RBI वा�ण��यक�ब�क��से�अथ��व�था�के�चलन�के�अनुसार�कुछ�उपाय�करने�का�अनुरोध�करता�है।�
Correct Answer: A
Your Answer:
Explanation

Solution (a)

Explanation:

तरीके�और�साधन�अ��म�( Ways and Means Advances )

भारतीय��रज़व��ब�क�, क���और�रा�य�सरकार��को�सरकार�के�एक�ब�कर�के��प�म��अ�थायी�ऋण�सु�वधाएं�दे ता�है।�इस�अ�थायी�ऋण�सु�वधा�को


तरीके�और�साधन�अ��म�( WMA) कहा�जाता�है।�

क���सरकार�के��लए�WMA योजना�को�क���सरकार�के�घाटे �का��व�पोषण�करने�के��लए�एडहॉक�(अ�थायी) �े जरी��बल��क��चार�दशक


पुरानी��णाली�को�समा�त�करने�के�बाद�1 अ�ैल�, 1997 को�आरंभ��कया�गया�था।�ड��यूएमए�योजना�को�सरकार�क���ा��तय��और�भुगतान
म��अ�थायी�असंतुलन�को�पूरा�करने�के��लए��डज़ाइन��कया�गया�था।�य�द�आरबीआई�से�त�काल�नकद��क��ज�रत�है�तो�सरकार��ारा�इस
सु�वधा�का�लाभ�उठाया�जा�सकता�है।�ड��यूएमए�क��सीमाएँ�भारतीय��रज़व��ब�क�और�भारत�सरकार��ारा�पार�प�रक��प�से�तय�क��जाती�ह�।�

QUESTION 14.
वह�दर��जस�पर�अनुसू�चत�ब�क�, सरकारी���तभू�तय��के��व���RBI से�रात�के��लए�धनरा�श�उधार�ले�सकते�ह��(आपातकालीन���थ�त�म��जब
अंतर-ब�क�तरलता�पूरी�तरह�से�सूख�जाती�है) कहा�जाता�है�-

a) पूंजी�पया��तता�अनुपात�( Capital Adequacy Ratio )


b) नकद���वाह�पया��तता�सु�वधा�( Cash flow adequacy facility )
c) सीमांत��था�य�व�सु�वधा�( Marginal standing facility )
d) तरलता�समायोजन�सु�वधा�( Liquidity adjustment facility )
Correct Answer: C
Your Answer:
Explanation

Solution (c)

Approach:

सीमांत��थायी�सु�वधा�( Marginal Standing Facility ) वह�दर�है��जस�पर�अनुसू�चत�ब�क�सरकारी���तभू�तय��के��व���भारतीय


�रजव��ब�क�( RBI) से�रात�भर�के��लए�धनरा�श�उधार�ले�सकते�ह�।�

IASbaba
Score:
Web: http://ilp.iasbaba.com/
69.00 /
Email: ilp@iasbaba.com
Page 104 200
AIPTS/ILP VETERANS -
Exam Title :
2020 T...
Email : sahil.law.jmi@gmail.com
Contact : 9540210113

तरलता�समायोजन�सु�वधा� सीमांत��थायी�सु�वधा�

• �यूनतम�बोली�रा�श�1 करोड़�है।�
• �यूनतम�बोली�रा�श�5 करोड़�है।� • केवल�अनुसू�चत�वा�ण��यक�ब�क�ही�बोली�लगा�सकते
• RBI के�सभी��ाहक�बोली�लगाने�के��लए�पा��ह�। ह�।�
• ब�क�RBI को�सरकारी���तभू�त�नह��बेच�सकते�ह� • ब�क�अपने�SLR कोटा�से�RBI को�सरकारी���तभू�त
जो�ब�क�के�SLR कोटा�का��ह�सा�है।� का��व�य�सकते�ह�।�
• एलएएफ�उधार�दर�हमेशा�रेपो�दर�होती�है।� • MSF क��दर�हमेशा�रेपो�दर�से�अ�धक�होती�है��य��क
ब�क��को�तुर�त�धन�क��आव�यकता�होती�है।�

�या�आप�जानते�ह��?

• सीमांत��थायी�सु�वधा�क��दर�वत�मान�म��6 जून�2019 को�25 आधार�अंक��क���गरावट�के�बाद�6.00% पर�है।�

QUESTION 15.
नकद�आर��त�अनुपात�( CRR) के�बारे�म���न�न�ल�खत�कथन��पर��वचार�कर�।�

1. यह�वह�रा�श�है�जो��े�ीय��ामीण�ब�क��( RRB) को�छोड़कर�सभी�अनुसू�चत�वा�ण��यक�ब�क��( SCB) को�अपनी�कुल��नवल�मांग


और�समय�दे यता��( DTL) के�संदभ��म��आरबीआई�के�साथ��कसी�भी�आधार�या�सीमा�क��दर�के��बना�बनाए�रखना�आव�यक�है।�
2. जब�आरबीआई�कम�नकद�आर��त�अनुपात�क��घोषणा�करता�है�, तो�इसका�अथ��है��क�वा�ण��यक�ब�क��के�पास�उधार�दे ने�के��लए
अ�धक�पैसा�है।�

ऊपर��दए�गए�कथन��म��से�कौन�सा�सही�है�/ ह��?

a) केवल�1
b) केवल�2
c) दोन��1 और�2
d) न�तो�1 और�न�ही�2
Correct Answer: C
Your Answer:
Explanation

Solution (c)

Explanation:

नकद�आर��त�अनुपात�(सीआरआर) वह�रा�श�है�जो��े�ीय��ामीण�ब�क��(आरआरबी) को�छोड़कर�सभी�अनुसू�चत�वा�ण��यक�ब�क�


(एससीबी) को�अपने�कुल��नवल�मांग�और�समय�दे यता��(डीट�एल) के�संदभ��म��आरबीआई�के�साथ��कसी�भी�आधार�या�सीमा�क��दर�के
�बना�बनाए�रखना�आव�यक�है।) ब�क��क��तरलता�और�सॉ�व�सी�सु�न��त�करने�के��लए।�

RBI अ�ध�नयम�1934 क��धारा�42 (1) के�संदभ��म��, अनुसू�चत�वा�ण��यक�ब�क��को�RBI के�पास�एक�औसत�नकद��शेष�को�बनाए


रखना�आव�यक�है�, �जसक��रा�श�भारत�म��कुल�शु��माँग�और�समय�दे यताएँ�( NDTL) के�3% से�कम�नह��होगी�और�उ�च�दर�कुल�के�20
% से�अ�धक�नह��होगी।�

कम�सीआरआर�का�अथ��है��क�ब�क�ऋण�के��प�म��अ�धक�पैसा�दे �सकते�ह��= कम��याज�दर�= स�ता�ऋण�= अ�धक�लोग��वसाय�शु�


करने�या�घर�बनाने�या�कार�खरीदने�के��लए�ऋण�लेते�ह��= अथ��व�था�म��वृ��।�हालां�क�, मु�ा�फ��त�को�भी�ज�म�दे �सकता�है�, अगर�लोग�
के�पास�बाजार�म��खरीद�के��लए�उपल�ध�व�तु��क��तुलना�म��अ�धक�नकद��है।�

IASbaba
Score:
Web: http://ilp.iasbaba.com/
69.00 /
Email: ilp@iasbaba.com
Page 105 200
AIPTS/ILP VETERANS -
Exam Title :
2020 T...
Email : sahil.law.jmi@gmail.com
Contact : 9540210113

कथन��व�ेषण:

कथन�1 कथन�2

स�य� स�य�

नकद�आर��त�अनुपात�( CRR) वह�रा�श�है�जो��े�ीय


�ामीण�ब�क��( RRB) को�छोड़कर�सभी�अनुसू�चत
जब�आरबीआई�कम�नकद�आर��त�अनुपात�क��घोषणा�करता�है�,
वा�ण��यक�ब�क��( SCB) को�अपने�कुल�शु��मांग�और
तो�इसका�अथ��है��क�वा�ण��यक�ब�क��के�पास�उधार�दे ने�के��लए
समय�दे यता��( DTL) के�संदभ��म��RBI के�साथ��बना
अ�धक�पैसा�है।�
�कसी�भी�आधार�या�सीमा�क��दर�के��बना�बनाए�रखना
आव�यक�है।�

�या�आप�जानते�ह��?

• मांग�दे यताएं�, वे�दे यताएं�ह��जो�मांग�पर�दे य�ह��तथा�समय�दे यताएं�वे�ह��जो�मांग�के�समय�पर�दे य�ह�।�

QUESTION 16.
�न�न�ल�खत�म��से�कौन�पूंजी�खाते�( Capital Account ) के�अंतग�त�आता�है�?

1. �वदे शी�ऋण�
2. ��य���वदे शी��नवेश�
3. �नजी��ेषण�( Private Remittances )
4. पोट� फो�लयो��नवेश�

नीचे��दए�गए�कूट�का�उपयोग�करके�सही�उ�र�चुन�:

a) 1, 2 और�3
b) 1, 2 और�4
c) 2, 3 और�4
d) 1, 3 और�4
Correct Answer: B
Your Answer:
Explanation

Solution (b)

Explanation:

पूंजीगत�खाते�म���वदे शी�ऋण�, �वदे शी���य���नवेश�और�पोट� फो�लयो��नवेश�शा�मल�ह�।�इसम���ेषण�( remittances ) शा�मल�नह��ह�।�

पूंजीगत�खाता��कसी�दे श�म��और�उसके�बाहर�होने�वाले�साव�ज�नक�और��नजी�अंतररा�ीय��नवेश��का�शु��प�रणाम�है।�पोट� फो�लयो��नवेश


शेयर��और�बांड��क��खरीद�है।�एफडीआई��कसी�दे श�म���वदे �शय��या�बाहरी�दे श��म���नवेश�करने�वाले�नाग�रक���ारा��कया�जाने�वाला��नवेश�है।�

IASbaba
Score:
Web: http://ilp.iasbaba.com/
69.00 /
Email: ilp@iasbaba.com
Page 106 200
AIPTS/ILP VETERANS -
Exam Title :
2020 T...
Email : sahil.law.jmi@gmail.com
Contact : 9540210113

भुगतान�संतुलन�के�तीन�मु�य�घटक�( BOP):

1. चालू�खाता�
2. पूंजी�खाता�
3. आ�धका�रक��रजव��लेनदे न�( Official Reserve Transactions )

चालू�खाते�म��व�तु�और�सेवा��के��नया�त�और�आयात�, �नवेश�आय�, और�एकतरफा��थाना�तरण�(�ेषण�, उपहार�, अनुदान�आ�द) से


संबं�धत�सभी�लेनदे न�शा�मल�ह�।�पूंजी�खाते�म��सभी�अंतररा�ीय�प�रसंप���लेनदे न�(एफडीआई�, एफपीआई�आ�द) शा�मल�ह�।�जब�भी�BoP
घाटा�या�BoP अ�धशेष�होता�है�, तो�RBI जैसे�क���य�ब�क���ारा�आ�धका�रक��रज़व��लेनदे न��कए�जाते�ह�।�ये�लेनदे न�अंतररा�ीय�आर��त
प�रसंप��य��, जैसे��क�सोने�और��मुख�अंतरा��ीय�मु�ा��के��प�म���कए�जाते�ह�।�

QUESTION 17.
�न�न�ल�खत�म��से�कौन��ाथ�मकता��े��के�अंतग�त��व�भ���े�णय��का��ह�सा�ह��?

1. �श�ा�
2. सू�म�, लघु�और�म�यम�उ�म�
3. सामा�जक�अवसंरचना�
4. आवास�
5. नवीकरणीय�ऊजा��

उपयु��कूट�चुन�:

a) 2, 3 और�4
b) 2, 3 और�5
c) 1, 2 और�3
d) 1, 2, 3, 4 और�5
Correct Answer: D
Your Answer:
Explanation

Solution (d)

Approach:

पीएसएल�( PSL) वा�ण��यक�ब�क��के��लए�आरबीआई��ारा��नधा��रत�मानदं ड�ह��, �जसम��ब�क��को��व�ीय�वष��म��उनके��व�ीय�ऋण�का�40


% कुछ��वशेष��े���म��उधार�दे ने�के��लए�माना�जाता�है।�नीचे��दए�गए��े���को�RBI �ारा��ाथ�मकता�वाले��े���के��प�म��अ�नवाय���कया�गया
है।�

�ाथ�मकता��े��म���न�न�ल�खत��े�णयां�शा�मल�ह�:

IASbaba
Score:
Web: http://ilp.iasbaba.com/
69.00 /
Email: ilp@iasbaba.com
Page 107 200
AIPTS/ILP VETERANS -
Exam Title :
2020 T...
Email : sahil.law.jmi@gmail.com
Contact : 9540210113

1. कृ�ष�
2. सू�म�, लघु�और�म�यम�उ�म�
3. �नया�त�ऋण�
4. �श�ा�
5. आवास�
6. सामा�जक�अवसंरचना�
7. नवीकरणीय�ऊजा��
8. अ�य�

QUESTION 18.
�न�न�म��से�कौन�सा�कथन�‘ गो�डीलॉ�स�अथ��व�था�’ श�द�का�सबसे�अ�छा�वण�न�करता�है�?

a) एक�अथ��व�था�जो�इतनी�त�त�नह��है��क�यह�मु�ा�फ��त�का�कारण�बनती�है�, तथा�इतनी�ठं डी�नह���क�यह�मंद��का�कारण�बनती�है�,


जो�बाजार�के�अनुकूल�मौ��क�नी�त�क��अनुम�त�दे ती�है।�
b) एक�अथ��व�था�जो�मु�ा�फ��त�के�साथ�ब�त�त�त�है�तथा�बाजार�के�अनुकूल�मौ��क�नी�त��ारा�संचा�लत�है।�
c) एक�अथ��व�था�जो�त�त�और�ठं डी�के�म�य�म��होती�है�, �सरे�श�द��म��, म�यम�आ�थ�क��वकास�को�बनाए�नह��रखती�है�और��जसम��उ�च
मु�ा�फ��त�होती�है।�
d) ऐसी�अथ��व�था�जो��ो�साहन�उपाय��का�समथ�न�करने�के��लए�न�तो�पया��त��प�से�ठं डी�है�, न�ही��वकास�को�बनाए�रखने�के��लए
पया��त��प�से�त�त�है।�
Correct Answer: A
Your Answer:
Explanation

Solution (a)

Explanation:

गो�डीलॉ�स�अथ��व�था�एक�ऐसी�अथ��व�था�है�जो�ब�त�त�त�या�ठं डा�नह��है�, �सरे�श�द��म��, म�यम�आ�थ�क��वकास�को�बढ़ावा�दे ता�है�,


और�इसम��कम�मु�ा�फ��त�है�, जो�बाजार�के�अनुकूल�मौ��क�नी�त�क��अनुम�त�दे ती�है।�

गो�डीलॉ�स�अथ��व�था�एक�ऐसी�अथ��व�था�का�वण�न�करती�है�जो�इतनी�त�त�नह��है��क�यह�मु�ा�फ��त�का�कारण�बनती�है�, और�इतनी
ठं डी�नह���क�यह�मंद��का�कारण�बनती�है।�यह�श�द�एक�अथ��व�था�का�वण�न�करता�है�जो�पूण��रोजगार�और�आ�थ�क���थरता��दान�करके
एक�इ�तम���थ�त�म��चल�रही�है।�

हालां�क�, एक�एंट�-गो�डीलॉ�स�अथ��व�था�वह�है�जो�न�तो��ो�साहन�उपाय��का�समथ�न�करने�के��लए�पया��त�ठं डी�है�, और�न�ही��वकास�को


बनाए�रखने�के��लए�पया��त��प�से�त�त�है।�

�या�गो�डीलॉ�स�अथ��व�था�एक�वा�त�वकता�है�?

• अथ��व�था�क��ऐसी���थ�त��रकवरी�चरण��के�दौरान�हो�सकती�है।�उदाहरण�के��लए�, 90 के�दशक�के�उ�रा�� �क��अमे�रक�


अथ��व�था�को�गो�डीलॉ�स�अथ��व�था�माना�जाता�था��य��क�यह�" ब�त�त�त�नह��थी�, ब�त�ठं डी�भी�नह��था�, ले�कन�केवल
सही�थी�" ।��य��क�हमारे�पास��ापार�च��ह��, एक�गो�डीलॉ�स�अथ��व�था�को�एक�अ�थायी�अव�था�माना�जाना�चा�हए।�

QUESTION 19.
�न�न�ल�खत�म��से��कस�उपाय�से�अथ��व�था�म��मु�ा�क��आपू�त��म��वृ���होगी�?

1. क���य�ब�क��ारा�जनता�से�सरकारी���तभू�तय��क��खरीद�
2. वा�ण��यक�ब�क��म��जनता��ारा�मु�ा�जमा�करना�
3. सरकार��ारा�क���य�ब�क�से�उधार�लेना�

IASbaba
Score:
Web: http://ilp.iasbaba.com/
69.00 /
Email: ilp@iasbaba.com
Page 108 200
AIPTS/ILP VETERANS -
Exam Title :
2020 T...
Email : sahil.law.jmi@gmail.com
Contact : 9540210113

नीचे��दए�गए�कूट�का�उपयोग�करके�सही�उ�र�चुन�:

a) केवल�1
b) केवल�1 और�2
c) केवल�1 और�3
d) 1, 2 और�3
Correct Answer: C
Your Answer:
Explanation

Solution (c)

कथन��व�ेषण:

कृपया��यान�द� ��क�RBI से��नकलने�वाला�कोई�भी�पैसा�मु�ा�आपू�त��म��वृ���करता�है।�

कथन�1 कथन�2 कथन�3

स�य� अस�य� स�य�

जब�सरकार�RBI से�उधार�लेती�है�, तो�मु�ा


जब�मु�ा�वा�ण��यक�ब�क��म��जनता��ारा
जब�RBI सरकारी���तभू�तय��को �फर�से�RBI से�बाहर�आ�जाती�है।�
जमा�क��जाती�है�, तो�इसका�केवल�जनता
जनता�से�खरीदता�है�, तो�पैसा�RBI से से�वा�ण��यक�ब�क��म��धन�ह�तांतरण�होता सरकार�क��मांग�को�पूरा�करने�के��लए�,
�नग�त�होता�है�, इससे�मु�ा�क��आपू�त� है।� RBI नए�पैसे�छापेगा�और�इससे�मु�ा
बढ़�जाएगी।�
आपू�त��म��वृ���होगी�और�इस��कार�कथन�
मु�ा�आपू�त��पर�शु���भाव�शू�य�होता�है।�
3 भी�सही�है।�

QUESTION 20.
�न�न�ल�खत�कथन��पर��वचार�कर�:

1. एक��वशेष�समय�म��एक�अथ��व�था�के�पास�सभी��वदे शी�मु�ा��का�कुल�योग�, इसक���वदे शी�मु�ा�संप���/ भंडार�है।�


2. �वदे शी�मु�ा�भंडार�आ�थ�क�मंद��के�मामले�म��भारत�के��लए�र�ा�क��पहली�पं���के��प�म��काय��करता�है।�
3. �वदे शी�मु�ा�भंडार�भारत�सरकार�के��लए�भारतीय��रजव��ब�क��ारा��बं�धत��कया�जाता�है।�

उपरो��कथन��म��से�कौन�सा�सही�है�/ ह��?

a) 1 और�2
b) 1 और�3
c) 2 और�3
d) 1, 2 और�3
Correct Answer: D
Your Answer:
Explanation

IASbaba
Score:
Web: http://ilp.iasbaba.com/
69.00 /
Email: ilp@iasbaba.com
Page 109 200
AIPTS/ILP VETERANS -
Exam Title :
2020 T...
Email : sahil.law.jmi@gmail.com
Contact : 9540210113

Solution (d)

Explanation:

�वदे शी�मु�ा�संप���

• एक��वशेष�समय�म��एक�अथ��व�था�के�पास�सभी��वदे शी�मु�ा��का�कुल�योग�इसक���वदे शी�मु�ा�संप���/ भंडार�है।�


• तरल��वदे शी�मु�ा�का��बंधन�करने�के��लए�एक�अथ��व�था�क��कुल��मता�इसका��वदे शी�मु�ा�(�वदे शी�मु�ा) आर��त�है।�
• इसम��मूल��प�से�तीन�घटक�शा�मल�ह�- �वदे शी�मु�ा�संप���, कुल�सोने�का�भंडार�और�आईएमएफ�म��एक�अथ��व�था�के�कुल��वशेष
आहरण�अ�धकार�(एसडीआर)।�
• भारतीय��रजव��ब�क��ारा�भारत�सरकार�के��लए��रज़व��का��बंधन��कया�जाता�है�और�मु�य�घटक��वदे शी�मु�ा�संप���है।�
• �वदे शी�मु�ा�भंडार�आ�थ�क�मंद��के�मामले�म��भारत�के��लए�र�ा�क��पहली�पं���के��प�म��काय��करता�है।��वदे शी�मु�ा�भंडार�बाहरी
�ापार�और�भुगतान�क��सु�वधा��दान�करता�है�तथा�भारत�म���वदे शी�मु�ा�बाजार�के���मक��वकास�और�रखरखाव�को�बढ़ावा�दे ता�है।�

QUESTION 21.
रा�ीय�आय�( National income ) म���न�न�ल�खत�त�व�शा�मल�ह��-

1. एक�नए�घर�का��नमा�ण�
2. लॉटरी�पुर�कार�क��जीत�
3. शेयर��क���ब���से�शेयरधारक��क��कमाई�
4. एक��नयो�ा��ारा�एक�कम�चारी�को��दया�गया��कराया-मु��घर�

उपयु��कूट�चुन�:

a) 2 और�4
b) 1, 3 और�4
c) 1 और�4
d) 1, 2, 3 और�4
Correct Answer: C
Your Answer:
Explanation

Solution (c)

कथन��व�ेषण:

एक��नयो�ा��ारा�एक
शेयर��क���ब���से
एक�नए�घर�का��नमा�ण� लॉटरी�पुर�कार�क��जीत� कम�चारी�को��दया�गया
शेयरधारक��क��कमाई�
�कराया-मु��घर�

स�य� अस�य� अस�य� स�य�

IASbaba
Score:
Web: http://ilp.iasbaba.com/
69.00 /
Email: ilp@iasbaba.com
Page 110 200
AIPTS/ILP VETERANS -
Exam Title :
2020 T...
Email : sahil.law.jmi@gmail.com
Contact : 9540210113

हां�, इसे�रा�ीय�आय�म��शा�मल नह��, इसे�रा�ीय�आय�म�


नह��, इसे�रा�ीय�आय�म��शा�मल हां�, यह�आय��व�ध��ारा�रा�ीय
�कया�जाएगा��य��क�यह�पूंजी शा�मल�नह���कया�जाएगा
नह���कया�जाएगा��य��क�यह आय�म��शा�मल�है��य��क�यह
�नमा�ण�का�एक��ह�सा�है�तथा �य��क�यह�एक��व�ीय�दावा�है
अथ��व�था�म��व�तु��और कम�चा�रय��को��दए�जाने�वाले�'
इससे�अथ��व�था�म��व�तु� तथा��कसी�भी�उ�पादक
सेवा��के��वाह�को�नह��जोड़ता वेतन�के�समान�' का�एक��ह�सा
और�सेवा��का�उ�पादन�होता ग�त�व�ध�म��योगदान�नह��करता
है।� है।�
है।� है।�

QUESTION 22.
�न�न�ल�खत�कथन��पर��वचार�कर�:

1. ब�क�बैल�स�शीट�(खाता�बही) पर�जमा�रा�श�को�दे यता��ेणी�( liability category ) म��रखा�जाता�है।�


2. उधार��दए�गए�धन�को�संप���क���ेणी�म��रखा�जाता�है।�

सही��वक�प�का�चयन�कर�:

a) केवल�1
b) केवल�2
c) दोन��
d) कोई�नह��
Correct Answer: C
Your Answer:
Explanation

Solution (c)

कथन��व�ेषण:

कथन�1 कथन�2

स�य� स�य�

ब�क�बैल�स�शीट�पर�जमा�रा�श�को�दे यता��ेणी�म��रखा�गया�है
उधार�धन�को�प�रसंप���क���ेणी�म��रखा�जाता�है��य��क�यह��याज
�य��क�ब�क��को�उस�रा�श�का�भुगतान��याज�स�हत�वापस
के��प�म��ब�क�के��लए�आय�उ�प��करता�है।�
करना�होगा।�

QUESTION 23.
चतुधा�तुक�( quaternary ) और�पंचसं�यक�( quinary ) ग�त�व�धय��म���या�अंतर�है�?

a) चतुधा�तुक�ग�त�व�धयां�आईट��संब��सेवा��से�संबं�धत�ह��, जब�क�पंचसं�यक�ग�त�व�धयां��व�ीय�सेवा��से�संबं�धत�ह�।�
b) चतु�व�ध�ग�त�व�धयाँ�, सूचना��सं�करण�से�संबं�धत�ह��, जब�क�, पंचसं�यक�ग�त�व�धयां��नण�य�लेने�से�संबं�धत�ह�।�
c) a और�b दोन��
d) इनमे�से�कोई�भी�नह��

IASbaba
Score:
Web: http://ilp.iasbaba.com/
69.00 /
Email: ilp@iasbaba.com
Page 111 200
AIPTS/ILP VETERANS -
Exam Title :
2020 T...
Email : sahil.law.jmi@gmail.com
Contact : 9540210113

Correct Answer: B
Your Answer:
Explanation

Solution (b)

Approach/Elimination

तृतीयक�ग�त�व�धय��को�चतुधा�तुक�( quaternary ) और�पंचसं�यक�( quinary ) ग�त�व�धय��म��वग�कृत��कया�गया�है।�पहले�म�


सूचना��का�सं�हण�, उ�पादन�, �सं�करण�और��सार�शा�मल�है।�उदाहरण�के��लए�, सॉ�टवेयर�डेवलपस��, कर�सलाहकार�आ�द।�

पंचसं�यक�( quinary ) ग�त�व�धय��म���नमा�ण�, पुन:�व�था�, और��वचार��क���ा�या�पर��यान�क���त��कया�जाता�है।�यह�उ�च��तरीय


�नण�य�लेने�से�संबं�धत�है।�इस�लए�, इसे�' गो�ड�कॉलर�' �ोफेशन�भी�कहा�जाता�है।�उदाहरण�के��लए�, �ावसा�यक�अ�धकारी�, अनुसंधान
वै�ा�नक�आ�द।�

कथन��व�ेषण:

कथन�( a) कथन�( b)

अस�य� स�य�

चतुधा�तुक�ग�त�व�धय��म��सूचना�का�सं�हण�, उ�पादन�, �सं�कर


ण�और��सार�शा�मल�है।�उदाहरण�के��लए�, सॉ�टवेयर�डेवलपस� पंचसं�यक�ग�त�व�धय��म���नमा�ण�, पुन: �व�था�, औ
, कर�सलाहकार�आ�द।� र��वचार��क���ा�या�पर��यान�क���त��कया�जाता�है।
यह�उ�च��तरीय��नण�य�लेने�से�संबं�धत�है।�
�व�ीय�सेवाएँ�पंचसं�यक�ग�त�व�धय��का��ह�सा�नह��ह�।�

QUESTION 24.
भारत�म��योजना�के�मूल��स�ांत��न�न�ल�खत�म��से�कौन�से�ह��?

1. �वकास�
2. आधु�नक�करण�
3. आ�म��नभ�रता�
4. सामा�जक��याय�

सही��वक�प�चुन�:

a) 1, 2 और�3
b) 1 और�3
c) 1, 3 और�4
d) उपरो��सभी�
Correct Answer: D
Your Answer:
Explanation

Solution (d)

IASbaba
Score:
Web: http://ilp.iasbaba.com/
69.00 /
Email: ilp@iasbaba.com
Page 112 200
AIPTS/ILP VETERANS -
Exam Title :
2020 T...
Email : sahil.law.jmi@gmail.com
Contact : 9540210113

Approach:

�वकास�, आधु�नक�करण�, आ�म�नभ�रता�और�सामा�जक��याय�- ये�सभी�योजना�के�मूल��स�ांत�ह�।�

नई�तकनीक�को�अपनाने�को�आधु�नक�करण�कहा�जाता�है।�हालां�क�, आधु�नक�करण�केवल�नई�तकनीक�के�उपयोग�के��लए�ही�नह��, ब��क


सामा�जक����कोण�म��भी�इस�तरह�क��मा�यता�के��प�म��प�रवत�न�करता�है��क�म�हला��को�पु�ष��के�समान�अ�धकार�होना�चा�हए।�

अपने��वयं�के�संसाधन��का�उपयोग�करना�आ�म�नभ�रता�कहलाता�है।�एक�रा��अपने��वयं�के�संसाधन��का�उपयोग�करके�या�अ�य�दे श��से
आया�तत�संसाधन��का�उपयोग�करके�आ�थ�क��वकास�और�आधु�नक�करण�को�बढ़ावा�दे �सकता�है।�पहली�सात�पंचवष�य�योजना��ने
आ�म�नभ�रता�को�मह�व��दया�, �जसका�अथ��है�उन�व�तु��के�आयात�से�बचना�जो�भारत�म��ही�उ�पा�दत�क��जा�सकती�थ�।��वदे श��म��, �वशेष
कर�भोजन�पर�हमारी��नभ�रता�को�कम�करने�के��लए�इस�नी�त�को�एक�आव�यकता�माना�गया।�

QUESTION 25.
�न�न�ल�खत�म��से�कौन�सा�कथन�मु�ा�के�मू�य�ास�( depreciation ) के�संदभ��म��सही�है�/ ह��?

1. यह�अ�य�मु�ा��के�संबंध�म��घरेलू�मु�ा�के�मू�य�म���गरावट�है�
2. यह�सरकार�के�ह�त�ेप�से�जानबूझकर��कया�जाता�है�

उ�चत�उ�र�चुन�:

a) केवल�1
b) केवल�2
c) दोन��1 और�2
d) न�तो�1 और�न�ही�2
Correct Answer: A
Your Answer:
Explanation

Solution (a)

Approach:

मु�ा�मू�य�ास�( depreciation ) एक�या�अ�धक��वदे शी�संदभ��मु�ा��के�संबंध�म���कसी�दे श�क��मु�ा�के�मू�य�का��ास�है�, आमतौर�पर


एक�अ�थायी��व�नमय�दर��णाली�म���जसम��कोई�आ�धका�रक�मु�ा�मू�य�नह��रखा�जाता�है।��लो�ट�ग�ए�सच�ज�रेट��स�टम�म��, मु�ा�का�मू�य
ऊपर�(या�नीचे) जाता�है�अगर�आपू�त��क��तुलना�म��इसके��लए�मांग�अ�धक�(या�कम) हो�जाती�है।�

�सरी�ओर�, जब�सरकार�अपनी�मु�ा�क���व�नमय�दर�कम�करती�है�, तो�इसे�अवमू�यन�( Devaluation ) कहा�जाता�है�, यह�जानबूझकर


�कया�जाता�है�और�दे श�के��नया�त�को�बढ़ावा�दे ने�के��लए��कया�जाता�है।�

कथन��व�ेषण:

कथन�( 1) कथन�( 2)

स�य� अस�य�

IASbaba
Score:
Web: http://ilp.iasbaba.com/
69.00 /
Email: ilp@iasbaba.com
Page 113 200
AIPTS/ILP VETERANS -
Exam Title :
2020 T...
Email : sahil.law.jmi@gmail.com
Contact : 9540210113

मु�ा�मू�य�ास�एक�या�अ�धक��वदे शी�संदभ��मु�ा��के�संबंध�म�
यह�जानबूझकर�सरकार�के�ह�त�ेप�से�नह���कया�गया�है।�
�कसी�दे श�क��मु�ा�के�मू�य�का��ास�है।�

QUESTION 26.
�न�न�म��से�, �ड�पोजेबल�(�य�करने�यो�य) आय�के�बारे�म��सही��ववरण�क��पहचान�कर��-

a) यह�आयकर�क��कटौती�के�बाद�प�रवार��के�पास�उपल�ध�धन�है।�
b) यह�प�रवार��के�पास�उपल�ध�तरल�धन�है।�
c) यह�प�रवार���ारा�अ�ज�त�सकल�धन�है।�
d) एक�बार�बचत�करने�के�बाद�यह�प�रवार��के��लए�उपल�ध�धन�है।�
Correct Answer: A
Your Answer:
Explanation

Solution (a)

Explanation:

�ड�पोजेबल�आय�, �जसे��य�करने�यो�य����गत�आय�(डीपीआई) के��प�म��भी�जाना�जाता�है�, वह�धनरा�श�है�जो�घर��को�खच��करने�और


बचत�के��लए�उपल�ध�होती�है�, �जसका�आयकर�के�बाद��हसाब�लगाया�जाता�है।��ड�पोजेबल����गत�आय�को�अ�सर�कई��मुख�आ�थ�क
संकेतक��म��से�एक�के��प�म��मॉ�नटर��कया�जाता�है�, �जसका�उपयोग�अथ��व�था�क��सम����थ�त�को�मापने�के��लए��कया�जाता�है।�

अथ�शा��ी�डीपीआई�का�उपयोग�घर��क��बचत�और�खच��क��दर��को��नधा��रत�करने�के��लए�एक��ारं�भक��ब���के��प�म��करते�ह�।�

�ड�पोजेबल�आय�सभी�आव�यक�भुगतान��को�घटाता�है�, जैसे��क�बंधक�, �वा��य�बीमा�, खा��और�प�रवहन�, �ववेकाधीन�आय�के�समान


होता�है।��ड�पोजेबल�आय�के�इस��ह�से�को�आय�अज�क�चुनता�है�या�वैक��पक��प�से�खच���कया�जा�सकता�है�, इसे�बचाया�जा�सकता�है।
���गत�बचत�दर�, �ड�पोजेबल�आय�का�वह���तशत�है�जो�बाद�क��तारीख�म��सेवा�नवृ���या�उपयोग�के��लए�बचत�जाती�है।�उपभोग�करने�के
�लए�सीमांत��वृ����ड�पोजेबल�आय�के���येक�अ�त�र��डॉलर�के���तशत�का���त�न�ध�व�करती�है�, जब�क�सीमांत��वृ���को�बचाने�के��लए
जो���तशत�बच�जाता�है�उसे�दशा�ता�है।�

QUESTION 27.
�ा�फ़कल�व��जो��कसी�अथ��व�था�म��मु�ा�फ��त�और�बेरोजगारी�के�म�य�संबंध�को�रेखां�कत�करता�है�

a) लाफ़र�व��
b) �फ�ल�स�व��
c) �ाइडमैन�व��( Friedman curve )
d) रान�व��( Rahn curve )
Correct Answer: B
Your Answer:
Explanation

Solution (b)

Analysis:

�फ�ल�स�व��

IASbaba
Score:
Web: http://ilp.iasbaba.com/
69.00 /
Email: ilp@iasbaba.com
Page 114 200
AIPTS/ILP VETERANS -
Exam Title :
2020 T...
Email : sahil.law.jmi@gmail.com
Contact : 9540210113

• �च�मय�व��जो�एक�अथ��व�था�म��मु�ा�फ��त�और�बेरोजगारी�के�म�य�संबंधो�को�दशा�ता�है।�
• इसका�अथ��है��क�अथ��व�था�म��घटती�बेरोजगारी�, ( यानी�, रोजगार�के��तर�म��वृ��) मु�ा�फ��त�क��उ�च�दर�के�साथ�सहसंब�
होगी।�जब�क�बेरोजगारी�और�मु�ा�फ��त�के�बीच�अ�पका�लक�संबंध�है�, यह�लंबे�समय�तक�नह��दे खा�जा�सकता�है।�
• 1968, �म�टन���डमैन�ने�जोर�दे कर�कहा��क��फ�ल�स�व��केवल�अ�पाव�ध�म��लागू�था�तथा�लंबे�समय�म��, मु�ा�फ��त�क��नी�तय�
से�बेरोजगारी�म��कमी�नह��होगी।�
• ��डमैन�ने�तब�सही�भ�व�यवाणी�क��थी��क�1973-75 क��मंद��म��, मु�ा�फ��त�और�बेरोजगारी�दोन��म��वृ���होगी।�लंबे�समय�तक
चलने�वाले��फ�ल�स�व��को�अब�बेरोजगारी�क���ाकृ�तक�दर�पर�एक�ऊ�वा�धर�रेखा�के��प�म��दे खा�जाता�है�, जहां�मु�ा�फ��त�क��दर
का�बेरोजगारी�पर�कोई��भाव�नह��पड़ता�है।�

लाफ़र�व��

• यह�कराधान�क��दर��और�सरकारी�राज�व�के�प�रणाम�व�प��तर��के�बीच�एक�सै�ां�तक�संबंध��दखाता�है।�यह�कर�यो�य�आय�लोच
क��अवधारणा�को�दशा�ता�है�- यानी�, कराधान�क��दर�म��प�रवत�न�के���तउ�र�म��कर�यो�य�आय�म��प�रवत�न�होता�है।�

�ाइडमैन�कव��( Friedman Curve)

• जैसे�कोई�व��नह��है�

र��व��( Rahn Curve)

IASbaba
Score:
Web: http://ilp.iasbaba.com/
69.00 /
Email: ilp@iasbaba.com
Page 115 200
AIPTS/ILP VETERANS -
Exam Title :
2020 T...
Email : sahil.law.jmi@gmail.com
Contact : 9540210113

• रा��व��अमे�रक��आ�थ�क�अथ�शा��ी��रचड��ड��यू�र���ारा�1996 म����ता�वत�एक�आ�थ�क��स�ांत�को�दशा�ने�के��लए��योग��कया
जाने�वाला�एक��ाफ�है�, जो�बताता�है��क�आ�थ�क��वकास�को�अ�धकतम�करने�वाले�सरकारी��य��का�एक��तर�है।��स�ांत�का
उपयोग��ला�सकल�उदारवा�दय���ारा�सम��सरकारी��य�और�कराधान�म��कमी�के��लए��कया�जाता�है।�
• सरकारी�ऋण�क��उ�चत�रा�श�पर�प�र�े�य�के�दो�पर�पर�जुड़े�पहलू�ह�।�एक�यह�है��क��या�सरकारी�ऋण�एक�बोझ�है�तथा��सरा�ऋण
के��व�पोषण�के�मु�े�के�बारे�म��है।�
• अ�सर�यह�तक���दया�गया�है��क�साव�ज�नक�ऋण�से�कोई�फक��नह��पड़ता��य��क�हम�इसे�अपने�आप�पर�बकाया�मानते�ह�।�' ऐसा
इस�लए�है��य��क�य��प�पी�ढ़य��के�बीच�संसाधन��का�ह�तांतरण�होता�है�, �य�श���रा��के�भीतर�बनी�रहती�है।�हालाँ�क�, �वदे �शय�
पर�बकाया��कसी�भी�ऋण�म��एक�बोझ�के��प�म��शा�मल�होता�है��य��क�हम���याज�भुगतान�के��लए��वदे श�म��आय�भेजना�पड़ता�है।�
• य�द�सरकार�बु�नयाद��ढांचे�म���नवेश�करती�है�, तो�भ�व�य�क��पी�ढ़यां�बेहतर�हो�सकती�ह��, बशत��ऐसे��नवेश��पर���तफल��याज�दर�से
अ�धक�हो।�उ�पादन�म��वृ���से�वा�त�वक�ऋण�का�भुगतान��कया�जा�सकता�है।�ऋण�को�तब�बोझ�नह��समझना�चा�हए।�ऋण�क��वृ��
को�सम���प�से�अथ��व�था�क��वृ���से�आंका�जाना�चा�हए।�इस�लए�ऋण�बोझ�है�य�द�यह�उ�पादन�म��भ�व�य�क��वृ���को�कम
करता�है।�

QUESTION 28.
नीचे��दए�गए�कथन��म��से�कौन�सा�/ से�नकद�आर��त�अनुपात�( CRR) के�बारे�म��सही�है�?

1. आरबीआई�ब�क��के��रजव��जैसे�नकद�आर��त�अनुपात�( CRR) को�अपने�पास�रखता�है।�


2. सीआरआर�वृ���का�इ��वट��माक�ट�, बॉ�ड�माक�ट�, �रयल�ए�टे ट�और�ऑटोमोबाइल�से�टर�पर�नकारा�मक��भाव�पड़ता�है।�

उपयु��कूट�चुन�:

a) केवल�1
b) केवल�2
c) दोन��1 और�2
d) न�तो�1 और�न�ही�2
Correct Answer: A
Your Answer:
Explanation

Solution (a)

Approach:

IASbaba
Score:
Web: http://ilp.iasbaba.com/
69.00 /
Email: ilp@iasbaba.com
Page 116 200
AIPTS/ILP VETERANS -
Exam Title :
2020 T...
Email : sahil.law.jmi@gmail.com
Contact : 9540210113

नकद�आर��त�अनुपात�(सीआरआर) वह�रा�श�है�जो�ब�क��क��तरलता�और�सॉ�व�सी�सु�न��त�करने�के��लए��े�ीय��ामीण�ब�क��(आरआरबी)
को�छोड़कर�सभी�अनुसू�चत�वा�ण��यक�ब�क��(एससीबी) को�अपने�कुल�शु��मांग�और�समय�दे यता��(डीट�एल) के�संदभ��म��आरबीआई�के
साथ��कसी�भी�आधार�या�सीमा�क��दर�के��बना�बनाए�रखना�आव�यक�है।)

जब�भी�RBI, CRR दर�को�बढ़ाता�है�, ब�त�अ�धक�तरलता�को�बाजार��से��नकाल��लया�जाता�है।�ब�क��के�पास�ऋण�के��प�म��दे ने�के��लए


कम�नकद��उपल�ध�होती�है।�

कथन��व�ेषण:

कथन�1 कथन�2

स�य� अस�य�

नकद�आर��त�अनुपात�( CRR) वह�रा�श�है�जो�सभी सीआरआर�वृ���का�इ��वट��माक�ट�, �रयल�ए�टे ट�और


अनुसू�चत�वा�ण��यक�ब�क��को�RBI के�पास�बनाए�रखना ऑटोमोबाइल�से�टर�पर�नकारा�मक��भाव�पड़ेगा�(ले�कन
आव�यक�होता�है।� बॉ�ड�माक�ट�पर�नह�)

�व�भ���े���और��े���पर�सीआरआर�बढ़ोतरी�का��भाव�-

• इ��वट��माक�ट�पर: CRR वृ���ब��क�ग��णाली�म��कम�तरलता�लाएगी।�य�द�ब�क��के�पास�कम�नकद��है�, तो�वे�कम�पैसा�उधार�द� गे�या


इ��वट��म���नवेश�नह��कर�गे।�
• कमो�डट��माक�ट�पर: CRR वृ���से�बाजार�म��कम�तरलता�आएगी�और�लोग�कमो�डट��माक�ट�म��कम��नवेश�कर�गे।�
• बॉ�ड�माक�ट�पर: CRR वृ���से�बॉ�ड�माक�ट�को�मदद��मलेगी��य��क�लॉ�ग�टम��यी�ड�(द�घ�कालीन�लाभ) बढ़े गी।�
• ब��क�ग�पर: बढ़���ई��याज�दर��कम�ऋण�वृ���का�कारण�बनती�ह��और�आगे�सम��लाभ�दता�के�बारे�म��बताती�ह�।�इस��कार�, ब��क�ग
शेयर��को��बकवाली�दबाव�का�सामना�करना�पड़�सकता�है।�
• अचल�संप���पर: CRR वृ���से�बाजार��म��तरलता�कम�हो�जाएगी।�उपभो�ा��को�कम��याज�दर��पर�होम�लोन�लेने�क��इ�छा�होगी
, �जसके�प�रणाम�व�प�मांग�म���गरावट�आएगी।�इसके�अलावा��नमा�ण�कंप�नय��को�उ�च�दर��पर�उधार�लेना�होगा�, �जससे�उनके
ओवरहेड्स�बढ़� गे।�
• ऑटो�पर: ऑटो�पर�भी�यही�बात�लागू�होती�है।�उधार�दर��म��वृ���के�कारण�ऑटोमोबाइल�क��मांग�घट�जाएगी�और�उधार�क��दर��म�
वृ���के�कारण�कंपनी�क��लागत�म��तेजी�से�वृ���होगी।�इस�लए�, ऑटो�शेयर��म��भी��गरावट�दे खने�को��मल�सकती�है।�

QUESTION 29.
�न�न�ल�खत�कथन��पर��वचार�कर�:

1. पुनमू��यांकन�( Revaluation ):: सरकार��कसी�भी��वदे शी�मु�ा�के��व���अपनी�मु�ा�क���व�नमय�दर�म��वृ���करती�है�


2. अ�भमु�यन�( Appreciation ):: य�द�एक�मु��चलायमान( floating ) घरेलू�मु�ा�, एक��वदे शी�मु�ा�के�मू�य�के��व���अपना
मू�य�बढ़ाती�है�

उपरो��म��से�कौन�सही�ढं ग�से�सुमे�लत�है�/ है�?

a) केवल�1
b) केवल�2
c) दोन��1 और�2
d) न�तो�1 और�न�ही�2

IASbaba
Score:
Web: http://ilp.iasbaba.com/
69.00 /
Email: ilp@iasbaba.com
Page 117 200
AIPTS/ILP VETERANS -
Exam Title :
2020 T...
Email : sahil.law.jmi@gmail.com
Contact : 9540210113

Correct Answer: C
Your Answer:
Explanation

Solution (c)

कथन��व�ेषण:

कथन�1 कथन�2

स�य� स�य�

अ�भमु�यन�( Appreciation ) :: �वदे शी�मु�ा�बाजार�म��, य�द


एक�मु���वाह�घरेलू�मु�ा�, �वदे शी�मु�ा�के�मू�य�के��व���अपना
पुनमू��यांकन�( Revaluation ) :: �वदे शी�मु�ा�बाजार मू�य�बढ़ाती�है�, तो�यह�अ�भमु�यन�है।�
म���योग��कया�जाने�वाला�श�द��जसका�अथ��है��क�सरकार
घरेलू�अथ��व�था�म��, य�द��कसी�अचल�संप���के�मू�य�म��वृ����ई�है
�कसी�भी��वदे शी�मु�ा�के��व���अपनी�मु�ा�क���व�नमय�दर
, तो�इसे�अ�भमु�यन�के��प�म��भी�जाना�जाता�है।��व�भ�
को�बढ़ाती�है।�
प�रसंप��य��के��लए�अ�भमु�यन�दर��कसी�भी�सरकार��ारा�तय�नह�
क��जाती�है��य��क�वे�कई�कारक��पर��नभ�र�करती�ह��जो��क�अ��य
ह�।�

QUESTION 30.
क���य�सां��यक��काया�लय�( CSO) के�बारे�म��नीचे��दए�गए�कौन�से�कथन�सही�ह��/ ह��?

1. यह��ामीण�, शहरी�, औ�ो�गक���मक��और�संयु��(�ामीण�स�हत�शहरी) के��लए�अ�खल�भारतीय�तथा�रा�य��/ क���शा�सत��दे श�


के��लए�उपभो�ा�मू�य�सूचकांक�(सीपीआई) जारी�करता�है।�
2. यह�ऊजा��सां��यक��, सामा�जक�और�पया�वरण�सां��यक��का�भी��नग�मन�करता�है�तथा�रा�ीय�औ�ो�गक�वग�करण�तैयार�करता�है।�

सही�उ�र�चुने:

a) केवल�1
b) केवल�2
c) दोन��1 और�2
d) न�तो�1 और�न�ही�2
Correct Answer: B
Your Answer:
Explanation

Solution (b)

कथन��व�ेषण:

IASbaba
Score:
Web: http://ilp.iasbaba.com/
69.00 /
Email: ilp@iasbaba.com
Page 118 200
AIPTS/ILP VETERANS -
Exam Title :
2020 T...
Email : sahil.law.jmi@gmail.com
Contact : 9540210113

कथन�1 कथन�2

अस�य� स�य�

क���य�सां��यक��काया�लय�( CSO) दे श�म��सां��यक�य


ग�त�व�धय��का�सम�वय�करता�है�और�सां��यक�य�मानक��का
�वकास�करता�है।�
औ�ो�गक���मक��के��लए�उपभो�ा�मू�य�सूचकांक
(सीपीआई) भारत�सरकार�के��म��यूरो��ारा�जारी��कए�जाते इसक��ग�त�व�धय��म��रा�ीय�खात��का�संकलन�, औ�ो�गक�उ�पादन
ह�।�( CSO �ारा�नह�) सूचकांक�, शहरी�/ �ामीण�/ उपभो�ा�के��लए�उपभो�ा�मू�य
सूचकांक�, मानव��वकास�सां��यक��, रा�य��और�क���शा�सत��दे श�
इस�लए�, कथन�( 1) गलत�है।�
म���ल�ग�सां��यक��शा�मल�ह��तथा�ऊजा��सां��यक��, सामा�जक�और
पया�वरण�सां��यक��का��सार�और�रा�ीय�औ�ो�गक�वग�करण
तैयार�करता�है।�

QUESTION 31.
�वदे शी�मु�ा�बाजार�म���योग��कया�जाने�वाला�श�द�कौन�सा�है�जो�अपने��वदे शी�मु�ा�बाजार�म���कसी�भी�अथ��व�था�म��आसानी�से�उपल�ध
होने�वाली�मु�ा�को�दशा�ता�है�?

a) �ल�भ�मु�ा�( Hard currency )


b) त�त�मु�ा�( Hot currency )
c) नरम�मु�ा�( Soft currency )
d) स�ती�मु�ा�( Cheap currency )
Correct Answer: C
Your Answer:
Explanation

Solution (c)

�व�ेषण:

�ल�भ�मु�ा�( Hard त�त�मु�ा�( Hot नरम�मु�ा�( Soft स�ती�मु�ा�( Cheap


Currency) currency) Currency) currency)

अस�य� अस�य� स�य� अस�य�

IASbaba
Score:
Web: http://ilp.iasbaba.com/
69.00 /
Email: ilp@iasbaba.com
Page 119 200
AIPTS/ILP VETERANS -
Exam Title :
2020 T...
Email : sahil.law.jmi@gmail.com
Contact : 9540210113

यह�अंतरा��ीय�मु�ा�है
�जसम��सबसे�अ�धक
�व�ास��दखाया�जाता�है
तथा�इसक��आव�यकता
हर�अथ��व�था�को�होती
है।� अथ�शा��ी�जे. एम. क��स�(
1930) �ारा�पहली�बार
�व��क��सबसे�मजबूत
�वदे शी�मु�ा�बाजार�का�एक �योग��कया�गया�एक�श�द।
मु�ा�वह�है��जसम��उ�च �वदे शी�मु�ा�बाजार�म���योग
श�द�और��कसी�भी�त�त�मु�ा अगर�कोई�सरकार�अपने
�तर�क��तरलता�होती�है।� �कया�जाने�वाला�एक�श�द
के��लए�एक�अ�थायी�नाम�है। बॉ�ड�को�उनक��प�रप�वता
जो�ऐसी�मु�ा�को�दशा�ता�है
मूल��प�से�, उ�चतम�के कुछ�कारण��के�कारण�, य�द (पूण�-प�रप�वता�क��क�मत�
जो��कसी�भी�अथ��व�था�म�
साथ-साथ�अ�य�धक एक�त�त�मु�ा��कसी�समय�के पर) से�पहले�खरीदना�शु�
अपने��वदे शी�मु�ा�बाजार�म�
�व�वध��नया�त�वाली �लए�एक�अथ��व�था�को�तेज कर�दे ती�है�, तो�जो�पैसा
आसानी�से�उपल�ध�है।
अथ��व�था�जो�अ�य ग�त�से��वक�सत�कर�रही�है�, अथ��व�था�म���वा�हत�होता
उदाहरण�के��लए�, भारतीय
दे श��(उ�च��तर�क� तो�कठोर�मु�ा�के�त�त�होने�के है�, उसे�स�ती�मु�ा�के��प�म�
�वदे शी�मु�ा�बाजार�म���पया
तकनीक�, र�ा�उ�पाद��, �लए�जाना�जाता�है।�द��ण जाना�जाता�है�, �जसे�स�ती
एक�नरम�मु�ा�है।�यह�मूल
जीवन�र�क�दवा��और पूव��ए�शयाई�संकट�के�मामले मु�ा�भी�कहा�जाता�है।�
�प�से�हाड��मु�ा�के��लए
पे�ो�लयम�उ�पाद��के म��, अमे�रक��डॉलर�त�त�हो
�वपरीत�श�द�है।�
गया�था।� ब��क�ग�उ�ोग�म��, इसका�अथ�
�प�म�) के��लए
है�तुलना�मक��प�से�कम�/
बा�यकारी�आयात�ह��,
नरम��याज�दर��क��अव�ध।�
�व��म��अपनी�मु�ा�के
�लए�उ�च�मांग�भी
बनाएंगे�और��ल�भ�मु�ा
बन�गी।�इनक��सदै व�कमी
होती�है।�

QUESTION 32.
�न�न�ल�खत�कथन��पर��वचार�कर��

1. �य��के�कुल�मू�य��को�मापना�, जो�फम��को�अं�तम�व�तु��और�सेवा��के��लए��ा�त�होते�ह��, जो�वे�उ�पा�दत�करते�ह��, �य��व�ध�(


Expenditure method ) कहलाती�है।�
2. सभी�फम���ारा�उ�पा�दत�अं�तम�व�तु��और�सेवा��के�कुल�मू�य��को�मापने�क���व�ध�को�उ�पाद��व�ध�( Product method )
कहा�जाता�है।�
3. सभी�कारक�भुगतान��के�कुल�योग�को�मापने�क���व�ध�को�आय��व�ध�( Income method ) कहा�जाता�है�

गलत�कथन�चुन��

a) केवल�1 और�2
b) केवल�2 और�3
c) केवल�1 और�3
d) इनमे�से�कोई�भी�नह��
Correct Answer: D
Your Answer:
Explanation

Solution (d)

IASbaba
Score:
Web: http://ilp.iasbaba.com/
69.00 /
Email: ilp@iasbaba.com
Page 120 200
AIPTS/ILP VETERANS -
Exam Title :
2020 T...
Email : sahil.law.jmi@gmail.com
Contact : 9540210113

कथन��व�ेषण:

सभी�कथन�सही�ह�।�

कथन�1 कथन�2 कथन�3

स�य� स�य� स�य�

कंप�नयां�अं�तम�व�तु��और�सेवा��के
सभी�फम���ारा�उ�पा�दत�अं�तम�व�तु��और सभी�कारक�भुगतान��के�कुल�योग�को
�लए��य�के�कुल�मू�य��को�मापते��ए�, प्
सेवा��के�कुल�मू�य��को�मापने�क���व�ध�को मापने�क���व�ध�को�आय��व�ध�कहा
रा�त�करती�ह��, जो�वे�उ�पा�दत�करती�ह��,
उ�पाद��व�ध�कहा�जाता�है।� जाता�है।�
�य��य��व�ध�कहलाती�है।�

QUESTION 33.
�न�न�ल�खत�कथन��पर��वचार�कर�:

1. बचत�ब�क�खाते�म��समय�और�मांग�दोन��दे यताएं�होती�ह�।�
2. चालू�ब�क�खाते�म��समय�दे यताएं�होती�ह��, ले�कन�कोई�मांग�दे यताएं�नह��होती�ह�।�
3. साव�ध�जमा�( �फ��ड��डपॉ�जट) म��मांग�दे नदा�रयां�होती�ह��ले�कन�समय�दे नदा�रयां�नह��होती�ह�।�

उपरो��कथन��म��से�कौन�सा�सही�ह��?

a) केवल�1
b) 2 और�3
c) 1 और�3
d) उपरो��सभी�
Correct Answer: A
Your Answer:
Explanation

Solution (a)

कथन��व�ेषण:

कथन�1 कथन�2 कथन�3

स�य� अस�य� अस�य�

IASbaba
Score:
Web: http://ilp.iasbaba.com/
69.00 /
Email: ilp@iasbaba.com
Page 121 200
AIPTS/ILP VETERANS -
Exam Title :
2020 T...
Email : sahil.law.jmi@gmail.com
Contact : 9540210113

एक�बचत�खाता�एक�जमा�खाता�है�जो
एक�खुदरा�ब�क�म��रखा�जाता�है�जो��याज
का�भुगतान�करता�है�ले�कन�ए�सच�ज�के
मा�यम�के�संक�ण��अथ��म��धन�का�उपयोग
नह���कया�जा�सकता�है�(उदाहरण�के��लए
, एक�चेक��लखकर)।�

इन�खात��ने��ाहक��को�मौ��क��रटन� चालू�ब�क�खाता�उन��वसा�यय���ारा�खोला
अ�ज�त�करते��ए�अपनी�तरल�संप���के जाता�है��जनके�ब�क�के�साथ��नय�मत�लेनदे न
एक��ह�से�को�अलग�करने��दया।�ब�क�के क��अ�धक�सं�या�है।�इसम���डपॉ�जट�, �वद् साव�ध�जमा�100% समय�दे यता�है।�तय
�लए�, एक�बचत�खाते�म��पैसा�तुरंत�कॉल �ॉल�और�कॉ��ा��ांजै�शन�शा�मल�ह�।� समय�से�पहले�आप��फ��ड��डपॉ�जट�से
करने�यो�य�नह��हो�सकता�है�और�कुछ पैसा�नह���नकाल�सकते।�अगर�आपको
इसे��डमांड��डपॉ�जट�अकाउंट�के��प�म��भी
�ा�धकार�म��, आर��त�आव�यकता�को इससे�पैसे��नकालने�ह��, तो�ब�क�आपसे
जाना�जाता�है।�
लागू�नह��करता�है।�ब�क�के�वॉ�ट�म��नकद जुमा�ना�वसूलेगा।�
का�उपयोग�इस��कार��कया�जा�सकता�है इसके��लए�ब�क��याज�नह��दे ता�है।�ब��क
, उदाहरण�के��लए�, �याज�दे ने�वाले�ऋण� यह�सेवा�शु�क�लेता�है।�
को��न�ध�दे ने�के��लए।�

पैसे�क��सीमा�जो�आप��कसी�भी�समय
�नकाल�सकते�ह��, वह�है�ब�क�क��दे यता
जो�उसे�अपने�पास�रखनी�होती�है।�शेष
रा�श�एक�समय�दे यता�है��जसके��लए
आपको�ब�क�को�पूव��सूचना�दे नी�होगी।�

QUESTION 34.
भारत�म��सकल�घरेलू�बचत�म���न�न�ल�खत�म��से��कसक��सबसे�बड़ी��ह�सेदारी�है�?

a) हाउसहो�ड�बचत�
b) �नजी�कॉप�रेट��े��
c) साव�ज�नक��े��
d) इनमे�से�कोई�भी�नह��
Correct Answer: A
Your Answer:
Explanation

Solution (a)

Approach:

सकल�घरेलू�बचत�और�सकल�घरेलू�पूंजी��नमा�ण�

डेटा�सकल�घरेलू�बचत�और�सकल�घरेलू�पूंजी��नमा�ण�को�संद�भ�त�करता�है।�यह����गत��प�से��नजी�कॉप�रेट��े��, साव�ज�नक��े��और
हाउसहो�ड�के��े��के�बारे�म���ववरण�दे ता�है।�सकल�घरेलू�बचत�दे श�क���व�ीय���थ�त�और��वकास�को�इं�गत�करता�है�, �य��क�घरेलू�बचत
साव�ज�नक�सेवा��को��न�ध�दे ने�के��लए�सरकार�के�उधार�का�मु�य��ोत�है।�

आरबीआई�क���रपोट� �के�अनुसार�, हाउस�हो�ड�बचत�सबसे�अ�धक�है��जसके�बाद��नजी�कॉरपोरेट��े��और�साव�ज�नक��े��आते�ह�।�

QUESTION 35.
सकल�रा�ीय�उ�पाद�( GNP) क��गणना�क��जाती�है�

IASbaba
Score:
Web: http://ilp.iasbaba.com/
69.00 /
Email: ilp@iasbaba.com
Page 122 200
AIPTS/ILP VETERANS -
Exam Title :
2020 T...
Email : sahil.law.jmi@gmail.com
Contact : 9540210113

a) मू�य�ास�स�हत�अथ��व�था�म��सभी�कंप�नय���ारा�जोड़े�गए�सकल�मू�य�का�योग�माप�कर।�
b) मू�य�ास�को�छोड़कर�अथ��व�था�म��सभी�फम���ारा�जोड़े�गए�सकल�मू�य�का�योग�माप�कर।�
c) मू�य�ास�और��वदे श��म��शु��कारक�आय�स�हत�अथ��व�था�म��सभी�फम���ारा�जोड़े�गए�सकल�मू�य�का�योग�माप�कर।�
d) मू�य�ास�और��वदे श��म��शु��कारक�आय�को�छोड़कर�अथ��व�था�म��सभी�फम���ारा�जोड़े�गए�सकल�मू�य�के�योग�को�माप�कर।�
Correct Answer: C
Your Answer:
Explanation

Solution (c)

Explanation:

वृहद�आ�थ�क�चर�( macroeconomic variable) जो�इस�तरह�के�वध�न�और�घटाव�को��यान�म��रखता�है�, सकल�रा�ीय�उ�पाद�( GN


P) के��प�म��जाना�जाता�है।�

जीएनपी�दे श�के��नवा�सय���ारा�उ�पा�दत�कुल�उ�पादन�के�कुल�मौ��क�मू�य�को�मापता�है।�इस�लए�, दे श�क��सीमा��के�भीतर��वदे शी


�नवा�सय���ारा�उ�पा�दत��कसी�भी�उ�पादन�को�जीएनपी�क��गणना�म��बाहर�रखा�जाता�है�, जब�क�दे श�के��नवा�सय���ारा�अपनी�सीमा��के
बाहर�उ�पा�दत��कसी�भी�उ�पादन�को��गना�जाता�है।�

जीएनपी�एक�अ�य�मह�वपूण��आ�थ�क�उपाय�से�संबं�धत�है��जसे�सकल�घरेलू�उ�पाद�(जीडीपी) कहा�जाता�है�, जो�दे श�क��सीमा��के�भीतर


उ�पा�दत�सभी�उ�पादन�को�जोड़ता�है�, �बना�इसके��क�उ�पादन�के�साधन��को��कसके��ारा��कया�गया�है।�जीएनपी�जीडीपी�के�साथ�शु��होता
है�, �वदे शी��नवेश�से��नवा�सय��क���नवेश�आय�को�जोड़ता�है�, और��कसी�दे श�के�भीतर�अ�ज�त��वदे शी��नवा�सय��क���नवेश�आय�को�घटाता
है।�

जीएनपी�= जीडीपी�+ शेष��व��म���नयो�जत�उ�पादन�के�घरेलू�कारक���ारा�अ�ज�त�फै�टर�आय�- घरेलू�अथ��व�था�म��काय�रत�शेष��व��के


उ�पादन�के�कारक���ारा�अ�ज�त�आय।�

इस�लए�, �वदे श�से�जीएनपी�= जीडीपी�+ �वदे श�से�शु��कारक�आय�(नेट�फै�टर�आय)

( �वदे श�से�शु��कारक�आय�= शेष��व��म���नयो�जत�उ�पादन�के�घरेलू�कारक���ारा�अ�ज�त�फै�टर�आय�- घरेलू�अथ��व�था�म���नयो�जत�शेष


�व��के�उ�पादन�के�कारक���ारा�अ�ज�त�फै�टर�आय)।�

QUESTION 36.
तरलता�समायोजन�सु�वधा�( LAF) के�संबंध�म���न�न�ल�खत�पर��वचार�कर�।�

1. यह�ब�क��को�पुनख�रीद�समझौत��के�मा�यम�से�धन�उधार�लेने�क��अनुम�त�दे ता�है।�
2. एलएएफ�म��दो�उपकरण�होते�ह��- सां�व�धक�तरलता�अनुपात�और�नकद�आर��त�अनुपात।�

ऊपर��दए�गए�कथन��म��से�कौन�सा�सही�है�/ ह��?

a) केवल�1
b) केवल�2
c) दोन��1 और�2
d) न�तो�1 और�न�ही�2
Correct Answer: A
Your Answer:
Explanation

Solution (a)

Approach:

IASbaba
Score:
Web: http://ilp.iasbaba.com/
69.00 /
Email: ilp@iasbaba.com
Page 123 200
AIPTS/ILP VETERANS -
Exam Title :
2020 T...
Email : sahil.law.jmi@gmail.com
Contact : 9540210113

तरलता�समायोजन�सु�वधा�(एलएएफ) एक�मौ��क�नी�त�उपकरण�है�जो�ब�क��को�पुनख�रीद�समझौत��के�मा�यम�से�धन�उधार�लेने�क��अनुम�त
दे ता�है।�एलएएफ�का�उपयोग�ब�क��को�तरलता�म���दन�- ��त�दन�असंतुलन�को�समायो�जत�करने�म��सहायता�करने�के��लए��कया�जाता�है।
एलएएफ�म��रेपो�और��रवस��रेपो�ऑपरेशन�होते�ह�।�

वैधा�नक�तरलता�अनुपात�और�नकद�आर��त�अनुपात�, ऋण��नयं�ण�उपकरण�(�े�डट�कं�ोल�इं�ट�म�ट) ह��जो��वयरेबल��रज़व��अनुपात�के


अंतग�त�आते�ह�।�ये�वे��रज़व��ह���ज�ह��अनुसू�चत�ब�क��को�बनाए�रखना�पड़ता�है�और�वे��े�डट��नमा�ण�को��नयं��त�करने�के��लए��व�वध�होते�ह�।�

कथन��व�ेषण:

कथन�1 कथन�2

स�य� अस�य�

तरलता�समायोजन�सु�वधा�(एलएएफ) ब�क��को�पुनख�रीद तरलता�समायोजन�सु�वधा�( LAF) म��दो�उपकरण�होते�ह��- रेपो


समझौत��के�मा�यम�से�धन�उधार�लेने�क��अनुम�त�दे ती�है।� और��रवस��रेपो�प�रचालन।�

QUESTION 37.
�न�न�ल�खत�म��से�कौन��कसी�दे श�क��रा�ीय�आय�क��गणना�करने�क���व�ध�नह��है�?

a) �याज��व�ध�( Interest Method )


b) आय��व�ध�( Income method )
c) �य��व�ध�( Expenditure method )
d) मू�य�- व�ध�त��व�ध�( Value – Added Method )
Correct Answer: A
Your Answer:
Explanation

Solution (a)

Approach

रा�ीय�आय�क��गणना�के��लए- एक�अथ��व�था�को�तीन�अलग-अलग�कोण��से�दे खा�जाता�है:

1. एक�अथ��व�था�म��उ�पादन�इकाइय��को��ाथ�मक�, मा�य�मक�और�तृतीयक��े���म��वग�कृत��कया�जाता�है।�इस�वग�करण�के�आधार
पर�, रा�ीय�आय�को�मापने�के��लए�मू�य�व�ध�त�प��त�( value-added method) का�उपयोग��कया�जाता�है।�
2. अथ��व�था�को�उ�पादन�के��व�भ���कार��के�कारण����य��और�प�रवार��को�संयोजन�के��प�म��भी�दे खा�जाता�है।�इस�संयोजन�के
आधार�पर�, रा�ीय�आय�का�अनुमान�लगाने�के��लए�आय��व�ध�( income method) का�उपयोग��कया�जाता�है।�
3. अथ��व�था�को�खपत�, बचत�और��नवेश�के��लए�उपयोग�क��जाने�वाली�इकाइय��के�सं�ह�के��प�म��दे खा�जाता�है।�इस�सं�ह�के
आधार�पर�, रा�ीय�आय�क��गणना�के��लए�अं�तम��य�प��त�( final expenditure method) का�उपयोग��कया�जाता�है।�

�याज��व�ध�( Interes आय��व�ध�( Income अं�तम��य��व�ध�( Final मू�य�व�ध�त��व�ध�( Value-


t Method) Method) Expenditure Method) added Method)

IASbaba
Score:
Web: http://ilp.iasbaba.com/
69.00 /
Email: ilp@iasbaba.com
Page 124 200
AIPTS/ILP VETERANS -
Exam Title :
2020 T...
Email : sahil.law.jmi@gmail.com
Contact : 9540210113

अं�तम��य��व�ध�, �जसे�अं�तम
मू�य�व�ध�त��व�ध�, �जसे�नेट
आय��व�ध�, �जसे�कारक�आय उ�पाद��व�ध�के��प�म��भी�जाना
आउटपुट��व�ध�भी�कहा�जाता�है�,
�व�ध�के��प�म��भी�जाना�जाता जाता�है�, का�उपयोग��कसी
का�उपयोग�अथ��व�था�क�
है�, का�उपयोग�रा�ीय�उ�पाद�के �न��त�समय�के�दौरान�आ�थ�क
उ�पादन�इकाइय��के�सकल�घरेलू
उ�पादन�म��उपयोग��कए�जाने �े��के�भीतर�अं�तम�व�तु��और
उ�पाद�म��योगदान�को�मापने�के
वाले�उ�पादन�के�मूल�कारक��के सेवा��के�उ�पादन�के��लए
�लए��कया�जाता�है।��सरे�श�द��म�
�लए�अ�ज�त�सभी�आय�क� उ�पादन�इकाइय���ारा��कए�गए
, एक�अथ��व�था�म����येक
�याज��व�ध��कसी�दे श�क� गणना�करने�के��लए��कया�जाता अं�तम��य�को�मापने�के��लए
उ�ोग��ारा�जोड़े�गए�मू�य-व�ध�त
रा�ीय�आय�क��गणना है।�परंपरागत��प�से�, उ�पादन �कया�जाता�है।�
�व�ध�के�उपाय।�मू�य-व�ध�त��व�ध
करने�क���व�ध�नह��है।� के�चार�कारक�ह��, अथा�त्�भू�म�, के�मा�यम�से�रा�ीय�आय�क�
ये��य�उपभोग�और��नवेश�पर
�म�, पूंजी�और�संगठन। गणना�के��लए�, पहले�बाजार�मू�य
�कए�जाते�ह�।�यह��व�ध�मू�य-
तदनुसार�, चार�कारक�भुगतान ( GVAmp) म��जोड़े�गए�सकल
व�ध�त��व�ध�के��वपरीत�है।�ऐसा
ह��, अथा�त्��कराया�, कम�चा�रय� मू�य�, बाजार�मू�य�( NVAmp)
इस�लए�है��य��क�मू�य-व�ध�त
का�मुआवजा�, �याज�और पर�जोड़ा�गया�शु��मू�य�और
प��त��ब���प��से�रा�ीय�आय
लाभ।��म��त�आय�का�एक�और कारक�लागत�( NVAfc) पर
का�अनुमान�लगाती�है�, जब�क
कारक�भुगतान�है।� जोड़ा�गया�शु��मू�य�क��गणना
�य�प��त�खरीद�प��से�रा�ीय
आय�क��गणना�करती�है।� करना�आव�यक�है।�

QUESTION 38.
भारत�म��कृ�ष�आय�को�कराधान�से�मु���कया�गया�है।��न�न�ल�खत�म��से�कौन�सी�आय�भारत�म��कृ�ष�आय�के�अंतग�त�नह��आती�है�?

a) भू�म�से��ा�त�कोई�भी��कराया�जो�कृ�ष��योजन�के��लए�उपयोग��कया�जाता�है।�
b) ��तरो�पत�( replanted ) वृ���क���ब���और�फूल�तथा�लताएं�उगाने�से��ा�त�आय।�
c) बीज��क���ब���से��ा�त�आय।�
d) पो����या�मधुम�खी�पालन�के�मा�यम�से�उ�प��आय।�
Correct Answer: D
Your Answer:
Explanation

Solution (d)

Explanation:

कृ�ष�भू�म�से�अ�ज�त�आय�को�आयकर�अ�ध�नयम�1961 क��धारा�10 (1) के�तहत�कर��से�छू ट�द��गई�है।�कृ�ष�आय�क��गणना��वसाय


आय�के�समान�ही�क��जाती�है।�

भारत�म��कृ�ष�आय�के��प�म��प�रभा�षत��या�है�?

• भू�म�से��ा�त�कोई�भी��कराया�जो�कृ�ष��योजन��के��लए�उपयोग��कया�जाता�है।�
• कृ�ष�संचालन��जसम��मु�य��प�से�कृ�ष�उपज�का��सं�करण�शा�मल�होता�है��जसे�तब�कृ�ष�उपज�के��प�म��बाजार�म��बेचा�जाता�है।�
• एक�फाम�हाउस�से��ा�त�आय�जो�आपके��वयं�के�जमीन�के��नकट�है।�तथा��जसका�उपयोग�भंडारगृह�या�आवास�इकाई�के��प�म���कया
जा�सकता�है।�
• यह�ब�त�आव�यक�है��क�भू�म�मौजूद�है�और�इसका�उपयोग�कृ�ष�काय��के��लए��कया�जाता�है।�कृ�ष�काय��का�अथ��है��क�बीज�लगाए
जाएं�और�भू�म�से�बीज�अंकु�रत�करने�तथा�भू�म�पर�खेती�करने�का��यास��कया�जाए।�
• हम���वचार�कर���क�आप�अपनी�भू�म�से��कराये�क��आय��ा�त�करते�ह�।�यह�ब�त�आव�यक�है��क�इस�भू�म�पर�कृ�ष�काय���कए�जाएं।
तभी�आप�यह�बता�सकते�ह���क�आपक��भू�म�से��ा�त��कराये�क��आय�कृ�ष�आय�है।�
• ��तरो�पत�( replanted) वृ���क���ब���तथा�फूल�और�लताएं�उगाने�से��ा�त�आय�
• बीज��क���ब���से��ा�त�आय।�

IASbaba
Score:
Web: http://ilp.iasbaba.com/
69.00 /
Email: ilp@iasbaba.com
Page 125 200
AIPTS/ILP VETERANS -
Exam Title :
2020 T...
Email : sahil.law.jmi@gmail.com
Contact : 9540210113

• अपने�सहभागी�से��ा�त�लाभ�जो�कृ�ष�उपज�या�ग�त�व�धय��म��लगा��आ�है।�
• पौध��और�फूल��क���ब���जैसे�नस�री�संचालन�से�अ�ज�त�आय।�

भारत�म��कृ�ष�आय�के��प�म���या�नह��माना�जाता�है�?

• आइए�हम�इस�बात�पर��वचार�कर���क�आपके�पास�मु�ग�याँ�ह��या�मधुम�खी�के�छ�े�का��बंधन।�इसे�भारत�म��कृ�ष�आय�नह��माना�जाता
है।�
• य�द�आपको�डेयरी�और�अनायास�( spontaneously) उगाए�गए�पेड़��से�आय�होती�है।�
• समु��के�पानी�क��भू�म�को�भरकर�नमक�का�उ�पादन�करके�अ�ज�त�आय।�
• खान��से�रॉय�ट��क��आय�और�खड़ी�फसल�क��खरीद।�
• म�खन�और�पनीर�बनाने�से�आय।�
• फाम�हाउस�म��ट�वी�सी�रयल�क��शू�ट�ग�को�कृ�ष�आय�नह��माना�जाता�है�और�कर�लगाया�जाता�है।�
• औ�ो�गक��े���या�स�पदा�और��ावसा�यक�भवन��और�स��जय��से��घरे�एक���त��े��के�पास���थत�भू�म�को�इस�भू�म�पर�उगाया
जाता�है�, तब�इस�भू�म�का�कृ�ष�भू�म�के��प�म��गठन�नह��होता�है�और�आय�पर�कर�लगता�है।�

QUESTION 39.
एक�अथ��व�था�म��मु�ा�क��कुल�मांग�( Total demand ) होती�है�

a) लेन-दे न�क��मांग�जो�वा�त�वक�जीडीपी�और�का�प�नक�( speculative ) मांग�के�सीधे�आनुपा�तक�है�, जो�बाजार�क���याज�दर�से


�वपरीत��प�से�संबं�धत�है।�
b) लेन-दे न�क��मांग�जो�वा�त�वक�जीडीपी�और�का�प�नक�( speculative ) मांग�के��वपरीत�आनुपा�तक�है�जो���य���प�से�बाजार
क���याज�दर�से�संबं�धत�है�
c) लेन-दे न�क��मांग�जो��याज�दर�और�का�प�नक�( speculative ) मांग�क��बाजार�दर�के�सीधे�आनुपा�तक�है�जो�वा�त�वक�जीडीपी�से
�वपरीत�है।�
d) लेन-दे न�क��मांग�जो��याज�दर�और�का�प�नक�( speculative ) मांग�क��बाजार�दर�के��वपरीत�आनुपा�तक�है�जो�सीधे�वा�त�वक
जीडीपी�से�संबं�धत�है।�
Correct Answer: A
Your Answer:
Explanation

Solution (a)

Explanation:

एक�अथ��व�था�म��मु�ा�क��कुल�मांग�लेनदे न�क��मांग�से�बनी�होती�है�जो�वा�त�वक�जीडीपी�और�का�प�नक�( speculative) मांग�के�सीधे


आनुपा�तक�होती�है�जो�बाजार�क���याज�दर�से��वपरीत�होती�है।�पहले�वाला�वा�त�वक�जीडीपी�और�मू�य��तर�के�सीधे�आनुपा�तक�है�, जब�क
बाद�वाला��याज�क��बाजार�दर�से��वपरीत�है।�

QUESTION 40.
�दवाला�और�शोधन�अ�मता�( Bankruptcy and insolvency ) एक��वषय�है��जस�पर�-

a) केवल�संसद�कानून�बना�सकती�है�
b) केवल�रा�य��वधान�सभा�कानून�बना�सकती�ह��
c) संसद�और�रा�य��वधानसभा�दोन��कानून�बना�सकती�ह��
d) संसद���ताव��के�मा�यम�से�रा�य��वधानसभा��क��सहम�त�से�कानून�बना�सकती�है�
Correct Answer: C
Your Answer:
Explanation

Solution (c)

IASbaba
Score:
Web: http://ilp.iasbaba.com/
69.00 /
Email: ilp@iasbaba.com
Page 126 200
AIPTS/ILP VETERANS -
Exam Title :
2020 T...
Email : sahil.law.jmi@gmail.com
Contact : 9540210113

Explanation/Elimination:

�दवाला�और�शोधन�अ�मता�एक�ऐसा��वषय�है��जस�पर�संसद�और�रा�य��वधानसभा�दोन��कानून�बना�सकते�ह�।�

समवत��सूची�या�सूची- III ( सातव��अनुसूची) भारत�क��सं�वधान�क��सातव��अनुसूची�म��द��गई�52 �वषय��क��सूची�है।�इसम��क���और�रा�य


सरकार�दोन���ारा��वचार�क��जाने�वाले��वषय�शा�मल�है।�

सूची�म��वत�मान�म��52 �वषय��म��शा�मल�ह��- �दवा�लयापन�और��दवाला।�

QUESTION 41.
�न�न�ल�खत�जोड़े�पर��वचार�कर�:

( एनबीएफसी�/ ग�त�व�धय��का��कार) : ( �नयामक)

1. व�चर�कै�पटल�फंड: : सेबी�
2. प�शन�फंड: : बीमा��नयामक�और��वकास��ा�धकरण�( IRDA)
3. मच�ट�ब��क�ग�कंप�नयां: : आरबीआई�
4. हाउ�स�ग�फाइन�स�कंप�नयां: : शहरी�काय��मं�ालय�

उपरो��म��से�कौन�सी�जोड़ी�सही�ढं ग�से�सुमे�लत�है�/ ह��?

a) केवल�1, 2 और�3
b) केवल�3 और�4
c) 1, 2, 3 और�4
d) केवल�1 और�2
Correct Answer: D
Your Answer:
Explanation

Solution (d)

Elimination:

तीसरी�जोड़ी�को�दे ख�।�इसे�आसानी�से�समा�त��कया�जा�सकता�है�, �य��क�एक�मं�ालय�हाउ�स�ग�फाइन�स�कंप�नय��के��लए��नयामक�सं�था


नह��हो�सकता�है।�इस�लए�, आप�सीधे�तीन��वक�प��को�ख�म�कर�सकते�ह��और�उ�र�पर�प�ंच�सकते�ह�।�

कथन��व�ेषण:

कथन�1 कथन�2 कथन�3 कथन�4

स�य� स�य� अस�य� अस�य�

IASbaba
Score:
Web: http://ilp.iasbaba.com/
69.00 /
Email: ilp@iasbaba.com
Page 127 200
AIPTS/ILP VETERANS -
Exam Title :
2020 T...
Email : sahil.law.jmi@gmail.com
Contact : 9540210113

व�चर�कै�पटल
हाउ�स�ग�फाइन�स�कंप�नय��को
फंड् स�को�सेबी IRDA �ारा�प�शन�फंड�को मच�ट�ब��क�ग�कंप�नय��को�सेबी
नेशनल�हाउ�स�ग�ब�क��ारा
�ारा��व�नय�मत �व�नय�मत��कया�जाता�है� �ारा��व�नय�मत��कया�जाता�है�
�व�नय�मत��कया�जाता�है।�
�कया�जाता�है�

�या�आप�जानते�ह��?

RBI �ारा�सभी�NBFC को��व�नय�मत�नह���कया�जाता�है।�केवल�उन�एनबीएफसी�जो�आरबीआई��ारा�प�रभा�षत�' ����सपल��बजनेस


मानदं ड�' को�पूरा�करते�ह��, वे��वयं�को�आरबीआई�के�साथ�पंजीकृत�करवाएंगे�और�इन�सं�था��को�आरबीआई��ारा��व�नय�मत�और
पय�वे�ण��कया�जाएगा।�

अ�त�र��जानकारी:

एक�कंपनी�, जो��न�न�ल�खत�दोन��मानदं ड��को�पूरा�करती�है�, को�RBI �ारा�NBFC के��प�म��पंजीकृत��कया�जाएगा।�

• " �मुख��वसाय�" के��प�म���व�ीय�ग�त�व�ध�तब�होती�है�जब��कसी�कंपनी�क���व�ीय�संप���कुल�संप���का�50% से�अ�धक�होती


है�
• �व�ीय�संप���से�आय�, सकल�आय�का�50% से�अ�धक�का�गठन�करती�है।�

QUESTION 42.
भारतीय��रज़व��ब�क�के��दशा�नद� श��के�अनुसार�, एक�ब�क�के�अपने�पास�वैधा�नक�तरलता�अनुपात�( SLR) कोटा�को�पूरा�करने�के��लए
�न�न�ल�खत�म��से�कौन�सा�घटक�हो�सकता�है�?

1. सरकारी���तभू�तयां�
2. सोना�
3. �तजोरी�क��नगद��(वॉ�ट�कैश)

ऊपर��दए�गए�कथन��म��से�कौन�सा�सही�है�/ ह��?

a) केवल�1 और�2
b) केवल�1 और�3
c) केवल�2 और�3
d) 1, 2 और�3
Correct Answer: D
Your Answer:
Explanation

Solution (d)

तीन��(सरकारी���तभू�त�, सोना�, �तजोरी�का�नकद) का�उपयोग�एसएलआर�कोटा�को�पूरा�करने�के��लए��कया�जा�सकता�है।�

अ�त�र��जानकारी:

आरबीआई��ारा�अनुसू�चत�वा�ण��यक�ब�क��के��लए�10 �दसंबर�, 2015 क��अ�धसूचना�के�अनुसार�, एसएलआर��न�नानुसार�होना�चा�हए:

• नकद�म�, या�
• सोने�म���जसका�मू�य, मौजूदा�बाजार�मू�य�से�अ�धक�नह�, या�
• �न�न�म��से��कसी�भी�उपकरण�म��अ�न�द���(Unencumbered) �नवेश, अथा�त्: -
◦ भारत�सरकार�क���दनां�कत���तभू�तयाँ�या�

IASbaba
Score:
Web: http://ilp.iasbaba.com/
69.00 /
Email: ilp@iasbaba.com
Page 128 200
AIPTS/ILP VETERANS -
Exam Title :
2020 T...
Email : sahil.law.jmi@gmail.com
Contact : 9540210113

◦ भारत�सरकार�के��े जरी��बल; या�


◦ रा�य�सरकार��के�रा�य��वकास�ऋण�(एसडीएल)
• ब��क�ग��व�नयमन�अ�ध�नयम, 1949 क��धारा�11 क��उपधारा�(2) के�तहत�भारत�के�बाहर��नग�मत�ब��क�ग�कंपनी��ारा��रज़व��ब�क
के�साथ��कए�जाने�वाले�आव�यक�जमा�और�अ�न�द���अनुमो�दत���तभू�तयाँ;
• भारतीय��रज़व��ब�क�अ�ध�नयम, 1934 क��धारा�42 के�तहत�इसके��ारा�बनाए��रज़व��ब�क�के�पास�अनुसू�चत�ब�क�के�पास��कसी
भी�शेष�रा�श�को�रखने�के��लए�आव�यकता।�

QUESTION 43.
RBI बैल�स�शीट�क��दे यता��( Liabilities ) म���न�न�ल�खत�म��से�कौन�से�स��म�लत�ह��?

1. RBI के�पास�भंडा�रत��वदे शी�मु�ा�( Foreign exchange )


2. आरबीआई�के�पास�वा�ण��यक�ब�क���ारा�रखी�गयी�नकद��तजोरी�
3. RBI के�साथ�भारत�सरकार�का�जमा�राजकोष�( Treasury deposits )
4. सरकारी���तभू�तयां�- भारतीय��रजव��ब�क��ारा�भारत�सरकार�को��दया�गया�ऋण�

नीचे��दए�गए�कूट�का�उपयोग�करके�सही�उ�र�चुन�:

a) केवल�1, 2 और�3
b) केवल�2, 3 और�4
c) केवल�1, 2 और�4
d) 1, 2, 3 और�4
Correct Answer: A
Your Answer:
Explanation

Solution (a)

Elimination:

कथन�4 कहता�है��क�सरकारी���तभू�तयाँ�(इस�मामले�म�) RBI �ारा��दए�गए�ऋण�ह��जो�यह�दशा�ता�है��क�यह�RBI क��संप���के��प�म��है।


चौथे�कथन�को�समा�त�करने�से�, केवल�एक�संभावना�(�वक�प�A) होगी�जो�सही�उ�र�है।�

Image Source: NCERT Class-12 Introductory Macroeconomics

QUESTION 44.

IASbaba
Score:
Web: http://ilp.iasbaba.com/
69.00 /
Email: ilp@iasbaba.com
Page 129 200
AIPTS/ILP VETERANS -
Exam Title :
2020 T...
Email : sahil.law.jmi@gmail.com
Contact : 9540210113

रेपो�दर�के�संबंध�म���न�न�ल�खत�कथन��पर��वचार�कर��

1. �रज़व��ब�क�, ब�क��को�रात�भर�क��तरलता��दान�करता�है�, �जस�पर�(�न��त) �याज�दर�होता�है�


2. उधार�लेने�वाले�ब�क��ारा��कसी�भी�संपा���क�( collateral ) क��आव�यकता�नह��है�
3. केवल�अनुसू�चत�वा�ण��यक�ब�क�ही�इस�सु�वधा�का�लाभ�उठा�सकते�ह��
4. उधार�लेने�वाले�ब�क�एसएलआर�कोटा���तभू�तय��का�संपा���क�के��प�म��उपयोग�कर�सकते�ह��

ऊपर��दए�गए�कथन��म��से�कौन�सा�सही�है�/ ह��?

a) केवल�1 और�2
b) केवल�1 और�3
c) केवल�1, 2 और�3
d) 1, 2, 3 और�4
Correct Answer: B
Your Answer:
Explanation

Solution (b)

Elimination:

य�द�वा�ण��यक�ब�क�आरबीआई�से�रेपो�दर�के�तहत�पैसे�उधार�लेते�ह��, तो�संपा���क�( Collaterals) क��आव�यकता�होती�है।�इस�त�य�से


कोई�भी�कथन�2 को�समा�त�कर�सकता�है�, इस��कार��वक�प�b सही�उ�र�है।�

कथन��व�ेषण:

कथन�1 कथन�2 कथन�3 कथन�4

स�य� अस�य� स�य� अस�य�

उधार�लेने�वाले�ब�क
रेपो�दर�एक��न��त�(�न��त) केवल�अनुसू�चत
संपा���क�के��प�म�
�याज�दर�है��जस�पर��रज़व��ब�क�, संपा���क�(Collateral वा�ण��यक�ब�क�ही
एसएलआर�कोटा
ब�क��को�रात�भर�क��तरलता��दान ) क��आव�यकता�है� इस�सु�वधा�का�लाभ
��तभू�तय��का�उपयोग�नह�
करता�है।� उठा�सकते�ह��
कर�सकते�ह��

अ�त�र��जानकारी:

( �न���) �याज�दर�, �जस�पर��रज़व��ब�क�, तरलता�समायोजन�सु�वधा�(एलएएफ) के�तहत�सरकार�और�अ�य�अनुमो�दत���तभू�तय��क�


संपा���क�के��व���ब�क��को�रात�भर�के��लए�तरलता��दान�करता�है।�

QUESTION 45.

IASbaba
Score:
Web: http://ilp.iasbaba.com/
69.00 /
Email: ilp@iasbaba.com
Page 130 200
AIPTS/ILP VETERANS -
Exam Title :
2020 T...
Email : sahil.law.jmi@gmail.com
Contact : 9540210113

�कसी�दे श�( RBI) के�मौ��क��ा�धकरण�क��कुल�दे यता�को�उ�च��ा�धकार��ा�त�मु�ा�( High Powered Money ) कहा�जाता�है।


उ�च��ा�धकार�वाली�मु�ा�म���न�न�शा�मल�ह��-

1. मु�ा�नोट�और��स�के�जो�जनता�के�पास��चलन�म��ह��
2. वा�ण��यक�ब�क���ारा�आरबीआई�के�पास�जमा�रा�श�
3. साव�ज�नक�ब�क��क��नकद��तजोरी�
4. भारत�सरकार��ारा�आरबीआई�के�पास�जमा�रा�श�

ऊपर��दए�गए�कथन��म��से�कौन�सा�सही�है�/ ह��?

a) केवल�1 और�3
b) केवल�2 और�4
c) केवल�1, 2 और�3
d) 1, 2, 3 और�4
Correct Answer: D
Your Answer:
Explanation

Solution (d)

Do you know?

• उ�च��ा�धकार�मु�ा�(हाई�पावड��मनी) को�मौ��क�आधार�या��रजव��मु�ा�या�आधार�मु�ा�भी�कहा�जाता�है।�
• �दए�गए�सभी��वक�प�हाई�पावड��मु�ा�का��ह�सा�ह�।�

अ�त�र��जानकारी:

�रजव��मु�ा�( M0) अथ��व�था�म��तरल�मु�ा�के��नमा�ण�का�आधार�बनता�है।�

एक�कम�(या�उ�च) ब�क�दर�आर��त�जमा�अनुपात�( rdr) को�कम�(या�बढ़ाती�है) तथा�इस�लए�मु�ा�के�गुणक�का�मू�य�(या�घटता) बढ़ता�है


, जो�( 1 + cdr) / (cdr + rdr) है।�इस��कार�, उ�च��ा�धकार�मु�ा�( H) क���कसी�भी�रा�श�के��लए�, कुल�मु�ा�क��आपू�त��बढ़�जाती
है।�

QUESTION 46.
�न�न�ल�खत�म��से�कौन�अमे�रक��डॉलर�के�मुकाबले��पये�के�मू�य�ास�का�प�रणाम�हो�सकता�है�?

a) �नया�त�आकष�क�हो�जाता�है�जब�क�आयात�एक�अनाकष�क�महंगा�मामला�बन�जाता�है�
b) आयात�आकष�क�हो�जाते�ह��जब�क��नया�त�एक�अनाकष�क�मामला�बन�जाता�है�
c) आयात�और��नया�त�दोन��अनाकष�क�हो�जाते�ह��
d) आयात�और��नया�त�दोन��आकष�क�हो�जाते�ह��
Correct Answer: A
Your Answer:
Explanation

Solution (a)

कथन��व�ेषण:

IASbaba
Score:
Web: http://ilp.iasbaba.com/
69.00 /
Email: ilp@iasbaba.com
Page 131 200
AIPTS/ILP VETERANS -
Exam Title :
2020 T...
Email : sahil.law.jmi@gmail.com
Contact : 9540210113

�वक�प�A �वक�प�B

स�य� अस�य�

�पये�के�मू�य�ास�के��प�म���नया�त�आकष�क�हो�सकता�है।� �पए�के�अ�धमु�यन�होने�पर�आयात�आकष�क�हो�सकता�है।�

एक��नया�तक�तुलना�मक��प�से�अ�धक��पये�अ�ज�त�करेगा आयात�करते�समय�एक�आयातक�तुलना�मक��प�से�कम�(�पए
य�द�वह�अ�ज�त�डॉलर�का�आदान-�दान�करता�है।� के�संदभ��म�) भुगतान�करेगा।�

अ�त�र��जानकारी:

• अ�धकांश�अंतरा��ीय��ापार�अमे�रक��डॉलर�म��होता�है।�इस�लए�, जैसा��क��पये�म���गरावट�आती�है�, �नया�त�अ�धक�लाभदायक�हो


जाता�है�, �य��क��नया�तक�डॉलर�के�आदान-�दान�के��लए�अ�धक��पये�अ�ज�त�करता�है।�
• �सरी�ओर�, आयात�महंगा�हो�जाता�है��य��क�आयातक�को�डॉलर�के�भुगतान�के��लए�अ�धक��पये�दे ने�क��आव�यकता�होती�है।�
• फामा��यु�टक�स�और�आईट��जैसे��नया�त�से�जुड़े�उ�ोग�मू�य�ास�के�साथ�लाभा��वत�होते�ह�।�
• इसी�तरह�, आयात�से�जुड़े�उ�ोग��(या�उनके�उ�पाद�के�मह�वपूण��घटक�आयात��कए�गए) को�उ�च�इनपुट�लागत�वहन�करना�पड़ता�है
, जो�अंत�म��अं�तम�उपयोगकता���को�दे ना�पड़ता�है।�

QUESTION 47.
�ापक�मु�ा�(�ॉड�मनी) और�संक�ण��मु�ा�(नैरो�मनी) के�बीच�अंतर��कस�पर�आधा�रत�है�?

a) �ापक�मु�ा�का�उपयोग�आम�तौर�पर�RBI �ारा�आम�जनता�को�धन�उधार�दे ने�के��लए��कया�जाता�है�जब�क�संक�ण��धन�का�उपयोग


क���सरकार��ारा�अपने�घाटे �के��व�पोषण�के��लए��कया�जाता�है�
b) �ापक�मु�ा�का�उपयोग�आम�तौर�पर�RBI �ारा�क���सरकार�को�पैसा�उधार�दे ने�के��लए��कया�जाता�है�, जब�क�क���य�सरकार��ारा
आम�जनता�को�पैसा�उधार�दे ने�के��लए�संक�ण��मु�ा�का�उपयोग��कया�जाता�है�
c) मु�य�अंतर�उनक��तरलता�के�बीच�है�
d) उन�दोन��म��कोई�अंतर�नह��है��य��क�दोन��एक�ही�और�समान�ह��
Correct Answer: C
Your Answer:
Explanation

Solution (c)

अ�त�र��जानकारी:

• M 1 और�M 2 को�संक�ण��मु�ा�के��प�म��जाना�जाता�है�जब�क�M 3 और�M 4 को��ापक�मु�ा�के��प�म��जाना�जाता�है।�


• M1 = जनता�के�पास�मु�ा�(नोट् स�और��स�के)+ वा�ण��यक�ब�क��के�पास�नेट��डमांड��डपॉ�जट�के��प�म��मु�ा�
• M 2 = M 1 + डाकघर�बचत�ब�क��के�पास�जमा�बचत�( Savings deposits)
• M3 = M1 + वा�ण��यक�ब�क��के�पास�साव�ध�जमा�( Net Time deposits)
• M4 = M3 + डाकघर�बचत�संगठन��के�पास�कुल�जमा�(रा�ीय�बचत�प��को�छोड़कर)
• तरलता�- M 1> M 2> M 3> M 4

QUESTION 48.
मांग�का�दोहरा�संयोग�( Double coincidence of wants ) �कसक���वशेषता�है�?

IASbaba
Score:
Web: http://ilp.iasbaba.com/
69.00 /
Email: ilp@iasbaba.com
Page 132 200
AIPTS/ILP VETERANS -
Exam Title :
2020 T...
Email : sahil.law.jmi@gmail.com
Contact : 9540210113

a) बाजार�अथ��व�था�
b) व�तु��व�नमय��णाली�
c) बाजार�अथ��व�था�और�व�तु��व�नमय��णाली�दोन��
d) इनमे�से�कोई�भी�नह��
Correct Answer: B
Your Answer:
Explanation

Solution (b)

मांग��के�दोहरे�संयोग�का�अथ��है��क�दोन��प���(खरीदार�और��व�ेता) को���येक�व�तु�को�बेचने�और�खरीदने�के��लए�सहमत�होना�होगा।�

अ�त�र��जानकारी:

यह�सम�या�उसी�समय�और�उसी��थान�पर�होने�वाले�लेन-दे न�के�कारण�, आव�यकता�, या�घटना��क��असंभवता�के�कारण�होती�है।�

QUESTION 49.
" यह�कसाई�, शराब�बनानेवाला�, रोट��वाले�के�परोपकार�से�नह��है�, �क�हम�अपने�रात�के�भोजन�क��उ�मीद�करते�ह��ब��क�उनके�अपने��हत
के�संबंध�से�है।�हम��वयं�को�उनक��मानवता�के��लए�नह��, ब��क�उनके��व-�ेम�के��लए�संबो�धत�करते�ह��, और�कभी�भी�उनसे�हमारी
आव�यकता��के�बारे�म��बात�नह��करते�ह��ब��क�उनके�लाभ�के��लए�बात�करते�ह��” ।�यह�पु�तक�" एन�इ��वायरी�इनटू �द�नेचर�एंड�कॉज
ऑफ�वे�थ�ऑफ�नेशंस�" से�एक�अंश�है�जो�दशा�ता�है�?

a) बाजार�अथ��व�था�
b) �नयं��त�अथ��व�था�( Command Economy )
c) �म��त�अथ��व�था�
d) बंद�अथ��व�था�( Closed Economy )
Correct Answer: A
Your Answer:
Explanation

Solution (a)

अ�त�र��जानकारी:

एडम���मथ�को�आधु�नक�अथ�शा���का�सं�थापक��पता�माना�जाता�है�(यह�उस�समय�राजनी�तक�अथ��व�था�के��प�म��जाना�जाता�था)।�

उनके���स��काय��“ एन�इं�वायरी�इनटू �द�नेचर�एंड�कॉज�ऑफ�वे�थ�ऑफ�नेशंस�” (1776) को�इस��वषय�पर�पहली��मुख��ापक�पु�तक


माना�जाता�है।�

पु�तक�से�उ��त�, " यह�कसाई�, शराब�बनानेवाला�, रोट��वाले�के�परोपकार�से�नह��है�, �क�हम�अपने�रात�के�भोजन�क��उ�मीद�करते�ह��ब��क


उनके�अपने��हत�के�संबंध�से�है।�हम��वयं�को�उनक��मानवता�के��लए�नह��, ब��क�उनके��व-�ेम�के��लए�संबो�धत�करते�ह��, और�कभी�भी
उनसे�हमारी�आव�यकता��के�बारे�म��बात�नह��करते�ह��ब��क�उनके�लाभ�के��लए�बात�करते�ह��” को�अ�सर�मु��बाजार�अथ��व�था�के��लए
एक�वकालत�के��प�म��उ��त��कया�जाता�है।�

QUESTION 50.
�वदे शी�मु�ा�अ�नवासी�(ब�क) खाता�( FCNR (B) खाता) के�बारे�म���न�न�ल�खत�कथन��पर��वचार�कर�।�

1. FCNR (B) खाते�को�साव�ध�जमा�(termed deposits ) के��प�म��खोला�जा�सकता�है�


2. FCNR (B) खाते�न�केवल�अ�नवासी�भारतीय��(NRI) ब��क�भारतीय��नवासी��ारा�भी�खोले�जा�सकते�ह��

ऊपर��दए�गए�कथन��म��से�कौन�सा�गलत��है�/ ह�?

IASbaba
Score:
Web: http://ilp.iasbaba.com/
69.00 /
Email: ilp@iasbaba.com
Page 133 200
AIPTS/ILP VETERANS -
Exam Title :
2020 T...
Email : sahil.law.jmi@gmail.com
Contact : 9540210113

a) केवल�1
b) केवल�2
c) 1 और�2 दोन��
d) न�तो�1 और�न�ही�2
Correct Answer: B
Your Answer:
Explanation

Solution (b)

कथन��व�ेषण:

कथन�1 कथन�2

स�य� अस�य�

FCNR (B) खाते�को�साव�ध�जमा�के��प�म��खोला�जा FCNR (B) खाते�केवल�NRI या�भारतीय�मूल�के�����( PIO)


सकता�है� �ारा�ही�खोले�जा�सकते�ह�।�

अ�त�र��जानकारी:

�वदे शी�मु�ा�गैर-�नवासी�( FCNR (B)) योजना�1975 म��शु��क��गई�त�कालीन��च�लत�FCNR (A) योजना�को�बदलने�के��लए�15 म


ई�, 1993 से�लागू�क��गई�थी�, जहां��वदे शी�मु�ा�जो�खम�RBI �ारा�वहन��कया�गया�था�और�बाद�म��भारत�क��सरकार��ारा।�

QUESTION 51.
�न�न�ल�खत�म��से�कौन�सा�' मू�य�ास�' ( Depreciation ), के�बारे�म��सही�है�?

a) यह�टू ट�- फूट�का�कुल�मू�य�है�जो�प�रसंप��य��म���आ�है।�


b) भारत�म���व��मं�ालय��नण�य�लेता�है�और�मू�य�ास�क��दर��क��घोषणा�करता�है।�
c) बाहरी��े��म��जब�उ���वदे शी�मु�ा�के�मुकाबले�घरेलू�मु�ा�का�मू�य�बढ़ता�है�तो�इसे�मू�य�ास�कहा�जाता�है।�
d) उपरो��सभी।�
Correct Answer: A
Your Answer:
Explanation

Solution (a)

Elimination:

• कथन�( a) काफ��आसान�है�, अगर�कोई�मू�य�ास�के�सामा�य�अथ��से�अवगत�है।�


• कथन�( b) काफ���ामक�है�या�हम�इस�बात�से�अवगत�नह��ह���क�मं�ालय�कौन�सा��नण�य�लेता�है।�
• कथन�( c) - य�द�हम�कुछ�सामा�य��ान�का�उपयोग�करते�ह��, तो�इस�कथन�को�आसानी�से�समा�त��कया�जा�सकता�है।��पये�/ मु�ा
मू�य�ास�(अवमू�यन) का�अथ��है�मू�य�म���गरावट�तथा�अ�धमु�यन�का�अथ��है�मू�य�म��वृ��।�इस�लए�, इस�कथन�को�समा�त�करते��ए
, हम���वक�प�( a) या�( b) के�साथ�छोड़��दया�जाता�है।�और�सुर��त�प��, आप�उ�र�के��प�म��कथन�( a) का��वक�प�चुन�सकते
ह�।�

IASbaba
Score:
Web: http://ilp.iasbaba.com/
69.00 /
Email: ilp@iasbaba.com
Page 134 200
AIPTS/ILP VETERANS -
Exam Title :
2020 T...
Email : sahil.law.jmi@gmail.com
Contact : 9540210113

कथन��व�ेषण:

कथन�1 कथन�2 कथन�3

स�य� अस�य� अस�य�

बाहरी��े��म��, य�द�घरेलू
मु�ा�का�मू�य��गरता�है�(जब
सरकार��अथ��व�था��क��उन�दर��को�तय
यह�बढ़ता�है) �वदे शी�मु�ा
करती�ह��और�उन�दर��क��घोषणा�करती�ह��, �ज
का�सामना�बाजार�तं��नह�
‘ मू�य�ास�’ प�रसंप��य��म��होने�वाले�टू ट नके��ारा�संप���का��ास�होता�है�(भारत�म�
कर�पाता�है�, तो�यह�घरेलू
- फूट�के�कुल�मू�य�से�संबं�धत�है।� वा�ण�य�और�उ�ोग�मं�ालय��ारा�, न��क��व�
मु�ा�म��' मू�य�ास�' क�
मं�ालय��ारा) तथा�एक�सूची��का�शत�क�
��येक�संप���(मनु�य�को�छोड़कर) का ��थ�त�है।�
जाती�है।�
अपने�उपयोग�क�����या�म��मू�य�ास
मु�ा�या��पया�मू�य�ास�तब
होता�है�, �जसका�अथ��है��क�वे�' टू टती�- �व�भ��प�रसंप��य��म��मू�य�ास�के�वा�त�वक
होता�है�जब��पये�का�मू�य
फूटती�ह��' । �तर�को��नधा��रत�करने�के��लए�सूची�का
घटता�है�(बढ़ता�नह��है)
उपयोग�अथ��व�था�के��व�भ��वग���ारा��कया
तथा�अ�धक��पये��वदे शी
जाता�है।�
मु�ा�क��एक�इकाई�खरीदने
के��लए�दे ने�पड़ते�ह�।�

QUESTION 52.
नेट�डोमे��टक��ोड�ट�( NDP) क��अवधारणा�के�उपयोग�के�संबंध�म���न�न�ल�खत�म��से�कौन�सा�कथन�सही�है�?

1. अनुसंधान�और��वकास�म��अथ��व�था�क��उपल��धय��को��दखाने�के��लए�मू�य�ास�( depreciation ) के��तर�म��कटौती�क�


जाती�है।�
2. इसका�उपयोग��व��क��अथ��व�था��क��तुलना�करने�के��लए�तुलना�मक��व�ेषण�म���कया�जाता�है।�

सही�उ�र�चुने:

a) केवल�1
b) केवल�2
c) दोन��1 और�2
d) न�तो�1 और�न�ही�2
Correct Answer: A
Your Answer:
Explanation

Solution (a)

Elimination/Approach:

• चूं�क�, नेट�डोमे��टक��ोड�ट�( NDP) (‘ मू�य�ास�’ के�मू�य�के�भार�को�समायो�जत�करने�के�बाद�गणना�क��गई�जीडीपी�है�, कथन


( 1) सही�लगता�है।�

IASbaba
Score:
Web: http://ilp.iasbaba.com/
69.00 /
Email: ilp@iasbaba.com
Page 135 200
AIPTS/ILP VETERANS -
Exam Title :
2020 T...
Email : sahil.law.jmi@gmail.com
Contact : 9540210113

• कथन�( 2) को�समा�त��कया�जा�सकता�है��य��क�हम�सदै व�समाचार�प���या��कसी�भी�अ�ययन�म��जीडीपी�के�बारे�म��पढ़ते�ह��जब�वे


��नया�क��अथ��व�था��क��तुलना�करते�ह�।�

NDP क��अवधारणा�के��व�भ��उपयोग�नीचे��दए�गए�ह�:

• केवल�घरेलू�उपयोग�के��लए�- अथ��व�था�के�मू�य�ास�के�कारण�नुकसान�क��ऐ�तहा�सक���थ�त�को�समझना।�इसके�अलावा�उ�ोग
और��ापार�क��तुलना�मक�अव�धय��म��मू�य�ास�क���े�ीय���थ�त�को�समझने�और��व�ेषण�करने�के��लए�उपयोग��कया�जाता�है।�
• अनुसंधान�और��वकास�के��े��म��अथ��व�था�क��उपल��धय��को��दखाने�के��लए��ज�ह�ने�ऐ�तहा�सक�समय�अव�ध�म��मू�य�ास�के
�तर�को�कम�करने�क��को�शश�क��है।�

कथन��व�ेषण:

कथन�1 कथन�2

स�य� अस�य�

एनडीपी�क��अवधारणा�का�उपयोग�अनुसंधान�और��वकास�के एनडीपी�का�उपयोग�तुलना�मक�अथ�शा���म��नह���कया�जाता�है�,
�े��म��अथ��व�था�क��उपल��धय��को��दखाने�के��लए��कया अथा�त�, �व��क��अथ��व�था��क��तुलना�करने�के��लए।�
जाता�है��ज�ह�ने�ऐ�तहा�सक�समय�अव�ध�म��मू�य�ास�के��तर
यह�मू�य�ास�क���व�भ��दर��के�कारण�है�जो��व��क���व�भ�
को�कम�करने�क��को�शश�क��है।�
अथ��व�था���ारा��नधा��रत�है।�

QUESTION 53.
सकल�घरेलू�उ�पाद�( GDP) के�बारे�म���न�न�ल�खत�कथन��पर��वचार�कर�:

1. जीडीपी�सभी�अं�तम�व�तु��और�सेवा��का�कुल�बाजार�मू�य�है।�
2. यह��कसी�दे श�क��सीमा��के�भीतर�उ�पा�दत�व�तु��क��गणना�करता�है।�
3. यह�व�तु��और�सेवा��के�गुणा�मक�पहलु��को�इं�गत�करता�है।�

उपरो��कथन��म��से�कौन�सा�गलत�है�/ ह��?

a) केवल�1 और�2
b) केवल�2 और�3
c) केवल�3
d) उपरो��सभी�
Correct Answer: C
Your Answer:
Explanation

Solution (c)

कथन��व�ेषण:

IASbaba
Score:
Web: http://ilp.iasbaba.com/
69.00 /
Email: ilp@iasbaba.com
Page 136 200
AIPTS/ILP VETERANS -
Exam Title :
2020 T...
Email : sahil.law.jmi@gmail.com
Contact : 9540210113

कथन�1 और�2 कथन�3

स�य� अस�य�

सकल�घरेलू�उ�पाद�(जीडीपी) एक�वष��के�दौरान�एक�रा��क�
सीमा�के�भीतर�उ�पा�दत�सभी�अं�तम�व�तु��और�सेवा��का यह�एक�‘ मा�ा�मक�’ अवधारणा�है�और�इसक��मा�ा�/ आकार
मू�य�है।� अथ��व�था�क��‘ आंत�रक�’ श���को�इं�गत�करता�है।�

सकल�घरेलू�उ�पाद�(जीडीपी) दे श�क��अथ��व�था�के��वा��य ले�कन�यह�अथ��व�था��ारा�उ�पा�दत�व�तु��और�सेवा��के�‘ गु


का�अनुमान�लगाने�के��लए�उपयोग��कए�जाने�वाले��ाथ�मक णा�मक�’ पहलु��के�बारे�म��कुछ�नह��कहता�है।�
संकेतक��म��से�एक�है।�

QUESTION 54.
सकल�रा�ीय�उ�पाद�( GNP) के�बारे�म���न�न�ल�खत�कथन��पर��वचार�कर�।�

1. जीएनपी�दे श�क��सीमा��के�भीतर�नाग�रक���ारा�उ�पा�दत�सभी�अं�तम�व�तु��और�सेवा��का�कुल�बाजार�मू�य�है।�
2. जीएनपी�दे श�क��जीडीपी�है�जब�इसम���वदे श��से��ा�त�आय�को�जोड़ा�जाता�है।�

गलत�उ�र�चुन�:

a) केवल�1
b) केवल�2
c) दोन��1 और�2
d) न�तो�1 और�न�ही�2
Correct Answer: A
Your Answer:
Explanation

Solution (a)

कथन��व�ेषण:

कथन�1 कथन�2

अस�य� स�य�

IASbaba
Score:
Web: http://ilp.iasbaba.com/
69.00 /
Email: ilp@iasbaba.com
Page 137 200
AIPTS/ILP VETERANS -
Exam Title :
2020 T...
Email : sahil.law.jmi@gmail.com
Contact : 9540210113

जीएनपी��कसी�रा��के�लोग���ारा�उ�पा�दत�व�तु��और
सेवा��का�कुल�मू�य�है।�
जीएनपी�एक�दे श�क��जीडीपी�है��जसम��‘ �वदे श��से��ा�त�आय�’ को
इससे�कोई�फक��नह��पड़ता��क�उस�दे श�क��भौगो�लक जोड़ा�जाता�है।�
सीमा��के�भीतर�या�बाहर�व�तु��/ सेवा��का�उ�पादन
�कया�जाता�है�या�नह�।�

QUESTION 55.
�न�न�म��से�सकल�रा�ीय�उ�पाद�के�संदभ��म��कौन�सी�/ को�' �वदे श�से�आय�' के�खंड�म���गना�जाता�है�?

1. �नजी��ेषण�( Private remittances )


2. बाहरी�ऋण��पर��याज�( Interest on external loans )
3. बाहरी�अनुदान�( External grants )

सही�उ�र�चुने:

a) केवल�1
b) केवल�1 और�2
c) केवल�1 और�3
d) उपरो��सभी�
Correct Answer: D
Your Answer:
Explanation

Solution (d)

Explanation:

सकल�रा�ीय�उ�पाद�(जीएनपी) एक�दे श�क��जीडीपी�है��जसम��‘ �वदे श��से�अपनी�आय�’ को�साथ�म��जोड़ा�जाता�है।�' �वदे श�से�आय�' खंड�म�


�गने�जाने�वाले�अवयव�ह�:

1. �ापार�संतुलन�
2. बाहरी�ऋण��का��याज�
3. �नजी��ेषण�( Private Remittances)

QUESTION 56.
तुलना�मक�अवधारणा��के��प�म��GDP और�GNP के�बारे�म���न�न�ल�खत�पर��वचार�कर�:

1. भारत�के�मामले�म��जीडीपी�सदै व�अपने�जीएनपी�से�कम�होता�है।�
2. जीडीपी�केवल�मा�ा�मक�माप�है�जब�क�जीएनपी�अथ��व�था�के�गुणा�मक�और�मा�ा�मक�पहलु��को�इं�गत�करता�है।�
3. जीएनपी�एक�अथ��व�था�क��सीमा-पार�आ�थ�क�ग�त�व�धय��क��भी�गणना�करता�है।�

सही�उ�र�चुने:

a) केवल�1 और�2
b) केवल�1 और�3
c) केवल�2 और�3
d) उपरो��सभी�

IASbaba
Score:
Web: http://ilp.iasbaba.com/
69.00 /
Email: ilp@iasbaba.com
Page 138 200
AIPTS/ILP VETERANS -
Exam Title :
2020 T...
Email : sahil.law.jmi@gmail.com
Contact : 9540210113

Correct Answer: C
Your Answer:
Explanation

Solution (c)

Elimination:

• य�द�आप�GDP और�GNP क��प�रभाषा�से�अवगत�ह��, तो�कथन�1 को�आसानी�से�समा�त��कया�जा�सकता�है।�(ता�लका�म��नीचे


समझाया�गया�है)

कथन��व�ेषण:

कथन�1 कथन�2 कथन�3

अस�य� स�य� स�य�

भारत�के�मामले�म��, जीएनपी�अपने
सकल�घरेलू�उ�पाद�क��तुलना�म��हमेशा
कम�है।�

सामा�य�सू��GNP = GDP + �वदे श जीएनपी�जीडीपी�क��तुलना�म��रा�ीय�आय


से�आय�है।� क��अ�धक��ापक�अवधारणा�है��य��क�यह
जीएनपी�को�मापने�के�दौरान�, एक
अथ��व�था�के�’ मा�ा�मक�’ और�साथ�ही�‘
ले�कन�यह�भारत�के�मामले�म��जीएनपी अथ��व�था�क��सीमा-पार�आ�थ�क
गुणा�मक�’ पहलू�के�बारे�म��भी�इं�गत�करता
= जीडीपी�+ (- �वदे श�से�आय�) = ग�त�व�धय��को�भी��यान�म��रखा�जाता�है।�
है�, अथा�त�, ‘ आंत�रक�’ और�साथ�ही
जीडीपी�- �वदे श�से�आय�हो�जाती�है।� अथ��व�था�क��‘ बाहरी�’ श��।�

इसका�अथ��यह�है��क�भारत�का�जीएनपी
अपने�सकल�घरेलू�उ�पाद�से�हमेशा�कम
होता�है।�

QUESTION 57.
कौन�से�/ को�' अं�तम�उपाय�का�ऋणदाता�' माना�जाता�है�?

a) अनुसू�चत�ब�क�
b) वा�ण��यक�ब�क�
c) नाबाड��
d) भारतीय��रजव��ब�क�
Correct Answer: D
Your Answer:
Explanation

Solution (d)

Explanation:

IASbaba
Score:
Web: http://ilp.iasbaba.com/
69.00 /
Email: ilp@iasbaba.com
Page 139 200
AIPTS/ILP VETERANS -
Exam Title :
2020 T...
Email : sahil.law.jmi@gmail.com
Contact : 9540210113

भारतीय��रजव�
अनुसू�चत�ब�क� वा�ण��यक�ब�क� नाबाड��
ब�क�

एक�क���य�ब�क�को
अलग-अलग�दे श�
म��अलग-अलग
नाम��से�जाना�जाता
है।�

भारत�म��, RBI
को�' अं�तम�उपाय
का�ऋणदाता�' मा
अनुसू�चत�ब�क: एक�ब�क�को
ना�जाता�है।�
संद�भ�त�करता�है�जो�भारतीय
�रज़व��ब�क�अ�ध�नयम�, वा�ण��यक�ब�क��के��वपरीत�,
वा�ण��यक�ब�क�वे�ब�क�ह��जो जब�एक
1934 क���सरी�अनुसूची�म� नाबाड��जैसे��वकास�ब�क��के
सीधे��ाहक��के�साथ�काय��कर�रहे वा�ण��यक�ब�क
सूचीब��है।� पास�जमा�या�ऋण�खाते�नह�
ह��- जमा�खाते�खोलना�, ऋण �व�ीय�संकट�का
होते�ह��तथा�वे�केवल��ाहक��से
दे ना�, �डमांड��ा�ट�जारी�करना�, सामना�करता�है
इस�अनुसूची�के�अंतग�त�नह� �ा�त�ऋण���ताव��क�����या
ऑनलाइन��ेषण�भेजना�आ�द�, और�अ�य��ोत��से
आने�वाले�ब�क��को�गैर- कर�गे�, ऋण��को�मंजूरी�द� गे
तथा��ाहक�इन�ब�क��से�संपक� धन��ा�त�करने�म�
अनुसू�चत�ब�क�कहा�जाता�है। और�रा�श�का�भुगतान�SBI जै
कर�सकते�ह��और�अपनी�सेवाएँ �वफल�रहता�है�तो
से�वा�ण��यक�ब�क�के�मा�यम
अनुसू�चत�ब�क�आमतौर�पर पूरी�कर�सकते�ह�।� क���य�ब�क�अं�तम
से��कया�जाएगा।�
भारत�म��संचा�लत��नजी�, �व उपाय�के�ऋणदाता
दे शी�और�रा�ीयकृत�ब�क�ह�।� क��मह�वपूण�
भू�मका��नभाता�है।
यह�ऋण�के��प�म�
�व�ीय�सहायता
�दान�करता�है।�

क���य�ब�क�क��यह
भू�मका�वा�ण��यक
ब�क�को��दवा�लया
होने�से�बचाती�है।�

QUESTION 58.
�व�नमय�दर��से�संबं�धत��न�न�ल�खत�मामल��पर��वचार�कर��तथा�सही�ढं ग�से��मलान��कए�गए�जोड़े�क��पहचान�कर�:

1. लचीली��व�नमय�दर:: यह�बाजार�क��मांग�और�आपू�त��क��श��य���ारा��नधा��रत�क��जाती�है।�
2. नॉ�मनल��व�नमय�दर:: �व�नमय�दर��को�मु�ा�के�संदभ��म��उ��त��कया�जाता�है।�
3. वा�त�वक��व�नमय�दर:: समान�मु�ा�का�घरेलू�से��वदे शी�मू�य��के�अनुपात�का�मापन।�

उ�चत�उ�र�चुन�:

a) 1 और�2
b) 2 और�3
c) 1 और�3
d) उपरो��सभी�

IASbaba
Score:
Web: http://ilp.iasbaba.com/
69.00 /
Email: ilp@iasbaba.com
Page 140 200
AIPTS/ILP VETERANS -
Exam Title :
2020 T...
Email : sahil.law.jmi@gmail.com
Contact : 9540210113

Correct Answer: A
Your Answer:
Explanation

Solution (a)

कथन��व�ेषण:

कथन�1 कथन�2 कथन�3

स�य� स�य� अस�य�

लचीली��व�नमय�दर��को��लो�ट�ग नॉ�मनल��व�नमय�दर�वह�दर�है��जस�पर
वा�त�वक��व�नमय�दर�- समान�मु�ा�के��लए
�व�नमय�दर�के��प�म��भी�जाना�जाता�है एक�मु�ा�का��सरे�के�साथ��व�नमय��कया
घरेलू�से��वदे शी�क�मत��का�अनुपात।�
जो�बाजार�क��मांग�और�आपू�त��क� जाता�है।�
श��य���ारा��नधा��रत�होती�है।� यह�मौजूदा��व�नमय�दर��और�क�मत��पर
इसे��कसी�अ�य�मु�ा�के�संबंध�म��एक�दे श
�कसी�अ�य�के�सापे��मु�ा�क���य�श���है।
पूरी�तरह�से�लचीली��णाली�म��, क���य क��मु�ा�के�मू�य�के��प�म��भी�माना�जाता
यह��कसी�दे श�क��मु�ा�क��इकाइय��क��सं�या
ब�क��नयम��के�एक�सरल�सेट�का�पालन है।�
का�अनुपात�है�जो��कसी�अ�य�दे श�क��व�तु�
करते�ह��- वे�सीधे��व�नमय�दर�के��तर क��माक�ट�बा�केट�को�खरीदने�के��लए
इसे���प�ीय�नॉ�मनल��व�नमय�दर�के��प
को��भा�वत�करने�के��लए�कुछ�नह� आव�यक�है�, �वदे शी�मु�ा�बाजार�म���सरे�दे श
म��भी�कहा�जाता�है�, इस�अथ��म���क�वे�एक
करते�ह��, �सरे�श�द��म��वे��वदे शी�मु�ा क��मु�ा��ा�त�करने�के�बाद�, �दए�गए�दे श�क�
मु�ा�के��लए��सरे�के��व����व�नमय�दर�ह�
बाजार�म��ह�त�ेप�नह��करते�ह��(और मु�ा�क��इकाइय��क��सं�या�के��लए।��दए�गए
तथा�वे�नॉ�मनल�ह���य��क�वे��व�नमय�दर
इस�लए�, कोई�आ�धका�रक�लेन-दे न दे श�म��सीधे�उस�माक�ट�बा�केट�को�खरीदना
को�मु�ा�के�संदभ��म��उ��त�करते�ह��, यानी
�रजव��नह��होते�ह�)।� आव�यक�होगा।�
��त�डॉलर�या���त�पाउंड�इतने��पये।�

QUESTION 59.
भारत�से��नया�त�सु�न��त�करने�के��लए�कौन�सा�संगठन�है�?

a) ए��जम�ब�क�
b) ईसीजीसी�
c) वा�णज़़य़�मं�ालय़�
d) भारतीय��रजव��ब�क�
Correct Answer: B
Your Answer:
Explanation

Solution (b)

Explanation:

ईसीजीसी��ल�मटे ड�(भारत�सरकार�के�पूव���नया�त�ऋण�गारंट���नगम) को�, भारत�सरकार�के��वा�म�व�म��, 1957 म���े�डट�जो�खम�बीमा�और


�नया�त�के��लए�संबं�धत�सेवाएं��दान�करके�दे श�से��नया�त�को�बढ़ावा�दे ने�के�उ�े �य�से��था�पत��कया�गया�था।�

IASbaba
Score:
Web: http://ilp.iasbaba.com/
69.00 /
Email: ilp@iasbaba.com
Page 141 200
AIPTS/ILP VETERANS -
Exam Title :
2020 T...
Email : sahil.law.jmi@gmail.com
Contact : 9540210113

यह�वा�ण�य�और�उ�ोग�मं�ालय�के��शास�नक��नयं�ण�म��काम�करता�है�, तथा�एक��नदे शक�मंडल��ारा��बं�धत�होता�है��जसम��सरकार�, भार


तीय��रजव��ब�क�, ब��क�ग�और�बीमा�एवं��नया�त�समुदाय�के���त�न�ध�शा�मल�होते�ह�।�

इन�वष��म��इसने�भारतीय��नया�तक��और�वा�ण��यक�ब�क��क���नया�त�ऋण�क��आव�यकता��के�अनु�प��व�भ���नया�त�ऋण�जो�खम�बीमा
उ�पाद�तैयार��कए�ह�।�

ईसीजीसी�मूल��प�से�एक��नया�त��ो�साहन�संगठन�है�, जो�भारतीय��नया�तक��को�ऋण�बीमा�कवर��दान�करके�उनक����त�पधा��मकता�म�
सुधार�करता�है।�ईसीजीसी�अपनी��ी�मयम�दर��को�इ�तम��तर�पर�रखता�है।�

अ�त�र��जानकारी�

ECGC �या�करता�है�?

• �नया�तक��को�व�तु��और�सेवा��के��नया�त�म��नुकसान�के��व����े�डट�जो�खम�बीमा�कवर�क��एक��ृंखला��दान�करता�है�
• �नया�त��े�डट�इं�योर�स�ब�क��और��व�ीय�सं�थान��को�कवर�करता�है�ता�क��नया�तक��को�उनसे�बेहतर�सु�वधाएं��मल�सक��
• इ��वट��या�ऋण�के��प�म���वदे श��म��संयु��उ�म��म���नवेश�करने�वाली�भारतीय�कंप�नय��को��वदे शी��नवेश�बीमा��दान�करता�है�

ईसीजीसी��नया�तक��क��मदद�कैसे�करता�है�?

• �नया�तक��को�भुगतान�जो�खम��के��व���बीमा�सुर�ा��दान�करता�है�
• �नया�त-संबं�धत�ग�त�व�धय��म��माग�दश�न��दान�करता�है�
• �व�भ��दे श��के�बारे�म��जानकारी�उपल�ध�कराता�है��जसके�पास��वयं�क���े�डट�रे�ट�ग�है�
• ब�क��/ �व�ीय�सं�थान��से��नया�त��व���ा�त�करना�आसान�बनाता�है�
• �नया�तक��को�खराब�ऋण��क��वसूली�म��सहायता�करता�है�
• �वदे शी�खरीदार��क��ऋण-यो�यता�पर�जानकारी��दान�करता�है�

QUESTION 60.
भारत�म��रा�ीय�आय�का�अनुमान��न�न��ारा�तैयार��कया�जाता�है:

a) �व��मं�ी�
b) रा�ीय�नमूना�सव��ण�काया�लय�
c) क���य�सां��यक��काया�लय�
d) जीडीपी�सव��ऑफ�इं�डया�
Correct Answer: C
Your Answer:
Explanation

Solution (c)

Explanation:

क���य�सां��यक��काया�लय�, जो�रा�ीय�सां��यक�य�संगठन�( NSO) के�दो�अंग��म��से�एक�है�, दे श�म��सां��यक�य�ग�त�व�धय��के�सम�वय


और�सां��यक�य�मानक��को��वक�सत�करने�तथा�बनाए�रखने�के��लए�उ�रदायी�है।�इसक��ग�त�व�धय��म��रा�ीय�खात��का�संकलन�भी�शा�मल
है�; उ�ोग��और�आ�थ�क�स�सर�का�वा�ष�क�सव��ण�, औ�ो�गक�उ�पादन�सूचकांक�, साथ�ही�उपभो�ा�मू�य�सूचकांक�का�संकलन।�यह��व�भ�
सामा�जक�सां��यक��, ��श�ण�, अंतरा��ीय�सहयोग�, औ�ो�गक�वग�करण�आ�द�से�भी�संबं�धत�है।�

CSO का�नेतृ�व�एक�महा�नदे शक��ारा��कया�जाता�है�, �जसे�रा�ीय�लेखा��भाग�, सामा�जक�सां��यक���भाग�, आ�थ�क�सां��यक���भाग�,


��श�ण��भाग�तथा�सम�वय�और��काशन��भाग�क��दे खभाल�करने�वाले�5 अ�त�र��महा�नदे शक���ारा�सहायता��दान�क��जाती�है।�

QUESTION 61.
भारत�म��ब��क�ग�लोकपाल�( Ombudsman ) क��सं�था�के�संदभ��म��नीचे��दए�गए�कथन��पर��वचार�कर��और�गलत�कथन�क��पहचान�कर�:

IASbaba
Score:
Web: http://ilp.iasbaba.com/
69.00 /
Email: ilp@iasbaba.com
Page 142 200
AIPTS/ILP VETERANS -
Exam Title :
2020 T...
Email : sahil.law.jmi@gmail.com
Contact : 9540210113

a) ब��क�ग�लोकपाल��ारा�पा�रत�आदे श�अं�तम�नह��ह��और�संबं�धत�प���के��लए�बा�यकारी�नह��ह�।�
b) ब��क�ग�लोकपाल�भारत�म��अ�नवासी�भारतीय��के�खात��क���शकायत��को��ा�त�कर�सकता�है�तथा�उन�पर��वचार�कर�सकता�है।�
c) ब��क�ग�लोकपाल�आरबीआई��ारा��नयु���कया�जाता�है।�
d) ब��क�ग�लोकपाल��ारा��दान�क��जाने�वाली�सेवाएं��भाय��(शु�क�यो�य) ह�।�
Correct Answer: D
Your Answer:
Explanation

Solution (d)

कथन��व�ेषण:

कथन�( a) कथन�( b) कथन�( c) कथन�( d)

स�य� स�य� स�य� अस�य�

ब��क�ग�लोकपाल�इंटरनेट�ब��क�ग
स�हत�ब��क�ग�काय��से�संबं�धत�कई
क�मय��से�संबं�धत��कसी�भी ब��क�ग�लोकपाल�भारतीय��रज़व�
�शकायत�को��ा�त�कर�सकता�है ब�क��ारा��नयु��एक�व�र�
ब��क�ग�लोकपाल�( Ombuds और�उस�पर��वचार�कर�सकता�है। अ�धकारी�है।�
ब��क�ग�लोकपाल��ारा��दान
man) �ारा�पा�रत�आदे श (अ�नवासी�भारतीय��से�स�हत)
उसके�पास�कुछ�ब��क�ग�सेवा� क��गई�सेवा��कसी�भी�शु�क
अं�तम�नह��ह��और�संबं�धत�प��
RBI ने�ब�क���ारा��ाहक��के��लए म��कमी�के��व����ाहक��क� से�मु��है।�
के��लए�बा�यकारी�नह��ह�।�
बड़ी�सं�या�म��सेवा�क�मय��का �शकायत��के��नवारण�क�
उ�लेख��कया�है�जहां��ाहक�( NR �ज�मेदारी�है।�
I स�हत) �शकायत�के�मा�यम�से
लोकपाल�से�संपक��कर�सकते�ह�।�

QUESTION 62.
�कसी�दे श�के�सकल�घरेलू�उ�पाद�( GDP) को�' सकल�' कहा�जाता�है��य��क�इसम��शा�मल�नह���कया�जाता�है�

a) म�य�थ�उ�पाद��को�
b) पूंजीगत�मू�य�ास�को�
c) बाहर�काम�करने�वाले�भारतीय�
d) �ाथ�मक�ग�त�व�धय���ारा�उ�प��आय�
Correct Answer: B
Your Answer:
Explanation

Solution (b)

Explanation:

• जीडीपी�म��पूंजीगत�व�तु��का�मू�य�ास�शा�मल�नह��है।�

IASbaba
Score:
Web: http://ilp.iasbaba.com/
69.00 /
Email: ilp@iasbaba.com
Page 143 200
AIPTS/ILP VETERANS -
Exam Title :
2020 T...
Email : sahil.law.jmi@gmail.com
Contact : 9540210113

• �नवल�घरेलू�उ�पाद�(एनडीपी) दे श�के�पूंजीगत�व�तु��पर�सकल�घरेलू�उ�पाद�(जीडीपी) म��से�मू�य�ास�के�घटाने�के�बाद�बराबर�होता


है।�

QUESTION 63.
�न�न�ल�खत�म��से�कौन�सा��ामीण�प�रवार��को���य��ऋण�अनुदान�सहायता��दान�करता�है�?

1. �े�ीय��ामीण�ब�क�
2. रा�ीय�कृ�ष�और��ामीण��वकास�ब�क�
3. भू�म��वकास�ब�क�

नीचे��दए�गए�कूट�का�उपयोग�करके�सही�उ�र�चुन�:

a) केवल�1 और�2
b) केवल�2
c) केवल�1 और�3
d) 1, 2 और�3
Correct Answer: C
Your Answer:
Explanation

Solution (c)

�व�ेषण:

�वक�प�1 और�3 �वक�प�2

स�य� अस�य�

नाबाड��" ��य��" ऋण�सहायता�नह��दे ता�है।�यह�म�य�थ�जैसे


�े�ीय��ामीण�ब�क�और�भू�म��वकास�ब�क��ामीण�प�रवार�
माइ�ोफाइन�स�कंप�नय��, सहकारी�स�म�त�, आरआरबी�के�मा�यम�से
को���य��ऋण�सहायता��दान�करते�ह�।�
ऋण��दान�करता�है।�

QUESTION 64.
�े�ीय��ामीण�ब�क��(आरआरबी) के�संबंध�म���न�न�ल�खत�म��से�कौन�सा�कथन�सही�है�/ ह��?

1. RRB को�रा�ीय�कृ�ष�और��ामीण��वकास�ब�क�( NABARD) �ारा��व�नय�मत��कया�जाता�है�


2. आरआरबी�केवल��ामीण��े���म��काम�करते�ह�।�

उपयु��कूट�चुन�:

a) केवल�1
b) केवल�2
c) दोन��1 और�2
d) न�तो�1 और�न�ही�2
Correct Answer: A

IASbaba
Score:
Web: http://ilp.iasbaba.com/
69.00 /
Email: ilp@iasbaba.com
Page 144 200
AIPTS/ILP VETERANS -
Exam Title :
2020 T...
Email : sahil.law.jmi@gmail.com
Contact : 9540210113

Your Answer:
Explanation

Solution (a)

कथन��व�ेषण:

कथन�1 कथन�2

स�य� अस�य�

�े�ीय��ामीण�ब�क��को�रा�ीय�कृ�ष�और��ामीण��वकास�ब�क�( �े�ीय��ामीण�ब�क�- उनके�प�रचालन��े��म��शहरी��े��भी�शा�मल


NABARD) �ारा��व�नय�मत��कया�जाता�है।� हो�सकते�ह�।�

अ�त�र��जानकारी:

�े�ीय��ामीण�ब�क�( RRB)

• �े�ीय��ामीण�ब�क�भारत�के��व�भ��रा�य��म���े�ीय��तर�पर�संचा�लत�वा�ण��यक�ब�क�(सरकारी�ब�क) ह�।�
• वे�मूल�ब��क�ग�और��व�ीय�सेवा��के�साथ�मु�य��प�से�भारत�के��ामीण��े���क��सेवा�करने�के�उ�े �य�से�बनाए�गए�ह�।�हालाँ�क�, आर
आरबी�क��शहरी�प�रचालन�के��लए�शाखाएँ�हो�सकती�ह��और�उनके�संचालन�के��े��म��शहरी��े��भी�शा�मल�हो�सकते�ह�।�
• आरआरबी�के�संचालन�का��े��रा�य�के�एक�या�अ�धक��जल��को�कवर�करने�वाले�भारत�सरकार��ारा�अ�धसू�चत��े��तक�सी�मत�है।
आरआरबी��व�भ���कार�के�काय��भी�करते�ह�।�

RRB �न�न�ल�खत�काय��म���व�भ��काय��करते�ह�:

• �ामीण�और�अध�-शहरी��े���म��ब��क�ग�सु�वधाएं��दान�करना।�
• सरकारी�काय��को�पूरा�करना�जैसे��क�मनरेगा�मज�र��क��मज�री�का��वतरण�, प�शन�का��वतरण�आ�द।�
• लॉकर�सु�वधा�, डे�बट�और��े�डट�काड��जैसी�पैरा-ब��क�ग�सु�वधाएं��दान�करना।�
• �ामीण�ब�क��के�पास��े�ीय��ामीण�ब�क�अ�ध�नयम�1976 का�वैधा�नक�समथ�न�है।�इस�अ�ध�नयम�ने�सरकार�को�समय-समय�पर
ब�क��क���थापना�करने�क��अनुम�त�द��, जहाँ�भी�इसे�आव�यक�माना�गया।�

�या�आप�जानते�ह��?

RRB के�पास�तीन�संबं�धत�सं�था��के�साथ�उनके�संबं�धत�शेयर�ह�:

• क���सरकार�→ 50%
• रा�य�सरकार�→ 15%
• �ायोजक�ब�क�→ 35%

QUESTION 65.
�न�न�ल�खत�म��से�कौन�सा�श�द�“ ब�का�युर�स�” (Bancassurance), श�द�को�सही�ढं ग�से�समझाता�है�?

a) यह�एक�वा�ण��यक�ब�क�म��जमा�रा�श�क��मांग�के��लए��दान��कया�गया�बीमा�है।�
b) यह�एक�रा�ीय�ब�क�के��ाहक��को�, RBI �ारा�, ब�क�म���ाहक��का��व�ास�जगाने�के��लए��दया�गया�आ�ासन�है।�
c) यह�ब�क��ारा�जीवन�बीमा�और�अ�य�बीमा�उ�पाद��तथा�सेवा��क���ब���को�संद�भ�त�करता�है।�

IASbaba
Score:
Web: http://ilp.iasbaba.com/
69.00 /
Email: ilp@iasbaba.com
Page 145 200
AIPTS/ILP VETERANS -
Exam Title :
2020 T...
Email : sahil.law.jmi@gmail.com
Contact : 9540210113

d) इनमे�से�कोई�भी�नह��
Correct Answer: C
Your Answer:
Explanation

Solution (c)

Explanation:

ब�का�युर�स�(Bancassurance) का�अथ��है�ब�क��के�मा�यम�से�बीमा�उ�पाद�बेचना।�ब�क�और�बीमा�कंपनी�एक�साझेदारी�म��आते�ह���जसम�
ब�क�अपने��ाहक��को�बीमा�कंपनी�के�बीमा�उ�पाद�बेचता�है।�

ब�का�युर�स��व�था�से�दोन��कंप�नय��को�लाभ�होता�है।�एक�ओर�, ब�क�बीमा�कंपनी�से��याज�आय�के�अलावा�शु�क�रा�श�(गैर��याज�आय)
अ�ज�त�करता�है�जब�क��सरी�ओर�, बीमा�फम��अपने�बाजार�प�ंच�और��ाहक��को�बढ़ाती�है।�ब�क�एक�म�य�थ�के��प�म��काम�करता�है�, �जस
से�बीमा�कंपनी�को�अपने�बाजार�म���ह�सेदारी�बढ़ाने�के��लए�अपने�ल��त��ाहक�तक�प�ंचने�म��मदद��मलती�है।�

QUESTION 66.
अंतरा��ीय�मु�ा�कोष�( IMF) �कसके�आधार�पर�रा���क��' रा�ीय�आय�' क��र�क��नकालता�है:

a) सकल�घरेलू�उ�पाद�( GDP)
b) सकल�रा�ीय�उ�पाद�( GNP)
c) �य�श���समानता�के�साथ�समायो�जत�सकल�रा�ीय�उ�पाद�
d) �य�श���समानता�के�साथ�समायो�जत�सकल�घरेलू�उ�पाद�
Correct Answer: D
Your Answer:
Explanation

Solution (d)

Explanation:

IMF के�अनुसार�, �य�श���समता�( PPP) ��येक�रा��म��जीवन��तर�के��लए�रा�ीय�आय�को�समायो�जत�करती�है�ता�क�रा�ीय�संप���का


सट�क�अनुमान�लगाया�जा�सके।�

वत�मान�म��यूएसए�और�जापान�के�बाद�आईएमएफ��ारा�पीपीपी�के��लए�समायो�जत�जीडीपी�के�संदभ��म��भारत�को�तीसरी�सबसे�बड़ी
अथ��व�था�के��प�म���थान��दया�गया�है।�

QUESTION 67.
�न�न�ल�खत�म��से��कसे�' उ�पादन�के�कारक�' ( factors of production ) कहा�जा�सकता�है�?

1. मानव��म�
2. भू�म�
3. उपभो�ा�व�तु��
4. पूंजीगत�व�तुए�ं

नीचे��दए�गए�कूट�का�उपयोग�करके�सही�उ�र�चुन�।�

a) केवल�1, 2 और�3
b) 2, 3 और�4 ही�
c) केवल�1

IASbaba
Score:
Web: http://ilp.iasbaba.com/
69.00 /
Email: ilp@iasbaba.com
Page 146 200
AIPTS/ILP VETERANS -
Exam Title :
2020 T...
Email : sahil.law.jmi@gmail.com
Contact : 9540210113

d) केवल�1, 2 और�4
Correct Answer: D
Your Answer:
Explanation

Solution (d)

Explanation:

�वक�प�1 और�2 �वक�प�3 �वक�प�4

स�य� अस�य� स�य�

उपभो�ा�व�तुए�ं , तैयार�उ�पाद�ह�।�वे�उ�पादन पूँजीगत�व�तुए�ँ ऐसी�मशीन��ह���जनका


मानव��म�और�भू�म�उ�पादन�म�
के�कारक�नह��ह��, ब��क�उ�पाद�ह�।�तो�, कथन उपयोग�अ�य�व�तु��के�उ�पादन�के��लए
योगदान�करते�ह�।�
3 गलत�है।� �कया�जाता�है।�तो�, कथन�4 भी�सही�है।�

अ�त�र��जानकारी:

• उ�पादन�के�कारक��म��भू�म�, �म�, पूंजी�और�उ�मशीलता�शा�मल�ह�।�


• पूंजी�को��न��त�(मशीन��आ�द) और�काय�शील�पूंजी�(पैसा�, क�चा�माल�आ�द) म���वभा�जत��कया�जा�सकता�है।�

QUESTION 68.
भारतीय��रजव��ब�क��न�न�ल�खत�म��से��कस�तरीके�से�ब�क��के�कामकाज�का�पय�वे�ण�करता�है�?

1. RBI ब�क��पर�नज़र�रखता�है�ता�क�वे��नधा��रत�नकद��संतुलन�बनाए�रख�।�
2. RBI अपने�अ�धदे श��ारा�ब�क��को�अ�य�धक�लाभ�कमाने�से�रोकता�है।�
3. आरबीआई�अनौपचा�रक��े��म��ऋणदाता��क��ऋण�ग�त�व�धय��क���नगरानी�करता�है।�
4. RBI यह�सु�न��त�करता�है��क�समावेशी��वकास�को��ा�त�करने�के��लए�ब�क�ऋण��को��ाथ�मकता�वाले��े���क��ओर�मोड़��दया
जाए।�

नीचे��दए�गए�कूट�का�उपयोग�करके�सही�उ�र�चुन�।�

a) केवल�2, 3 और�4
b) केवल�2 और�3
c) केवल�1 और�4
d) 1, 2, 3 और�4
Correct Answer: C
Your Answer:
Explanation

Solution (c)

कथन��व�ेषण:

IASbaba
Score:
Web: http://ilp.iasbaba.com/
69.00 /
Email: ilp@iasbaba.com
Page 147 200
AIPTS/ILP VETERANS -
Exam Title :
2020 T...
Email : sahil.law.jmi@gmail.com
Contact : 9540210113

कथन�1 कथन�2 कथन�3 कथन�4

स�य� अस�य� अस�य� स�य�

इसी�तरह�आरबीआई�दे खता�है��क
ब�क�न�केवल�लाभ�कमाने�वाले
ब�क�अपनी��ा�त�होने�वाली कोई�भी�संगठन�नह��है�जो �वसाय��और��ापा�रय��को�, बल्
जमा�रा�शय��म��से��यूनतम कथन�2 ब�त�चरम�है।� अनौपचा�रक��े��म��ऋणदाता� �क�छोटे �कृषक��, लघु�उ�ोग��, छो
नकद��शेष�बनाए�रखते�ह�।� क��ऋण�ग�त�व�धय��का टे �उधारकता���आ�द�को�भी�ऋण
ऐसा�कोई�जनादे श�नह��है�जो पय�वे�ण�करता�है।� दे ते�ह�।�
RBI वा�तव�म��नकद� ब�क��को�अ�य�धक�मुनाफा
संतुलन�बनाए�रखने�के��लए कमाने�से�रोकता�है।� वे�मनमानी��याज�दर��पर�उधार समय-समय�पर�, ब�क��को�RBI
ब�क��क���नगरानी�करता�है।� दे ते�ह�।� को�इस�बात�क��जानकारी�दे नी�होती
है��क�वे��कतना�, �कसको�, �कस
�याज�दर�पर�आ�द�, उधार�दे �रहे�ह��

QUESTION 69.
अथ��व�था�के�संदभ��म��, जब��या�होता�है�तब��टै ग�लेशन�होता�है:

1. आ�थ�क�वृ���म���गरावट�
2. उ�च�बेरोजगारी�
3. उ�च�मु�ा�फ��त�

नीचे��दए�गए�कूट�का�उपयोग�करके�सही�उ�र�चुन�।�

a) केवल�1 और�2
b) केवल�1 और�3
c) केवल�2 और�3
d) 1, 2 और�3
Correct Answer: D
Your Answer:
Explanation

Solution (d)

Approach/explanation:

�टै ग�लेशन�धीमी�आ�थ�क�वृ���और�अपे�ाकृत�उ�च�बेरोजगारी�- आ�थ�क�ठहराव�- बढ़ती�क�मत��, या�मु�ा�फ��त�, और�सकल�घरेलू�उ�पाद


(जीडीपी) म���गरावट�के�साथ�क����थ�त�है।�

इस�त�य�से��नपटने�के��लए�सरकार��के��लए��टै ग�लेशन�एक��वशेष��प�से�क�ठन�सम�या�सा�बत�हो�सकती�है�, �य��क�कम�मु�ा�फ��त�के


�लए��डज़ाइन�क��गई�अ�धकांश�नी�तयां�बेरोजगार��के��लए�क�ठन�होती�ह��, तथा�बेरोजगारी�को�कम�करने�के��लए��डज़ाइन�क��गई�नी�तयां
मु�ा�फ��त�को�बढ़ाती�ह�।�

QUESTION 70.
�न�न�ल�खत�म��से�कौन�सा�मु�ा�फ��त�के�संदभ��म��गलत�तरीके�से�मेल�खाता�है�?

IASbaba
Score:
Web: http://ilp.iasbaba.com/
69.00 /
Email: ilp@iasbaba.com
Page 148 200
AIPTS/ILP VETERANS -
Exam Title :
2020 T...
Email : sahil.law.jmi@gmail.com
Contact : 9540210113

a) कोर�मु�ा�फ��त�- भोजन�और��धन�स�हत�सभी�व�तु��और�सेवा��म��मू�य�वृ���शा�मल�होती�है।�
b) हेडलाइन�इ��लेशन�- व�तु��और�सेवा��क��क�मत��म��लगातार�वृ���शा�मल�होती�है�, �जसम��खा��और�ऊजा��क��क�मत��जैसे�व�तुएं
भी�शा�मल�ह�।�
c) संरचना�मक�मु�ा�फ��त�- सरकार�क��मौ��क�नी�त�के�कारण�अथ��व�था�म���न�म�त�मु�ा�फ��त�
d) उपरो��सभी�
Correct Answer: A
Your Answer:
Explanation

Solution (a)

कथन��व�ेषण:

कथन�( a) कथन�( b) कथन�( c)

अस�य� स�य� स�य�

व�तु�क��आपू�त��म��कमी�, �वकासशील
अथ��व�था�म��एक�सामा�य�संकट�, बढ़ती
मांग�ले�कन�व�तु��के�आव�यक��तर�का
हेडलाइन�मु�ा�फ��त�एक�अथ��व�था�के उ�पादन�करने�के��लए��नवेश�यो�य�पूंजी�क�
भीतर�कुल�मु�ा�फ��त�का�एक�उपाय�है�, �ज कमी�आ�द�के�कारण�संरचना�मक
कोर�मु�ा�फ��त�ऊजा��और�खा� मु�ा�फ��त�उ�प��होती�है।�
सम��खा��और�ऊजा��क��क�मत��(जैसे�, तेल
व�तु��को�छोड़कर�सभी�व�तु��और
और�गैस) जैसी�व�तुए�ं शा�मल�ह��, जो�ब�त
सेवा��म��मू�य�वृ���दशा�ती�है।� जब�भी�सरकार�उ�च��नवेश�यो�य�पूंजी�का
अ�धक�अ��थर�होती�ह��और�मु�ा�फ��त�क�
�बंधन�करके�उ�चतर��वकास�क��ओर�जाने
वृ���के��लए��वण�होती�ह�।�
म��कामयाब�रही�, तो�उसक��अथ��व�था�पर
मु�ा�फ��त�के�दबाव�थे�तथा��वकास�कम
मु�ा�फ��त�के�कारण�नकारा�मक��प�से
�भा�वत��आ�था।�

QUESTION 71.
�न�न�म��से�कौन�सा�मु�ा�आपू�त��को��नयं��त�करने�म��सहायक�नह��है�?

a) मु��बाजार�नी�त�( Free market policy )


b) सीआरआर�( CRR)
c) ब�क�दर�
d) सीमांत�आव�यकता�म��प�रवत�न�( Change in margin requirement )
Correct Answer: A
Your Answer:
Explanation

Solution (a)

Explanation/Approach

IASbaba
Score:
Web: http://ilp.iasbaba.com/
69.00 /
Email: ilp@iasbaba.com
Page 149 200
AIPTS/ILP VETERANS -
Exam Title :
2020 T...
Email : sahil.law.jmi@gmail.com
Contact : 9540210113

क���य�ब�क�, यानी�भारत�म��RBI, अथ��व�था�म��मु�ा�क��मा�ा�को��नयं��त�करता�है�जैसे�- �रज़व��आव�यकता�, CRR, ब�क�दर�, मु�


बाज़ार�संचालन�, मा�ा�मक�ई�ज़�ग�काय��म�आ�द�( मु��बाज़ार�नी�त�नह�)

क���य�ब�क�यह�सु�न��त�करने�के��लए�कड़ी�मेहनत�करते�ह���क��कसी�दे श�क��अथ��व�था��व�थ�रहे।�क���य�ब�क��का�एक�तरीका�यह�है��क
अथ��व�था�म���वाहमान�मु�ा�क��मा�ा�को��नयं��त�करना।�वे��याज�दर��को��भा�वत�करने�, आर��त�आव�यकता��को��था�पत�करने�और
अ�य����कोण��के�बीच�खुले�बाजार�संचालन�रणनी�त�को��नयो�जत�करके�ऐसा�कर�सकते�ह�।��व�थ�और��थायी�अथ��व�था�सु�न��त�करने�के
�लए��चलन�म��सही�मा�ा�म��मु�ा�होना�मह�वपूण��है।�

QUESTION 72.
�न�न�ल�खत�म��से�कौन�सी�सं�था�' जमा��माणप��' ( Certificate of Deposit ) जारी�कर�सकती�है�?

1. वा�ण��यक�ब�क�
2. �व�ीय�सं�थाए�
3. �नजी�कंप�नयां�
4. डाक�घर�

�न�न�ल�खत�म��से�कूट�का�चयन�कर�:

a) 1 और�2
b) केवल�3
c) 1, 2 और�4
d) उपरो��सभी�
Correct Answer: A
Your Answer:
Explanation

Solution (a)

Explanation:

जमा��माणप��(स�ट��फकेट�ऑफ़��डपॉ�ज़ट- CD) एक�पर�ा�य�( negotiable) मु�ा�बाजार�साधन�है�तथा�यह�एक��न�द���समय�अव�ध


के��लए�ब�क�या�अ�य�पा���व�ीय�सं�थान�म��जमा��कए�गए�धन�के��व���या�आव�धक�वचन�प��( Usance Promissory Note) के
�प�म��जारी��कया�जाता�है।�

भारतीय��रज़व��ब�क�( RBI) �ारा�समय-समय�पर�संशो�धत�जारी��कए�गए��व�भ���नद� श��के�अनुसार�, सीडी�जारी�करने�के��दशा�नद� श


वत�मान�म��शा�सत�ह�।�

सीडी�जारी�कर�सकते�ह��-

• अनुसू�चत�वा�ण��यक�ब�क�(�े�ीय��ामीण�ब�क��और��थानीय��े��ब�क��को�छोड़कर) ; तथा�
• चय�नत�अ�खल�भारतीय��व�ीय�सं�थान��(एफआई) , �ज�ह��आरबीआई��ारा�आरबीआई��ारा�तय�क��गई�अ��ेला�सीमा�के�भीतर
अ�पका�लक�संसाधन�जुटाने�क��अनुम�त�द��गई�है।�

�वक�प�1 और�2 �वक�प�3 और�4

स�य� अस�य�

IASbaba
Score:
Web: http://ilp.iasbaba.com/
69.00 /
Email: ilp@iasbaba.com
Page 150 200
AIPTS/ILP VETERANS -
Exam Title :
2020 T...
Email : sahil.law.jmi@gmail.com
Contact : 9540210113

सीडी�जारी�कर�सकते�ह��-

• अनुसू�चत�वा�ण��यक�ब�क�(�े�ीय��ामीण�ब�क��और
�थानीय��े��ब�क��को�छोड़कर) ; तथा� सीडी�जारी�करने�के��दशा�नद� श�वत�मान�म��भारतीय��रज़व��ब�क�( R
• चय�नत�अ�खल�भारतीय��व�ीय�सं�थान��(एफआई) , BI) �ारा�जारी��कए�गए��व�भ���नद� श���ारा�शा�सत�ह��तथा�इसम�
�ज�ह��आरबीआई��ारा�आरबीआई��ारा�तय�क��गई �नजी�कंप�नयां�या�डाकघर�शा�मल�नह��ह�।�
अ��ेला�सीमा�के�भीतर�अ�पका�लक�संसाधन�जुटाने
क��अनुम�त�द��गई�है।�

QUESTION 73.
‘ रा�ीय�आय�’ (NI) के�बारे�म���न�न�ल�खत�म��से�कौन�गलत�है�?

a) कारक�लागत�( factor cost ) पर��नवल�रा�ीय�उ�पाद�, NI है।�


b) दे श�क��जनसं�या�से��वभा�जत�रा�ीय�आय���त�����आय�( PCI) है।�
c) भारत�2015 से�कारक�लागत�के�आधार�पर�NI क��गणना�कर�रहा�है।�
d) भारत�म��CSO, NI आँकड़��क��गणना�करता�है।�
Correct Answer: C
Your Answer:
Explanation

Solution (c)

कथन��व�ेषण:

कथन�1 कथन�2 कथन�3 कथन�4

स�य� स�य� अस�य� स�य�

भारत�आ�धका�रक�तौर�पर�कारक
लागत�( factor cost) पर
रा�ीय�आय�( NI) को
��त�����आय�(पीसीआई) को�दे श रा�ीय�आय�क��गणना�करता�था। भारत�म��CSO, रा�ीय
कारक�लागत�पर�शु��रा�ीय
क��जनसं�या��ारा�रा�ीय�आय�को जनवरी�2015 से�, सीएसओ�ने आय�( NI) आँकड़��क�
उ�पाद�के�संदभ��म��मापा�जा
�वभा�जत�करके�मापा�जा�सकता�है।� बाजार�मू�य�या�बाजार�लागत�पर गणना�करता�है।�
सकता�है।�
इसक��गणना�करने�के��लए���वच
�कया�है।�

QUESTION 74.
एक�अथ��व�था�म��तरलता�जाल�( liquidity trap ) क����थ�त��न�न�म��से��कसक���वशेषता�है�?

1. उ�च�बचत�दर�
2. बॉ�ड�क��उ�च�मांग�
3. �व�तारवाद��मौ��क�नी�त�

IASbaba
Score:
Web: http://ilp.iasbaba.com/
69.00 /
Email: ilp@iasbaba.com
Page 151 200
AIPTS/ILP VETERANS -
Exam Title :
2020 T...
Email : sahil.law.jmi@gmail.com
Contact : 9540210113

नीचे��दए�गए�कूट�का�उपयोग�करके�सही�उ�र�चुन�:

a) केवल�1 और�2
b) केवल�2 और�3
c) केवल�1 और�3
d) 1, 2 और�3
Correct Answer: C
Your Answer:
Explanation

Solution (c)

Explanation:

तरलता�जाल�( Liquidity trap) एक�ऐसी���थ�त�है�जब��व�तारवाद��मौ��क�नी�त�(मु�ा�क��आपू�त��म��वृ��) �याज�दर�, आय�म��वृ���नह�


करती�है�तथा�इस�लए�आ�थ�क��वकास�को��ो�सा�हत�नह��करती�है।�

यह�ऐसी���थ�त�है��जसम���च�लत��याज�दर��कम�ह��और�बचत�दर��अ�धक�ह��, �जससे�मौ��क�नी�त�अ�भावी�हो�जाती�है।�

एक�तरलता�जाल�म��, उपभो�ा�बॉ�ड�से�बचने�और�अपने�फंड�को�बचत�म��रखने�के��लए�चुनते�ह��, �य��क��च�लत��व�ास�है��क��याज�दर��म�


ज�द�ही�वृ���होगी।��य��क�बांड��का��याज�दर��के��वपरीत�संबंध�होता�है�, ब�त�से�उपभो�ा�ऐसी�संप���नह��रखना�चाहते�, �जसक��क�मत
घटने�क��आशंका�हो।�

QUESTION 75.
�न�न�ल�खत�म��से�कौन�सा�कथन�‘ नो��ो�खाते�’ ( Nostro Account ) को�सही�ढं ग�से�प�रभा�षत�करता�है�?

a) यह�एक�खाता�है�जो�एक��वदे शी�ब�क�, एक�भारतीय�बैक�म���वदे शी�मु�ा�म��रखता�है।�


b) यह�एक�ऐसा�खाता�है��जसे�भारतीय�ब�क�को�RBI म��बनाए�रखना�होता�है।�
c) यह�एक�खाता�है�जो�एक�भारतीय�ब�क�, �वदे शी�ब�क�म���वदे शी�मु�ा�म��रखता�है।�
d) यह�एक�ऐसा�खाता�है��जसे�क���सरकार�को�RBI म��बनाए�रखना�होता�है।�
Correct Answer: C
Your Answer:
Explanation

Solution (c)

Explanation:

नो��ो�खाता�एक�खाते�को�संद�भ�त�करता�है�जो�एक�ब�क�, �कसी�अ�य�ब�क�म��एक��वदे शी�मु�ा�म��रखता�है।�नो��ो�, " हमारा�" के��लए�लै�टन


श�द�से��लया�गया�एक�श�द�है�, �जसका�उपयोग�अ�सर��वदे शी�मु�ा�और��ापार�लेनदे न�क��सु�वधा�के��लए��कया�जाता�है।�वो��ो�खाता�" तु
�हारा�" के��लए�लै�टन�श�द�से��लया�गया��वपरीत�श�द�है�, �क�कैसे�एक�ब�क�उन�खात��को�संद�भ�त�करता�है�जो�अ�य�ब�क��क��अपनी�घरेलू
मु�ा�म��इसके�खाते�रखते�ह�।�

नो��ो�खाता�और�वो��ो�खाता�एक�अलग����कोण�से�एक�ही�बात�को�संद�भ�त�करता�है।�उदाहरण�के��लए�, ब�क�X का�ब�क�Y क��घरेलु�मु�ा


म��Bank Y के�साथ�एक�खाता�है।�ब�क�X के��लए�, यह�एक�नॉ��ो�है�, �जसका�अथ��" आपक��खाता�बही�पर�हमारा�खाता�" है�, जब�क�ब�क
Y के��लए�, यह�एक�वो��ो�है�, �जसका�अथ��है�" हमारी�खाता�बही�पर�आपका�खाता।�" इन�खात��का�उपयोग�अंतररा�ीय�लेनदे न�क��सु�वधा
के��लए�और��व�नमय�दर�के�जो�खम�से�बचने�वाले�लेनदे न�को��नपटाने�के��लए��कया�जाता�है।�

QUESTION 76.

IASbaba
Score:
Web: http://ilp.iasbaba.com/
69.00 /
Email: ilp@iasbaba.com
Page 152 200
AIPTS/ILP VETERANS -
Exam Title :
2020 T...
Email : sahil.law.jmi@gmail.com
Contact : 9540210113

�े�ीय��ामीण�ब�क��(आरआरबी) के�बारे�म���न�न�ल�खत�कथन��पर��वचार�कर�:

1. वे�छोटे �और�सीमांत��कसान��को�सीधे�ऋण��दान�करते�ह�।�
2. वे�भारतीय��रज़व��ब�क��ारा�सह-�ायो�जत�( co-sponsored ) ह�।�
3. वे�अ�य�ब��क�ग�काय��भी�करते�ह�।�
4. रा�य�सरकार��आरआरबी�म��शेयर�हो�डर�ह�।�

आरआरबी�के�बारे�म��उपरो��म��से�कौन�सा�कथन�सही�है�?

a) 1, 2 और�3
b) 2, 3 और�4
c) 1, 3 और�4
d) उपरो��सभी�
Correct Answer: C
Your Answer:
Explanation

Solution (c)

Explanation:

�े�ीय��ामीण�ब�क�(आरआरबी) भारत�के��व�भ��रा�य��म���थानीय��तर�के�ब��क�ग�संगठन�ह�।�वे�मूल�ब��क�ग�और��व�ीय�सेवा��के�साथ�मु�य
�प�से�भारत�के��ामीण��े���क��सेवा�करने�के�उ�े �य�से�बनाए�गए�ह�।�हालाँ�क�, आरआरबी�क��शहरी�प�रचालन�के��लए�शाखाएँ�हो�सकती�ह�
तथा�उनके�संचालन�के��े��म��शहरी��े��भी�शा�मल�हो�सकते�ह�।�

आरआरबी�के�संचालन�का��े��रा�य�के�एक�या�अ�धक��जल��को�कवर�करने�वाले�भारत�सरकार��ारा�अ�धसू�चत��े��तक�सी�मत�है।
आरआरबी��व�भ���कार�के�काय��भी�करते�ह�।�

RRB �न�न�ल�खत��मुख��म���व�भ��काय��करते�ह��

• �ामीण�और�अध�-शहरी��े���म��ब��क�ग�सु�वधाएं��दान�करना।�सरकारी�काय��को�पूरा�करना�जैसे��क�मनरेगा�मज�र��क��मज�री�का
�वतरण�, प�शन�का��वतरण�आ�द।�
• लॉकर�सु�वधा�, डे�बट�और��े�डट�काड��जैसी�पैरा-ब��क�ग�सु�वधाएं��दान�करना।�

�े�ीय��ामीण�ब�क�का��वा�म�व�क���सरकार�, रा�य�सरकार�और��ायोजक�ब�क�के�पास�था�(पाँच�वा�ण��यक�ब�क�थे�, पंजाब�नेशनल�ब�क�,


भारतीय��टे ट�ब�क�, �स��डकेट�ब�क�, यूनाइटे ड�ब�क�ऑफ़�इं�डया�और�यूनाइटे ड�कम�श�यल�ब�क�, जो��े�ीय��ामीण�ब�क�के��ायोजक�थे) �जनम�
क���सरकार�के�शेयर��क���ह�सेदारी�50%, रा�य�सरकार�क��15% और��ायोजक�ब�क��क�- 35% है।�इससे�पहले�, भारतीय��रजव��ब�क�ने
इन�आरआरबी��ारा�लगाए�जाने�वाले��याज�क��दर�पर�सीमा��नधा��रत�कर�द��थी।�

कथन��व�ेषण:

कथन�1 कथन�2 कथन�3 कथन�4

स�य� अस�य� स�य� स�य�

IASbaba
Score:
Web: http://ilp.iasbaba.com/
69.00 /
Email: ilp@iasbaba.com
Page 153 200
AIPTS/ILP VETERANS -
Exam Title :
2020 T...
Email : sahil.law.jmi@gmail.com
Contact : 9540210113

�े�ीय��ामीण�ब�क�(आरआरबी)
�थानीय��तर�के�ब��क�ग�संगठन
ह�।�
�े�ीय��ामीण�ब�क�क���सरकार आरआरबी�अ�य�पैरा- शेयर�धारक��अनुपात�
वे�मु�य��प�से�बु�नयाद��ब��क�ग , रा�य�सरकार�और��ायोजक ब��क�ग�सु�वधाएं�जैसे�लॉकर
• क���सरकार�- 50%
और��व�ीय�सेवा��के�साथ ब�क�के��वा�म�व�और��ायो�जत सु�वधाएं�, डे�बट�और
• रा�य�सरकार- 15% और�
भारत�के��ामीण��े���क��सेवा ह�।� �े�डट�काड���दान�करते�ह�।
• �ायोजक�ब�क- 35% ।
करते�ह�।�वे�छोटे �और�सीमांत
�कसान��को�सीधे�ऋण��दान
करते�ह�।�

QUESTION 77.
' �य�श���समता�( PPP)' एक��कार�का�' �व�नमय�दर��णाली�' है।�पीपीपी�के�बारे�म���न�न�ल�खत�म��से�कौन�सा�कथन�सही�है�/ ह��?

1. �व�भ��मु�ा��क���व�नमय�दर��संबं�धत�दे श��म��उनक��सापे���य�श��य��के�आधार�पर��नधा��रत�क��जाती�ह�।�
2. इस��व�नमय�दर�का�उपयोग��वदे शी�मु�ा�बाजार�( FOREX market ) म��मु�ा��को�खरीदने�और�बेचने�के��लए��कया�जा�सकता
है।�

�न�न�म��से�कूट�चुन�:

a) केवल�1
b) केवल�2
c) दोन��1 और�2
d) न�तो�1 और�न�ही�2
Correct Answer: A
Your Answer:
Explanation

Solution (a)

कथन��व�ेषण:

कथन�1 कथन�2

स�य� अस�य�

�य�श���समता�क��अवधारणा��कसी�को�यह�अनुमान
�य�श���समता�(पीपीपी) �व�नमय�दर��णाली�का�उपयोग��वदे शी
लगाने�क��अनुम�त�दे ती�है��क�दो�दे श��क��मु�ा��क���य
मु�ा�बाजार�म��मु�ा��को�खरीदने�और�बेचने�के��लए�नह���कया�जा
श���के�बराबर�होने�के��लए�दो�मु�ा��के�बीच��व�नमय�दर
सकता�है।�
�या�होगी।�

अ�त�र��जानकारी�

IASbaba
Score:
Web: http://ilp.iasbaba.com/
69.00 /
Email: ilp@iasbaba.com
Page 154 200
AIPTS/ILP VETERANS -
Exam Title :
2020 T...
Email : sahil.law.jmi@gmail.com
Contact : 9540210113

का�प�नक�मु�ा��पांतरण�के��लए�पीपीपी�दर�का�उपयोग�करते��ए�, एक�मु�ा�क��एक�द��गई�रा�श�म��एक�समान��य�श���होती�है�, �जसका


उपयोग�सीधे�बाजार�क��व�तु��क��एक�बा�केट�खरीदने�के��लए��कया�जाता�है�या�पीपीपी�क��दर�को��सरी�मु�ा�म��प�रव�त�त�करने�के��लए
उपयोग��कया�जाता�है�तथा��फर�बाजार�क��बा�केट�का�उपयोग�करके�वह�मु�ा�क��खरीद�क��जाती�है�।��य�श���समता�से��व�नमय�दर�के�दे खे
गए��वचलन�को�वा�त�वक��व�नमय�दर�के��वचलन��ारा�मापा�जाता�है।�

पीपीपी��व�नमय�दर���ामक�अंतररा�ीय�तुलना��को�कम�करने�म��मदद�करती�ह��जो�बाजार��व�नमय�दर��के�उपयोग�के�साथ�उ�प��हो�सकती
ह�।�उदाहरण�के��लए�, मान�ली�जए��क�दो�दे श�दो�अलग-अलग�वष��म��एक-�सरे�के�समान�भौ�तक�मा�ा�का�उ�पादन�करते�ह�।�चूँ�क�बाजार�म�
�व�नमय�दर�म��पया��त�उतार-चढ़ाव�होता�है�, जब�एक�दे श�क��जीडीपी�को�अपनी�मु�ा�म��मापा�जाता�है�, जो�बाजार��व�नमय�दर��का�उपयोग
करके��सरे�दे श�क��मु�ा�म��प�रव�त�त�हो�जाता�है�, एक�दे श�एक�वष��म���सरे�दे श�क��तुलना�म��उ�च�वा�त�वक�जीडीपी�का�अनुमान�लगा�सकता
है�, ले�कन�इसम��कम�अ�य�; ये�दोन��सं�मण�उ�पादन�के�उनके�सापे���तर��क��वा�त�वकता�को���त�ब��बत�करने�म���वफल�ह�गे।�ले�कन�अगर
एक�दे श�क��जीडीपी�को��सरे�दे श��क��मु�ा�म��प�रव�त�त�बाजार��व�नमय�दर��के�बजाय�पीपीपी��व�नमय�दर��का�उपयोग�करके�प�रव�त�त��कया
जाता�है�, तो�गलत��न�कष��नह��होगा।�

आमतौर�पर�संयु��रा�य�अमे�रका�डॉलर�के�सापे��जीडीपी�पीपीपी��नयं�ण�लागत�और�जीवन��तर�क��अलग-अलग�लागत��के��लए��नयं��त
करता�है�, जो��कसी�भी�दे श�क��सकल�आय�का�सट�क��च�ण�करेगा।�

QUESTION 78.
मु�ा�फ��त�क��दर�म��तेजी�से�कमी�के��लए�कभी-कभी�आधार��भाव�( base effect ) को�उ�रदायी�ठहराया�जाता�है।�आधार��भाव��या�है
?

a) यह�मु�ा�फ��त�क��दर�क��गणना�पर��पछले�वष��के�मू�य��तर��का��भाव�है�
b) यह�फसल��क���वफलता�के�कारण�आपू�त��म��भारी�कमी�का��भाव�है�
c) यह�ती��आ�थ�क��वकास�के�कारण�मांग�म��वृ���का��भाव�है�
d) उपरो��म��से�कोई�भी�कथन�सही�नह��है�
Correct Answer: A
Your Answer:
Explanation

Solution (a)

Explanation:

आधार��भाव�( Base effect) �पछले�वष��क��इसी�अव�ध�म��मु�ा�फ��त�से�संबं�धत�है�, य�द��पछले�वष��क��इसी�अव�ध�म��मु�ा�फ��त�क��दर


ब�त�कम�थी�, तो�मू�य�सूचकांक�म��मामूली�वृ���भी�अब�मु�ा�फ��त�क��उ�च�दर�दे गी।��सरी�ओर�, य�द�मू�य�सूचकांक��पछले�वष��क��इसी
अव�ध�म��उ�च�दर�से�बढ़�गया�था�और�उ�च�मु�ा�फ��त�दर�दज��क��गई�थी�, तो�मू�य�सूचकांक�म��एक�समान��नरपे��वृ���अब�कम�मु�ा�फ��त
दर��दखाएगी।�

QUESTION 79.
पीयर-टू -पीयर�ल��ड�ग�(पी�2 पी) �लेटफाम��के�बारे�म���न�न�ल�खत�म��से�कौन�सा�कथन�सही�है�/ ह��?

1. उ�ह��गैर-ब��क�ग��व�ीय�कंप�नय��( NBFC) के��प�म��माना�जाता�है�


2. वे�भारतीय��रजव��ब�क�( RBI) �ारा��व�नय�मत�होते�ह��
3. वे�उधारकता��, ऋणदाता�और�साझेदार�ब�क�के�बीच�म�य�थ�के��प�म��काय��करते�ह��

उपयु��कूट�चुन�:

a) केवल�3
b) 1 और�3
c) 1 और�2

IASbaba
Score:
Web: http://ilp.iasbaba.com/
69.00 /
Email: ilp@iasbaba.com
Page 155 200
AIPTS/ILP VETERANS -
Exam Title :
2020 T...
Email : sahil.law.jmi@gmail.com
Contact : 9540210113

d) 1, 2 और�3
Correct Answer: D
Your Answer:
Explanation

Solution (d)

Explanation:

पी�2 पी�ऋण�, एक��ाउड-फं�ड�ग�मॉडल�(काफ��हद�तक�ऑनलाइन) है�जहां�लोग�अपने�पैसे�को�उन�लोग��के�साथ��नवेश�करना�चाहते�ह��जो


ऐसा�करना�चाहते�ह�।�यह�अवधारणा�बचत�करने�वाल��के�बजाय�अपने�पैसे�उधार�दे �कर�उ�च��याज��ा�त�करने�वाल��लोग��के�आसपास�क���त
है�और�उधारकता���को�तुलना�मक��प�से�कम��याज�दर��मलती�है।�

उधारकता��या�तो�����या�छोटे ��वसाय�होते�ह�।�ले�कन�एक�पारंप�रक�बचत�खाते�के��वपरीत�, य�द�उधारकता���दवा�लया�होता�है�, तो�कोई


भी�अपना�पैसा�खो�सकता�है।�

पीयर-टू -पीयर�ल��ड�ग�(पी�2 पी) �लेटफाम��को�गैर-ब��क�ग��व�ीय�कंप�नय��(एनबीएफसी) के��प�म��माना�जाएगा�तथा�भारतीय��रजव��ब�क


(आरबीआई) �ारा��व�नय�मत��कया�जाएगा।�

�या�आप�जानते�ह��?

एक�एनबीएफसी- पी�2 पी�करेगा�

• पीयर�टू �पीयर�ल��ड�ग�म��शा�मल���तभा�गय��को�एक�ऑनलाइन�माक�ट�लेस�या��लेटफॉम���दान�करने�वाले�एक�म�य�थ�के��प�म��काय�
करेगा�;
• कंपनी�अ�ध�नयम�, 2013 क��धारा�45I (bb) के�तहत�या�अ�ध�नयम�के��प�म��प�रभा�षत�जमा��वीकार�नह��करेगा�;
• �वयं�अपने�आपको�ऋण�नह��दे गा�;
• कोई�ऋण�वृ���या�ऋण�गारंट���दान�या��व��थत�नह��करेगा�;
• अपने��लेटफ़ॉम��से�जुड़े��कसी�भी�सुर��त�ऋण�क��सु�वधा�या�अनुम�त�नह��; यानी�केवल��लीन�ऋण�क��अनुम�त�होगी�;
• अपनी��वयं�क��बैल�स�शीट�पर�, ऋण�दे ने�के��लए�उधारदाता��से��ा�त�धन�, या�ऋण�दे ने�वाल��से��ा�त�धनरा�श�पर�पकड़�नह��होगी
;
• ऋण��व�श��बीमा�उ�पाद��को�छोड़कर��कसी�भी�उ�पाद�को�नही�बेच�ग��;
• �न�धय��के�अंतरा��ीय��वाह�क��अनुम�त�नह��;
• �ासं�गक�कानून��के�तहत��नधा��रत���तभा�गय��के��लए�लागू�कानूनी�आव�यकता��का�पालन�सु�न��त�करना।�
• भारत�म����थत�हाड�वेयर�पर�अपनी�ग�त�व�धय��और���तभा�गय��से�संबं�धत�सभी�डेटा�को�सं�हीत�और�संसा�धत�करना।�
• ��तभा�गय��पर�उ�चत��यान�रखना�;
• उधारकता���के��े�डट�मू�यांकन�और�जो�खम��ोफाइ�ल�ग�का�काय��करना�तथा�उनके�भावी�ऋणदाता��के��लए�इसका�खुलासा�करना
;
• इसक���े�डट�जानकारी�तक�प�ंचने�के��लए���तभागी�क��पूव��और��प��सहम�त�क��आव�यकता�होती�है�;
• ऋण�समझौत��और�अ�य�संबं�धत�द�तावेज��का��लेखन�;
• सं�वतरण�और�ऋण�रा�श�के�पुनभु�गतान�म��सहायता��दान�करना�;
• �लेटफाम��पर�उ�प��ऋण��क��वसूली�के��लए�सेवाएं��दान�करना।�

QUESTION 80.
�न�न�ल�खत�म��से�कौन�सी�एज�सी�गैर�ब��क�ग��व�ीय�कंपनी�( NBFC) को��नयं��त�करती�है�?

a) भारतीय��रजव��ब�क�
b) सेबी�( SEBI)
c) �व��मं�ालय�
d) रा�य�सरकार��
Correct Answer: A

IASbaba
Score:
Web: http://ilp.iasbaba.com/
69.00 /
Email: ilp@iasbaba.com
Page 156 200
AIPTS/ILP VETERANS -
Exam Title :
2020 T...
Email : sahil.law.jmi@gmail.com
Contact : 9540210113

Your Answer:
Explanation

Solution (a)

Explanation:

एक�गैर�ब��क�ग��व�ीय�कंपनी�( NBFC) भारत�के�कंपनी�अ�ध�नयम�, 1956 के�तहत�पंजीकृत�एक�कंपनी�है�, जो�ऋण��और�अ��म��, शेय


र��के�अ�ध�हण�, �टॉक�, बॉ�ड् स��कराया-खरीद�, बीमा��वसाय�या��चट��वसाय�के��वसाय�म��संल�न�है�, ले�कन�इसम��ऐसी�कोई�भी�सं�था
शा�मल�नह��है��जसके��मुख��वसाय�म��कृ�ष�, औ�ो�गक�ग�त�व�ध�या�अचल�संप���क���ब���, खरीद�या��नमा�ण�शा�मल�है।�

भारतीय��रज़व��ब�क�(आरबीआई) �ारा�भारतीय��रज़व��ब�क�अ�ध�नयम�, 1934 ( अ�याय�III B) के�ढांचे�के�भीतर�तथा�इसके��ारा�जारी


�कए�गए��नद� श��के�तहत�एनबीएफसी�के�काम�और�संचालन�को��व�नय�मत��कया�जाता�है।�

QUESTION 81.
ओपन�माक�ट�ऑपरेशंस�के�मा�यम�से�, RBI खरीद�और��ब���करता�है�

1. सरकारी���तभू�त�
2. सोना�
3. �वदे शी�मु�ा�( Forex)
4. पीएसयू�के�शेयर�

�न�न�ल�खत�म��से�कूट�का�चयन�कर�:

a) केवल�1
b) 2 और�3
c) 1,2 और�3
d) उपरो��सभी�
Correct Answer: A
Your Answer:
Explanation

Solution (a)

Approach/explanation:

ओपन�माक�ट�ऑपरेश�स�, RBI के�/ से�बाज़ार�तक�सरकारी���तभू�तय��( G-Sec) क��खरीद�और��ब���का�उ�लेख�करते�ह�।�ओपन�माक�ट


ऑपरेशंस�का�उ�े �य�अथ��व�था�म���टकाऊ�आधार�पर��पये�क��तरलता�क����थ�तय��को�समायो�जत�करना�है।�

जब�RBI बाजार��म��सरकारी���तभू�त�बेचता�है�, तो�ब�क�उ�ह��खरीद�लेते�ह�।�जब�ब�क�सरकारी���तभू�तय��क��खरीद�करते�ह��, तो�उनके�पास


औ�ो�गक�इकाइय��या�अ�य�वा�ण��यक��े���को�उधार�दे ने�क���मता�कम�होती�है।�इससे�अ�धशेष�नकद��कम�होती�है�, �पये�क��तरलता�कम
होती�है�और�फल�व�प�ऋण��नमा�ण�/ ऋण�आपू�त��म��कमी�होती�है।�जब�RBI ��तभू�तय��क��खरीद�करता�है�, तो�वा�ण��यक�ब�क�उ�ह�
अ�धक�अ�धशेष�नकद��के�साथ�पाते�ह��और�इससे��णाली�म��अ�धक�ऋण�पैदा�होता�है।�इस��कार�, अ�त�र��तरलता�के�मामले�म��, RBI �
णाली�से��पया�बाहर��नकालने�के��लए�G-sec क���ब���का�समथ�न�करता�है।�

इसी�तरह�, जब�अथ��व�था�म��तरलता�क��कमी�होती�है�, तो�RBI बाजार�से���तभू�तयां�खरीदता�है�, �जससे�तरलता�जारी�होती�है।�यहां�यह


�यान�दे ने�यो�य�है��क�सरकारी���तभू�तय��का�बाजार�भारत�म��अ�छ��तरह�से��वक�सत�नह���आ�है�, ले�कन��फर�भी�OMO ब�त�मह�वपूण�
भू�मका��नभाता�है।�

QUESTION 82.

IASbaba
Score:
Web: http://ilp.iasbaba.com/
69.00 /
Email: ilp@iasbaba.com
Page 157 200
AIPTS/ILP VETERANS -
Exam Title :
2020 T...
Email : sahil.law.jmi@gmail.com
Contact : 9540210113

भारतीय��पया��न�न�ल�खत�म��से��कस�दे श�म��एक�वैधा�नक��न�वदा�( legal tender ) है�?

1. बां�लादे श�
2. पा�क�तान�
3. नेपाल�
4. भूटान�

�न�न�ल�खत�म��से�कूट�का�चयन�कर�:

a) 1 और�2
b) 3 और�4
c) 1, 3 और�4
d) इनम��से�कोई�भी�नह��
Correct Answer: B
Your Answer:
Explanation

Solution (b)

Explanation:

भारतीय�मु�ा�को�नेपाल�और�भूटान�म��एक�वैधा�नक��न�वदा�( Legal tender) माना�जाता�है।�

वैधा�नक��न�वदा�, कानून��ारा�मा�यता��ा�त�भुगतान�का�कोई�आ�धका�रक�मा�यम�है��जसे��कसी�साव�ज�नक�या��नजी�ऋण�को�समा�त�करने
या��व�ीय�दा�य�व�को�पूरा�करने�के��लए�उपयोग��कया�जा�सकता�है।�

रा�ीय�मु�ा��ावहा�रक��प�से���येक�दे श�म��वैधा�नक��न�वदा�है।�एक�लेनदार�एक�ऋण�को�चुकाने�के��लए�वैधा�नक��न�वदा��वीकार�करने�के
�लए�बा�य�है।�वैधा�नक��न�वदा�केवल�रा�ीय��नकाय��ारा�जारी�क��जा�सकती�है�जो�ऐसा�करने�के��लए�अ�धकृत�है।�

QUESTION 83.
�न�न�ल�खत�पर��वचार�कर��

1. र�गना�( Creeping )- जब�मु�ा�फ��त�1% से�5% के�बीच�हो�


2. �ो�ट�ग�( Trotting )- जब�मु�ा�फ��त�6% से�10% के�बीच�हो�
3. सरपट�दौड़ना�( Galloping ) - जब�मु�ा�फ��त�11% से�20% के�बीच�हो�
4. भागना�( Runaway )- जब�मु�ा�फ��त�21% से�50% के�बीच�हो�
5. अ�त�उ�च�मु�ा�फ��त�( Hyperinflation ) - जब�मु�ा�फ��त�ब�त�अ�धक�और�50% से�ऊपर�है�

उपरो��म��से�कौन�सही�ढं ग�से�सुमे�लत�है�/ है�?

a) 1, 3 और�5
b) 1, 2, 4 और�5
c) 1, 2, 3, 4 और�5
d) 2, 3, 4 और�5
Correct Answer: C
Your Answer:
Explanation

Solution (c)

दर�के�संबंध�म��मु�ा�फ��त�के��कार�

IASbaba
Score:
Web: http://ilp.iasbaba.com/
69.00 /
Email: ilp@iasbaba.com
Page 158 200
AIPTS/ILP VETERANS -
Exam Title :
2020 T...
Email : sahil.law.jmi@gmail.com
Contact : 9540210113

मु�ा�फ��त�दर�के�आधार�पर��वभा�जत�क��जाती�है�, ले�कन�सीमांकन�क��कोई��प��रेखाएं�नह��होती�ह��तथा�इस�लए�आप�उ�ह��अ�सर�मु�य
�प�से�उपयोग��कए�जा�रहे�दे ख�सकते�ह��

• र�गना�( Creeping)- जब�मु�ा�फ��त�1% से�5% के�म�य�हो�


• �ो�ट�ग�( Trotting) - जब�मु�ा�फ��त�6% से�10% के�म�य�हो�
• सरपट�दौड़ना�( Galloping) - जब�मु�ा�फ��त�11% से�20% के�म�य�हो�
• भागना�( Runaway) - जब�मु�ा�फ��त�21% से�50% के�म�य�हो�
• अ�त�उ�च�मु�ा�फ��त�( Hyperinflation) - जब�मु�ा�फ��त�ब�त�अ�धक�और�50% से�ऊपर�हो�

QUESTION 84.
�न�न�ल�खत�कथन��पर��वचार�कर��

1. मौ��क�जमा�अनुपात�( Currency Deposit Ratio ), मु�ा�के��प�म��जनता�के�पास�मौजूद�मु�ा�का�अनुपात�है�जो�वे�ब�क�जमा


म��रखते�ह��
2. आर��त�जमा�अनुपात�( Reserve Deposit Ratio ), कुल�जमा�का�अनुपात�है�, �जसे�वा�ण��यक�ब�क��रजव��के��प�म��रखते
ह�।�

उपरो��म��से�कौन�सा�स�य�है�/ ह��?

a) केवल�1
b) केवल�2
c) दोन��
d) कोई�नह��
Correct Answer: C
Your Answer:
Explanation

Solution (c)

कथन��व�ेषण:

कथन�1 कथन�2

मौ��क�जमा�अनुपात�( Currency Deposit �रजव��जमा�अनुपात�( Reserve Deposit Ratio -


Ratio- CDR) RDR)

स�य� स�य�

IASbaba
Score:
Web: http://ilp.iasbaba.com/
69.00 /
Email: ilp@iasbaba.com
Page 159 200
AIPTS/ILP VETERANS -
Exam Title :
2020 T...
Email : sahil.law.jmi@gmail.com
Contact : 9540210113

यह�कुल�जमा�का�अनुपात�है�, जो�वा�ण��यक�ब�क��रज़व��के��प�म�
रखते�ह�।�

ब�क��म��लोग���ारा�जमा�रा�श�का�उपयोग�ब�क���ारा��सर��को�उधार
जैसा��क�नाम�से�पता�चलता�है��क�यह�मु�ा�के��प�म��जनता
दे ने�के��लए��कया�जाता�है�ले�कन�पूरी�जमा�रा�श�उधार�नह��होती�है�,
के�पास�मौजूद�धन�का�अनुपात�है�जो�वे�ब�क�जमा�म��रखते
इसका�कुछ��ह�सा��रज़व��धन�के��प�म��रखा�जाता�है।�
ह�।�
�रजव��मु�ा�को�2 भाग��म���वभा�जत��कया�गया�है�- ब�क��म��वॉ�ट
CDR = CU / DD
कैश�और�RBI के�पास�वा�ण��यक�ब�क��का�जमा।�
�यादातर�लोग�जमा�के�बजाय�नकद��रखना�पसंद�करते�ह�
जमाकता���क��नकद�मांग��को�पूरा�करने�के��लए�ब�क��म��पहले�भाग
और�यह�खच��के�मौसमी�पैटन��पर�भी��नभ�र�करता�है।
यानी�वॉ�ट�कैश�का�उपयोग��कया�जाता�है।�
उदाहरण�के��लए�- �योहार��के�दौरान�लोग�अ�धक�नकद�
रखते�ह��, �य��क��य�अ�धक�होते�ह�।� �रजव��रखना�ब�क��के��लए�महंगा�है��य��क�वे�इस��ह�से�को�भी�उधार
दे �सकते�ह��और��याज�कमा�सकते�ह��ले�कन�आरबीआई�यह�सु�न��त
करता�है��क�वे�अपने�पास�सुर��त�रख��ता�क�वे�खाताधारक��क�
नकद��क��मांग�को��बना��कसी�सम�या�के�पूरा�कर�सक�।�

QUESTION 85.
इस�श�द�का�उपयोग�उस���थ�त�को�संद�भ�त�करने�के��लए��कया�जाता�है�जहां�अप�फ��त�( deflation ) को�रोकने�के��लए�उपाय��कए�जाते
ह�।�

a) अ�त�मु�ा�फ��त�( Hyperinflation )
b) पुन: मु�ा�फ��त�Reflation
c) �व�फ��त�( Disinflation )
d) कोई�नह��
Correct Answer: B
Your Answer:
Explanation

Solution (b)

मु�ा�फ��त�म��शत�:

1. �टै ग�लेशन�( Stagflation): ऐसी���थ�त�जब�मु�ा�फ��त�और�बेरोजगारी�ब�त�उ�च�है�, ले�कन�आ�थ�क��वकास�धीमा�है।�

2. अप�फ��त�( Deflation) : साधारणतः�नकारा�मक�मु�ा�फ��त�(शू�य�से�नीचे) को�अप�फ��त�कहा�जाता�है।�जहां�क�मत���गर�रही�ह�।

• कम�पैसे�या�कम�मांग� 01F
86A क�मत��म��कमी� 86A कारखान��म��कम�उ�पादन� 86A कम��नवेश�
01F 01F

• यह�एक�स�प�ल�आकार�म��चला�जाता�है�अगर��नयं��त�नह���कया�जाता�है��जसे�अप�फ��त�संबंधी�स�प�ल�( deflationary
spiral) के��प�म��भी�जाना�जाता�है।�इसी�तरह�जब�यह�मु�ा�फ��त�के�साथ�सकारा�मक��दशा�म��होता�है�, तो�इसे�इ��लेटर- Y स�प�
ल�या�मज़�री-मू�य�स�प�ल�( wage-price spiral) के��प�म��जाना�जाता�है।�

3. �व�फ��त�( Disinflation) : �पछली�अव�ध�के�संबंध�म��मु�ा�फ��त�क��दर�म��उ�लेखनीय��गरावट�, ले�कन�अभी�भी�सकारा�मक�है।


(याद�रख��यह�अप�फ��त�से�अलग�है��य��क�अप�फ��त�शू�य�से�नीचे�क��मु�ा�फ��त�है)

4. पुन: मु�ा�फ��त�( Reflation): इस�श�द�का�उपयोग�उस���थ�त�को�संद�भ�त�करने�के��लए��कया�जाता�है�जहां�अप�फ��त�को�रोकने


के��लए�उपाय��कए�जाते�ह�।�कदम�राजकोषीय�नी�त�(कर��को�कम�करने) या�मौ��क�नी�त�(मु�ा�आपू�त��बढ़ाने�या��याज�दर��को�कम�करने) क�
तरह�हो�सकते�ह��

IASbaba
Score:
Web: http://ilp.iasbaba.com/
69.00 /
Email: ilp@iasbaba.com
Page 160 200
AIPTS/ILP VETERANS -
Exam Title :
2020 T...
Email : sahil.law.jmi@gmail.com
Contact : 9540210113

5. धीमापन- मंद��और�अवसाद�( Slowdown – Recession & Depression): ये�तीन�श�द�अंतर-संबं�धत�ह�।�आमतौर�पर


पहले�धीमापन�आता�है�, �फर�मंद��आती�है�और�अंत�म��अवसाद�होता�है�

• धीमापन�: �कसी�दे श�क��अथ��व�था�म���गरावट�- यह�जीडीपी�से�संबं�धत�है�, मु�ा�फ��त�से�नह��


• मंद�: य�द���मक�2 �तमा�हय��के��लए�धीमापन�होता�है�यानी�जीडीपी�2 ��मक��तमा�हय��के��लए��गरती�है�, तो�इसे�मंद��के��प�म�
जाना�जाता�है।�जीडीपी�और��नवेश�म��सामा�य�संकेतक��गर�रहे�होते�ह��
• अवसाद: आम�तौर�पर�य�द�मंद��लंबे�समय�तक�रहती�है�, तो�यह�कहा�जाता�है��क�अथ��व�था�अवसाद�म��है।�मु�य�संकेतक�जीडीपी
और��नवेश�म��ती���गरावट�के�साथ�व�तु��और�सेवा��क��मांग�म��भारी��गरावट�ह�।�उदा: 1930 का�महामंद��

QUESTION 86.
‘ लचीली��व�नमय�दर��णाली�’ ( flexible exchange rate system ) के�तहत�, �व�नमय�दर��नधा��रत�क��जाती�है�

a) क���य�ब�क��ारा�
b) �व���ापार�संगठन��ारा�
c) मु�य��प�से�बाजार�तं���ारा�
d) मु�ा��के�एक�समूह�के�भा�रत�सूचकांक�के��प�म��
Correct Answer: C
Your Answer:
Explanation

Solution (c)

Explanation:

लचीली��व�नमय�दर�( Flexible exchange rate)

लचीली��व�नमय�दर��को�मु�ा�क��वै��क�आपू�त��और�मांग��ारा��नधा��रत��व�नमय�दर��के��प�म��प�रभा�षत��कया�जा�सकता�है।��सरे�श�द��म�
, वे�बाजार��ारा��नधा��रत��वदे शी�मु�ा�क��क�मत��ह��, जो�आपू�त��और�मांग�के�कारण�तेजी�से�बदल�सकती�ह��, और�क���य�ब�क���ारा�न�तो
आंक��जाती�ह��और�न�ही��नयं��त�होती�ह�।��वपरीत�प�र��य�, जहां�क���य�ब�क��व�नमय�दर�को�सीमा�के�भीतर�रखने�के��लए��वदे शी�और�घरेलू
मु�ा�क��खरीद�और��ब���के�साथ�बाजार�म��ह�त�ेप�करते�ह��, �जसे�ब�ड�के��प�म��भी�जाना�जाता�है�, इसे��न��त��व�नमय�दर�( fixed
exchange rate) कहा�जाता�है।�

QUESTION 87.
भारत�म���न�न�ल�खत�म��से�कौन�सा�सेवा��े��का��ह�सा�नह��है�?

a) प�रवहन�
b) �नमा�ण�
c) होटल�और�रे�तरां�
d) बीमा�
Correct Answer: B
Your Answer:
Explanation

Solution (b)

मा�य�मक�ग�त�व�धयाँ�क�चे�माल�को�मू�यवान�उ�पाद��म��प�रव�त�त�करके��ाकृ�तक�संसाधन��के�मू�य�म��वृ���करती�ह�।�इस�लए�, मा�य�मक
ग�त�व�धयाँ��व�नमा�ण�, �सं�करण�और��नमा�ण�(अवसंरचना) उ�ोग��से�संबं�धत�ह�।�

मा�य�मक�ग�त�व�धय��म��लगे�लोग��को��लू�कॉलर���मक�कहा�जाता�है।�

IASbaba
Score:
Web: http://ilp.iasbaba.com/
69.00 /
Email: ilp@iasbaba.com
Page 161 200
AIPTS/ILP VETERANS -
Exam Title :
2020 T...
Email : sahil.law.jmi@gmail.com
Contact : 9540210113

QUESTION 88.
पूंजीगत��य�( Capital Expenditure ) के�संबंध�म���न�न�ल�खत�म��से�कौन�सा�सही�नह��है�?

a) इसम��वे��य�शा�मल�होते�ह��जो��थायी�संप���बनाते�ह��और�आव�धक�आय�उ�प��करते�ह�।�
b) यह�आव�यक�है��क�ठोस�संप���उ�पादक�या�राज�व�पैदा�करने�वाली�होनी�चा�हए।�
c) ऋण��का�पुनभु�गतान�, शेयर��म���नवेश�, क���सरकार��ारा�रा�य�सरकार�/ रा�य�, �वदे शी�सरकार�/ सरकार�और�सरकारी�कंप�नय��को
ऋण�- सभी�पूंजीगत��य�का��ह�सा�ह�।�
d) अ�थायी�संप���पर��य�को�आमतौर�पर�पूंजीगत��कृ�त�के��य�के��प�म��नह��माना�जा�सकता�है।�
Correct Answer: B
Your Answer:
Explanation

Solution (b)

कथन��व�ेषण:

कथन�( a) कथन�( b) कथन�( c) कथन�( d)

स�य� अस�य� स�य� स�य�

पूंजीगत��य�को�आम�तौर�पर
�कसी�साम�ी�क��बढ़ती��ई�ठोस
संप���(जैसे�सड़क��, औ�ो�गक
भवन��या�उपकरण��के��नमा�ण)
तथा��थायी�च�र��या�आवत�
दे नदा�रय��को�कम�करने�(ऋण�क�
चुकौती) के�उ�े �य�से��कए�गए��य
के��प�म��प�रभा�षत��कया�जा
सकता�है।� ऋण�का�पुनभु�गतान�, शेयर��म�
यह�ज�री�नह��है��क�ठोस�संप�� �नवेश�, क���सरकार��ारा�रा�य अ�थायी�संप���पर��य
एक�उ�पादक�संप���को�एक�माना
च�र��म��उ�पादक�होनी�चा�हए�या सरकार�/ रा�य�, �वदे शी�सरकार को�आमतौर�पर�पूंजीगत
जा�सकता�है�जो�अपने�कामकाज�के
यह��क�वे�राज�व�का�उ�पादन�भी / सरकार�और�सरकारी�कंप�नय� �कृ�त�के��य�के��प�म�
�लए��ासं�गक�सभी�शु�क��पर
कर�।� को�ऋण�- सभी�पूंजीगत��य नह��माना�जा�सकता�है।�
अ�धशेष�वहन�करने�के��लए�पया��त
का��ह�सा�ह�।�
राज�व�का�उ�पादन�करती�है।�

यह�कम�ही�अवसर��पर�आव�यक
और�उ�चत�हो�सकता�है��य��क
पूंजी�को�एक�ऐसी�योजना�के��प�म�
माना�जाए�जो��ावसा�यक��प�से
लाभकारी�नह��है�, ले�कन�बड़े��य
को�शा�मल�करती�है�जैसे�एक�नए
शहर�का��नमा�ण।�

QUESTION 89.

IASbaba
Score:
Web: http://ilp.iasbaba.com/
69.00 /
Email: ilp@iasbaba.com
Page 162 200
AIPTS/ILP VETERANS -
Exam Title :
2020 T...
Email : sahil.law.jmi@gmail.com
Contact : 9540210113

बाजार���थरीकरण�योजना�( MSS) के�संदभ��म��, �न�न�ल�खत�कथन��पर��वचार�कर�:

1. यह�RBI �ारा�सरकार�क��ओर�से���तभू�तय��के��नग�मन�के�मा�यम�से�बाजार�से�अ�त�र��तरलता�को�समा�त�करने�के��लए�उपयोग
�कया�जाने�वाला�एक�उपकरण�है।�
2. एमएसएस�के�तहत�जुटाए�गए�पैसे�को�एमएसएस�अकाउंट�के�नाम�से�अलग�खाते�म��रखा�जाता�है�, न��क�सरकारी�खाते�म��रखा�जाता
है�या�अपने�खच��के��लए�इ�तेमाल��कया�जाता�है।�

ऊपर��दए�गए�कथन��म��से�कौन�सा�स�य�है�/ ह��?

a) केवल�1
b) केवल�2
c) दोन��1 और�2
d) न�तो�1 और�न�ही�2
Correct Answer: C
Your Answer:
Explanation

Solution (c)

कथन��व�ेषण:

कथन�1 कथन�2

स�य� स�य�

एमएसएस�के�तहत�जुटाए�गए�पैसे�को�एमएसएस�अकाउंट�के
बाज़ार���थरीकरण�योजना�या�MSS भारतीय��रज़व��ब�क��ारा नाम�से�अलग�खाते�म��रखा�जाता�है�, न��क�सरकारी�खाते�म��रखा
सरकार�क��ओर�से��े जरी��ब�स�, �दनां�कत���तभू�त�आ�द�जैसे जाता�है�या�अपने�खच��के��लए�इ�तेमाल��कया�जाता�है।�
��तभू�तय��के��नग�मन�के�मा�यम�से�बाजार�से�अ�त�र��तरलता
MSS के�तहत��ा�त�धन�को�RBI के�पास�रखा�जाता�है।�इसे
को�समा�त�करने�के��लए�उपयोग��कया�जाने�वाला�एक
सरकार�को�ह�तांत�रत�नह���कया�जाता�है।�इसका�कारण�यह�है�,
उपकरण�है।�
अगर�इसे��थानांत�रत��कया�जाता�है�, तो�सरकार�उस
अथ��व�था�म��पैसा�खच��करेगी��जससे�तरलता�बढ़े गी।�

अ�त�र��जानकारी:

• MSS को�पहली�बार�अ�ैल�2004 म��आरंभ��कया�गया�था।�MSS के�बारे�म��मु�य�बात�यह�है��क�इसका�उपयोग�सरकारी�बॉ�ड


बेचकर��णाली�से�अ�त�र��तरलता�या�धन��नकालने�के��लए��कया�जाता�है।�
• �वमु��करण�के�बाद�, भारी�जमा�को�ब��क�ग��णाली�म��डाल��दया�गया।�उसी�समय�, ब�क�इसे��ाहक��को�उधार�नह��दे �सकते��य��क
यह��सफ��अ�थायी�धन�है।�RBI ने�ब�क��को�CRR के��प�म��सभी�अ�त�र��जमा�रखने�का��नद� श��दया�है।�ले�कन�यहां�, ब�क��को
नुकसान�होगा��य��क�उ�ह��जमाकता���को��याज�दे ना�होगा।�
• ब�क��को���तपू�त��करने�के��लए�, एमएसएस�नी�त�को�पुनज��वत��कया�गया�है।�यहां�, ब�क�जमा�से��ा�त�अ�त�र��धन�को�माक�ट
�टे बलाइजेशन�बॉ�ड् स�( MSBs) म��रख�सकते�ह�।�वे��याज�भुगतान�भी��ा�त�कर�सकते�ह�।�

QUESTION 90.

IASbaba
Score:
Web: http://ilp.iasbaba.com/
69.00 /
Email: ilp@iasbaba.com
Page 163 200
AIPTS/ILP VETERANS -
Exam Title :
2020 T...
Email : sahil.law.jmi@gmail.com
Contact : 9540210113

आय�के��वतरण�के�मापन�के��लए��न�न�ल�खत�म��से��कसका�उपयोग��कया�जाता�है�?

a) लाफ़र�व��
b) �फ�ल�स�व��
c) एंजेल�का��नयम�
d) �गनी-लोर�ज़�व��
Correct Answer: D
Your Answer:
Explanation

Solution (d)

Explanation:

एक�अथ��व�था�म��आय�का��वतरण�लोर�ज�व���ारा�दशा�या�जाता�है�तथा�आय�असमानता�क��मा�ा�को��गनी�गुणांक�के�मा�यम�से�मापा�जाता
है।�सरकार�के�पाँच��मुख�और�सामा�य�वृहद�आ�थ�क�ल�य��म��से�एक�आय�का�समान�(उ�चत) �वतरण�है।�

QUESTION 91.
RBI क��‘ संकुचना�मक�मौ��क�नी�त�’ ( Contractionary monetary policy ), के�भाग�/ प�रणाम��न�न�ल�खत�म��से�कौन�से�ह�
?

1. यह�अथ��व�था�म��उपल�ध�ऋण�क��मा�ा�को���तबं�धत�करता�है�
2. ब�क�दर�को�घटाना�मु�य�घटक�है�
3. इसका�उ�े �य�मु�ा�फ��त�का�मुकाबला�करना�है�

ऊपर��दए�गए�कथन��म��से�कौन�सा�सही�है�/ ह��?

a) केवल�2
b) 2 और�3 केवल�
c) 1, 2 और�3
d) केवल�1 और�3
Correct Answer: D
Your Answer:
Explanation

Solution (d)

जानती�हो�?

संकुचन�वाली�मौ��क�नी�त�को�महंगी�मु�ा�नी�त�( Dear Money policy) भी�कहा�जाता�है।�

कथन��व�ेषण:

कथन�1 कथन�2 कथन�3

स�य� अस�य� स�य�

IASbaba
Score:
Web: http://ilp.iasbaba.com/
69.00 /
Email: ilp@iasbaba.com
Page 164 200
AIPTS/ILP VETERANS -
Exam Title :
2020 T...
Email : sahil.law.jmi@gmail.com
Contact : 9540210113

संकुचन�वाली�मौ��क�नी�त�अथ��व�था�म� मु�ा�फ��त�का�मुकाबला�करने�के��लए
ब�क�दर�घटने�से�संकुचन�वाली�मौ��क
उपल�ध�ऋण�क��मा�ा�को�सी�मत�करती संकुचन�वाली�मौ��क�नी�त�का�उपयोग��कया
नी�त�के�कारण�म��मदद�नह���मलेगी।�
है।� जाता�है।�

अ�त�र��जानकारी:

संकुचन�वाली�मौ��क�नी�त�के��भाव:

• मु�ा�फ��त�म��कमी�
• बेरोजगारी�बढ़ा�सकते�ह��(�य��क�यह��वकास�दर�को�नीचे�लाता�है)
• �े�डट�(ऋण) क��एक�उ�च�लागत�के�कारण��वसाय�के��व�तार�म���गरावट�

संकुचन�वाली�मौ��क�नी�त�के��लए�उपकरण�

• �याज�दर��म��वृ���
• �रज़व��आव�यकता��को�बढ़ाना�जैसे��क�ब�क��के�एसएलआर�और�/ या�सीआरआर�

QUESTION 92.
�न�न�ल�खत�यु�म��पर��वचार�कर��

पु�तक�लेखक�

1. रा��के�धन�क���कृ�त�और�कारण�क��एक�जांच�: अम�य��सेन�
2. द�इकोनॉ�मक�क�सेकुएंसेज़�ऑफ�द�पीस�: एडम���मथ�
3. रोजगार�, �याज�और�मु�ा�का�सामा�य��स�ांत�: जे�एम�क��स�
4. असमानता�पुन:परी�ण�( Inequality Reexamined ) : काल��मा�स��

ऊपर�द��गई�कौन�सी�जोड़ी�सही�ढं ग�से�सुमे�लत�है�/ ह��?

a) केवल�3
b) केवल�3 और�4
c) केवल�1, 2 और�3
d) केवल�4
Correct Answer: A
Your Answer:
Explanation

Solution (a)

Approach:

यहां��वक�प�C को�समा�त�करना�आसान�है।�ले�कन�अ�य�तीन��वक�प�इतने�बारीक��से��दए�गए�ह���क��वक�प��को�ख�म�करना�क�ठन�है�जब
तक��क�कोई�इसे�सु�न��त�करने�के��लए�नह��जानता�है।�

आम�तौर�पर�, काल��मा�स��(जानबूझकर�तैयार��कए�गए) के�नाम�को�दे खकर�, कई�लोग�सोच�सकते�ह���क�यह�सच�हो�सकता�है��क�मा�स��ने


असमानता�पर�एक��कताब��लखी�होगी।�ले�कन�ऐसे�अ�य�धक�जो�खम�लेने�से�बच�।�

कथन��व�ेषण:

IASbaba
Score:
Web: http://ilp.iasbaba.com/
69.00 /
Email: ilp@iasbaba.com
Page 165 200
AIPTS/ILP VETERANS -
Exam Title :
2020 T...
Email : sahil.law.jmi@gmail.com
Contact : 9540210113

कथन�1 कथन�2 कथन�3 कथन�4

अस�य� अस�य� स�य� अस�य�

शां�त�का�आ�थ�क�प�रणाम�( The
असमानता�का�पुन: परी�ण�( I
एडम���मथ��ारा�रा���के�धन Economic रोजगार�, �याज�और�मु�ा�का
nequality
क���कृ�त�और�कारण�क��एक Consequences of the सामा�य��स�ांत�जे�एम�के�स
Reexamined) अम�य��सेन
जांच��लखा�गया�है।� Peace) जे�एम�के�स��ारा��लखा �ारा��लखा�गया�है�
�ारा��लखा�गया�है।�
गया�है�

QUESTION 93.
मच�ट��ड�काउंट�रेट�( MDR) से��या�अ�भ�ाय�है�

a) यह�एक��ाहक��ारा�अपने���त�ान��म���े�डट�और�डे�बट�काड��के�मा�यम�से��ाहक��से�भुगतान��वीकार�करने�के��लए�एक��ापारी��ारा
�लया�जाने�वाला�शु�क�है�
b) यह�एक�ब�क��ारा��कसी��ापारी��ारा��ाहक��से�उनके���त�ान��म���े�डट�और�डे�बट�काड��के�मा�यम�से�भुगतान��वीकार�करने�के��लए
�लया�जाने�वाला�शु�क�है�
c) यह�एक��ापारी��ारा��ाहक��को�उनके���त�ान��म���ब���पर�छू ट��दान�करने�के��लए��दया��कया�गया�शु�क�है�
d) इनमे�से�कोई�भी�नह��
Correct Answer: B
Your Answer:
Explanation

Solution (b)

मच�ट��ड�काउंट�रेट�एक�मच�ट�से�ब�क��ारा��ाहक��से�उनके���त�ान�म���े�डट�और�डे�बट�काड��के�मा�यम�से�भुगतान��वीकार�करने�के��लए
�लया�जाने�वाला�शु�क�है।�

अ�त�र��जानकारी:

• एमडीआर�काड��जारी�करने�वाले�ब�क�को�मुआवजा�दे ता�है, ब�क�जो��वाइ�प�ग�मशीन�(�वाइंट-ऑफ-सेल�ट�म�नल) और�नेटवक���दाता


जैसे�मा�टरकाड��या�वीज़ा�उनक��सेवा��के��लए�दे ता�है�
• RBI अ�धकतम�एमडीआर�शु�क��न�द���करता�है�जो���येक�काड��लेनदे न�पर�लगाया�जा�सकता�है।�

QUESTION 94.
भारतीय��रज़व��ब�क��न�न�ल�खत�म��से��कस�सं�था�के��लए�ब�कर�के��प�म��काय��करता�है�?

a) केवल�क���सरकार�के��लए�
b) क���सरकार�, सभी�रा�य�सरकार��और��थानीय�सरकार��के��लए�
c) ज�मू�और�क�मीर�को�छोड़कर�क���सरकार�और�सभी�रा�य�सरकार��के��लए�
d) �स��कम�को�छोड़कर�क���सरकार�और�सभी�रा�य�सरकार��के��लए�
Correct Answer: D
Your Answer:
Explanation

IASbaba
Score:
Web: http://ilp.iasbaba.com/
69.00 /
Email: ilp@iasbaba.com
Page 166 200
AIPTS/ILP VETERANS -
Exam Title :
2020 T...
Email : sahil.law.jmi@gmail.com
Contact : 9540210113

Solution (d)

वत�मान�म��, भारतीय��रज़व��ब�क��स��कम�को�छोड़कर�, भारत�म��सभी�रा�य�सरकार��(क��शा�सत��दे श�पुडुचेरी�स�हत) के��लए�ब�कर�के��प


म��काय��करता�है।�

अ�त�र��जानकारी:

चूं�क�RBI सरकार�का�ब�कर�है�, इस�लए�यह�सरकार�को�इसके��वतरण�से�संबं�धत��ा��तय��म���कसी�भी�कमी�को�पूरा�करने�के��लए


अ�पका�लक�ऋण��दान�करता�है।�

आरबीआई�रा�य�सरकार��को�अ�पका�लक�ऋण�भी��दान�करता�है।�

QUESTION 95.
�न�न�ल�खत�म��से�कौन�सा�कथन�RBI अ�ध�नयम�1934 क��" धारा�7" के�बारे�म��सही�है�?

1. यह�एक�ऐसा��ावधान�है��जसके�तहत�सरकार�आरबीआई�को�जन�हत�म��कुछ�कार�वाई�करने�के��लए��नद� श�दे �सकती�है�


2. ऐसे��नद� श�जारी�करने�के��लए�सरकार�को�आरबीआई�के�गवन�र�से�परामश��करने�क��आव�यकता�नह��है।�

नीचे��दए�गए�कूट�का�उपयोग�करके�सही�उ�र�चुन��

a) केवल�1
b) केवल�2
c) 1 और�2 दोन��
d) न�तो�1 और�न�ही�2
Correct Answer: A
Your Answer:
Explanation

Solution (a)

क���सरकार�समय-समय�पर�ब�क�को�इस�तरह�के��नद� श�दे �सकती�है�, जैसा��क�ब�क�के�गवन�र�के�परामश��के�बाद�, जन�हत�म��आव�यक�माना


जाता�है।�

इस�लए�कथन�1 सही�है�और�कथन�2 गलत�है।�

QUESTION 96.
�न�न�ल�खत�यु�म��पर��वचार�कर��

स�म�त�अ�य�ता�

1. भारत�के��लए�साव�ज�नक�ऋण�र�ज����पर�उ�च��तरीय�काय��बल: �बमल�जालान�
2. �ड�जटल�भुगतान�को�गहरा�करने�पर�उ�च��तरीय�स�म�त�: नंदन�नीलेकणी�
3. साव�ज�नक��े��के�ब�क��के��लए�तनाव��त�प�रसंप��य��के�पुनग�ठन�और�अ�धक�मू�य�बनाने�पर�स�म�त: सुनील�मेहता�

उपरो��म��से�कौन�सी�जोड़ी�सही�ढं ग�से�सुमे�लत�है�/ ह��?

a) केवल�1 और�2
b) केवल�2 और�3
c) 1, 2 और�3
d) केवल�2

IASbaba
Score:
Web: http://ilp.iasbaba.com/
69.00 /
Email: ilp@iasbaba.com
Page 167 200
AIPTS/ILP VETERANS -
Exam Title :
2020 T...
Email : sahil.law.jmi@gmail.com
Contact : 9540210113

Correct Answer: B
Your Answer:
Explanation

Solution (b)

कथन��व�ेषण:

Pair 1 Pair 2 Pair 3

अस�य� स�य� स�य�

साव�ज�नक��े��के�ब�क��के��लए�तनाव
भारत�के��लए�साव�ज�नक�ऋण�र�ज���
�ड�जटल�भुगतान�को�अ�धक�बढ़ाने�पर ��त�प�रसंप��य��के�पुनग�ठन�और
पर�उ�च��तरीय�टा�क�फोस��क�
हाई�लेवल�कमेट��क��अ�य�ता�नंदन अ�धक�मू�य�बनाने�क��उ�च-�तरीय
अ�य�ता�Y.M.Deosthalee �ारा
नीलेकणी��ारा��कया�गया�है।� स�म�त�क��अ�य�ता�सुनील�मेहता
क��गयी�है।�
�ारा�क��गयी�है।�

QUESTION 97.
�न�न�ल�खत�कथन��पर��वचार�कर��

1. �रज़व��ब�क�के�पास�दो�ब��क�ग�कंप�नय��के��वै��छक�समामेलन�( amalgamation ) को�मंजूरी�दे ने�के��लए��ववेकाधीन�श��याँ�ह��


2. �रज़व��ब�क�के�पास�एक�गैर-ब��क�ग�कंपनी�के�साथ�ब��क�ग�कंपनी�क���वै��छक�समामेलन�( amalgamation ) को�मंजूरी�दे ने�के
�लए��ववेकाधीन�श��याँ�ह�।�

ऊपर��दए�गए�कथन��म��से�कौन�सा�सही�है�/ ह��?

a) केवल�1
b) केवल�2
c) 1 और�2 दोन��
d) न�तो�1 और�न�ही�2
Correct Answer: A
Your Answer:
Explanation

Solution (a)

कथन��व�ेषण:

कथन�1 कथन�2

IASbaba
Score:
Web: http://ilp.iasbaba.com/
69.00 /
Email: ilp@iasbaba.com
Page 168 200
AIPTS/ILP VETERANS -
Exam Title :
2020 T...
Email : sahil.law.jmi@gmail.com
Contact : 9540210113

स�य� अस�य�

ये�श��यां�एक�गैर-ब��क�ग�कंपनी�के�साथ�एक�ब��क�ग�कंपनी�क�
�रजव��ब�क�के�पास�ब��क�ग��व�नयमन�अ�ध�नयम�, 1949
�वै��छक�समामेलन�तक�नह��होती�ह��, जहाँ�कंपनी�अ�ध�नयम�,
क��धारा�44 ए�के��ावधान��के�तहत�दो�ब��क�ग�कंप�नय��के
1956 क��धारा�391 से�394 के�बीच�समामेलन�का�संचालन�होता
�वै��छक�समामेलन�को�मंजूरी�दे ने�क���ववेकाधीन�श��याँ
है�, �जसके�संदभ��म��, समामेलन�क��योजना�को�उ�च��यायालय��ारा
ह�।�
अनुमो�दत��कया�जाना�है।�

अ�त�र��जानकारी:

1970 और�1980 क��ब��क�ग�कंप�नयाँ�(उप�म��का�अ�ध�हण�और�अंतरण) अ�ध�नयम�यह��दान�करते�ह���क�क���सरकार�, भारतीय��रज़व�


ब�क�( RBI) के�परामश��से�, �कसी�भी�रा�ीयकृत�ब�क�के��कसी�अ�य�रा�ीयकृत�ब�क�या�कोई�अ�य�ब��क�ग�सं�थान�के�साथ�समामेलन�के��लए
एक�योजना�बना�सकती�है।�

QUESTION 98.
�यायमू�त��बीएन��ीकृ�ण�स�म�त�को�हाल�ही�म���न�न�ल�खत�म��से��कसके�साथ�समाचार�म��दे खा�गया�है�?

a) डेटा�सुर�ा�
b) आरबीआई�का�अ�धशेष�भंडार�
c) सेबी�के�अ�धशेष�भंडार�
d) �वदे शी�सं�भु�बांड�( Foreign Sovereign Bonds )
Correct Answer: A
Your Answer:
Explanation

Solution (a)

�यायमू�त��बीएन��ीकृ�ण�स�म�त�ने�हाल�ही�म��(जुलाई�2019 म�) डेटा�सुर�ा�कानून�पर�अपनी��रपोट� ���तुत�क�।�

31 जुलाई�, 2017 को�, भारत�सरकार�ने��यायमू�त��बी.एन. �ीकृ�ण�, भारत�के�सव��च��यायालय�के�पूव���यायाधीश�क��अ�य�ता�म�


�वशेष���क��10- सद�यीय�स�म�त��नयु��क�।�" भारत�म���मुख�डेटा�सुर�ा�मु���क��पहचान�करने�और�उ�ह��संबो�धत�करने�के�तरीक��क�
�सफा�रश�करने�के��लए�" ।

IASbaba
Score:
Web: http://ilp.iasbaba.com/
69.00 /
Email: ilp@iasbaba.com
Page 169 200
AIPTS/ILP VETERANS -
Exam Title :
2020 T...
Email : sahil.law.jmi@gmail.com
Contact : 9540210113

Pic: https://economictimes.indiatimes.com/img/65171932/Master.jpg

QUESTION 99.
आ�थ�क�जनगणना�( Economic Census ) के�बारे�म���न�न�ल�खत�कथन��पर��वचार�कर��

1. यह�दे श�म��पहली�बार�अनौपचा�रक��े��के�आकार�का�पता�लगाने�के��लए�आयो�जत��कया�जा�रहा�है।�
2. सां��यक��और�काय��म�काया��वयन�मं�ालय�इस�जनगणना�का�संचालन�कर�रहा�है।�
3. इस�जनगणना�म��सभी�कृ�ष�और�गैर-कृ�ष�ग�त�व�धय��पर��वचार��कया�जाएगा।�

उपरो��कथन��म��से�कौन�सा�सही�है�/ ह��?

a) केवल�1 और�2
b) केवल�2

IASbaba
Score:
Web: http://ilp.iasbaba.com/
69.00 /
Email: ilp@iasbaba.com
Page 170 200
AIPTS/ILP VETERANS -
Exam Title :
2020 T...
Email : sahil.law.jmi@gmail.com
Contact : 9540210113

c) केवल�1 और�3
d) 1, 2 और�3
Correct Answer: B
Your Answer:
Explanation

Solution (b)

आ�थ�क�जनगणना�हाल�ही�म��खबर��म��थी।�

कथन�1 कथन�2 कथन�3

अस�य� स�य� अस�य�

दे श�म��पहली�बार�आ�थ�क�जनगणना�नह�
क��जा�रही�है।�

सरकार�ने�हाल�ही�म����पुरा�से�7 व� यह�सां��यक��और�काय��म�काया��वयन
आ�थ�क�जनगणना�(ईसी) शु��क�।�इसे मं�ालय�( MoSPI) �ारा�संचा�लत��कया
आ�थ�क�जनगणना�भारत�क��भौगो�लक
अग�त�और��सतंबर�म��अ�य�रा�य��और जा�रहा�है�
सीमा��के�भीतर���थत�सभी���त�ान��/
क��शा�सत��दे श��म��लॉ�च��कया�जाएगा।�
MoSPI ने�ईसी�के��लए�काया��वयन इकाइय��क��पूण��गणना�है।�
अब�तक�छह�आ�थ�क�स�सर��कए�गए�ह�: 1 एज�सी�के��प�म��कॉमन�स�व�स�स�टस��,
कृ�ष�आ�थ�क�जनगणना�म��शा�मल�नह�
977,1980,1990,1998,2005 औ CSC ई-गवन�स�स�व�स�इं�डया��ल�मटे ड
है।�
र�2013 (इले��ॉ�न�स�और�आईट��मं�ालय�के�SP
V) के�साथ�साझेदारी�क��है�
अथ��व�था�म��मह�वपूण���प�से�बड़े
असंग�ठत��े��क��सूचना�का�एकमा���ोत
ईसी�है।�

QUESTION 100.
‘ प�रसंप���पुन�न�मा�ण�कंप�नय��’ (ARC), के�बारे�म���न�न�ल�खत�म��से�कौन�सा�कथन�सही�है�?

1. ARCs, SARFAECI अ�ध�नयम�के�तहत��था�पत��कए�गए�ह��और�RBI �ारा��व�नय�मत�ह�।�


2. ARCs का�उ�े �य�सरकारी�संरचना��का�पुन�न�मा�ण�करना�और�ऐ�तहा�सक�मह�व�क��इमारत��का�नवीनीकरण�करना�है।�

नीचे�से�कूट�चुन�:

a) केवल�1
b) केवल�2
c) 1 और�2 दोन��
d) न�तो�1 और�न�ही�2
Correct Answer: A
Your Answer:
Explanation

IASbaba
Score:
Web: http://ilp.iasbaba.com/
69.00 /
Email: ilp@iasbaba.com
Page 171 200
AIPTS/ILP VETERANS -
Exam Title :
2020 T...
Email : sahil.law.jmi@gmail.com
Contact : 9540210113

Solution (a)

SARFAESI अ�ध�नयम�, ब�क��और��व�ीय�सं�थान��से�संप����ा�त�करने�के��लए�RBI �ारा��व�नय�मत�प�रसंप���पुन�न�मा�ण�कंप�नय��(


ARCs) क���थापना�का��ावधान�करता�है।�अ�ध�नयम�ब�क��और��व�ीय�सं�थान���ारा�प�रसंप���पुन�न�मा�ण�कंप�नय��(एआरसी) को��व�ीय
प�रसंप��य��क���ब���के��लए��दान�करता�है।�RBI ने�ARCs को��व�ीय�प�रसंप��य��क���ब���के��लए����या�पर�ब�क��को��दशा-�नद� श
जारी��कए�ह�।�

एसेट��रकं���शन�कंप�नय��(एआरसी) को�गैर-�न�पा�दत�आ��तय��(एनपीए) को�सुर��त�ऋणदाता��क��पु�तक��से�पुन�ा��त�करने�तथा�गैर-


�न�पा�दत�प�रसंप��य��(एनपीए) के�मू�य�को�अनलॉक�करने�के��लए�एक��णाली�लाने�के��लए�बनाया�गया�है।�भारतीय��रज़व��ब�क�( RBI),
ARCs के��लए�लाइस�स��दान�करता�है�तथा�ARCs को�SARFAESI अ�ध�नयम��ारा�सश���कया�गया�है।�

For more details of ARCs you can go through the following link: https://www.indiafilings.com/
learn/asset-reconstruction-company-arc-india/

IASbaba
Score:
Web: http://ilp.iasbaba.com/
69.00 /
Email: ilp@iasbaba.com
Page 172 200

You might also like